peds all 1

¡Supera tus tareas y exámenes ahora con Quizwiz!

Presentation of volvulus

*Nonspecific signs* that include vomiting and colicky abdominal pain

Presentation of PDA

- "Machine-like" murmur - Wide pulse pressure - Bounding pulses

Presentation of coarctation of the aorta

- *CHF & respiratory distress* (in 1st few months of life) - *Different pressures/pulses b/w upper & lower extremities* (↑ pressure in upper extremities, ↓ pulse in lower extremities)

Presentation of intussussception

- *Colicky abdominal pain* - *Bilious vomiting* - *Currant jelly stool* - Right quadrant sausage-shaped mass - Neurologic signs

Differentiating Croup from Epiglottitis

- *Croup*: hypoxia on presentation - *Epiglottitis*: hypoxia imminent

Best initial test for duodenal atresia, volvulus, & intussussception

- *Duodenal atresia & volvulus*: AXR - *Intussussception*: ultrasound

2 scenarios in which *currant jelly* is seen

- *Klebsiella pneumonia*: currant jelly sputum from lungs - *Intussussception*: currant jelly stool

Difference in how patients w/ mild and severe cases of diarrhea/gastroenteritis are rehydrated

- *Mild*: oral fluids - *Severe*: IV fluids

Difference b/w "true" & "false" diverticulum

- *True*: involves all layers of bowel - *False*: does not involve all layers of bowel

APGAR scoring for Grimace

- 0: No response - 1: Grimace/feeble cry - 2: Sneeze/cough

APGAR scoring for Activity

- 0: None - 1: Some flexion - 2: Active movement

APGAR scoring for Pulse

- 0: aystole or <60 bpm - 1: >60 but <100 bpm - 2: >100 bpm

APGAR scoring for Appearance

- 0: blue all over - 1: normal except extremities - 2: normal all over

Factors produced by Vitamin K

- 10, 9, 7, 2 - Proteins C and S

Presentation of Vitamin B9 (folate) deficiency

- Megaloblastic anemia - Hypersegmented neutrophils

Presentation of Vitamin B12 (cyanocobalamin) deficiency

- Megaloblastic anemia - Hypersegmented neutrophils - Peripheral neuropathy of the dorsal column tracts

Types of diaphragmatic hernias

- Morgagni - Bochdalek

Presentation of Varicella

- Multiple highly pruritic vesicular rash that begins on the face - Possible fever and malaise

Ophthalmia neonatorum can be attributed most commonly to what organisms?

- N. gonorrhoeae - C. trachomatis

Presentation of Wilms tumor

- Nausea, vomiting, abdominal pain, constipation - Large palpable abdominal mass - Aniridia

Elevated AFP levels indicated both

- Neural tube defects - Abdominal wall defects

Why is insulin therapy not the first step in management of an IDM?

- Newborn has ↑ serum glucose, hyperinsulinemia in utero - Upon delivery, maternal glucose no longer available - Hyperinsulinemia causes ↓ in serum glucose upon delivery

Indications for intubation and ABG analysis of a newborn

- Not breathing - Respiratory distress

W/ cryptorchidism, what is indicated to bring the testicle down into the scrotum after the age of 1? Why?

- Orchiplexy - Avoid sterility

Further findings in an IDM associated w/ macrosomia

- Organomegaly (except for brain) - Polycythemia/hyperviscosity - +/- Shoulder dystocia/brachial plexus palsy

Diagnostic tests for Whooping cough

- PCR of nasal secretions - ELISA for B. pertussis toxin

What is oxygenation of blood dependent on in transposition of the great vessels?

- PDA - ASD - VSD

Presentation of rubella

- PDA - Cataracts - Deafness - Hepatosplenomegaly - Thrombocytopenia - Blueberry muffin rash - Hyperbilirubinemia

Presentation of slipped capital femoral epiphysis

- Painful limp - Externally rotated leg

Most common etiology of Croup

- Parainfluenza virus types 1 and 2 - RSV (2nd most common)

Treatment of pharyngitis

- Penicillin (oral, 10 days) - Macrolides (if pencillin allergic)

Signs of hypoxia

- Peripheral cyanosis - Accessory muscle use

Presentation of Vitamin B6 (pyridoxine) deficiency

- Peripheral neuropathy - Must be given w/ INH

Contraindications to barium enema in patient w/ intussussception

- Peritoneal signs - Shock - Perforation

Presentation of CMV

- Periventricular calcifications w/ microcephaly - Chorioretinitis - Hearing loss - Petechiae

Treatment of rickets

- Phosphate, calcium, vitamin D (ergocalciferol & 1,25(OH2)) - Annual blood vitamin D monitoring

Routine management of newborn

- Physical examination - APGAR scoring - Eye care - Disease screening & prevention

Most common conditions causing sepsis

- Pneumonia - Meningitis

Presentation of Vitamin A deficiency

- Poor night vision - Hypoparathyroidism

Presenting features of necrotizing enterocolitis

- Premature, low birth weight - Fever, vomiting, abdominal distention

Types of ASDs

- Primum - Secundum - Sinus venosus

Causes of oligohydramnios

- Prune belly - Renal agenesis

Presentation of Vitamin A toxicity

- Pseudotumor cerebri - Hyperparathyroidism

Causes of low PO2 resulting in PDA

- Pulmonary compromise due to prematurity - Areas of high altitude

Presentation of syphilis

- Rash on palms & soles - Snuffles - Frontal bossing - Hutchinson 8th nerve palsy - Saddle nose

Treatment of CAH

- Replace mineralocorticoids & glucocorticoids - Genital reconstructive surgery

Treatment of Legg-Calvé-Perthes disease

- Rest + NSAIDs - Surgery on both hips

3 major causative agents of viral infectious diarrhea

- Rotavirus - Adenovirus - Norwalk virus

Intussussception is associated with?

- Rotavirus vaccine (previously used) - Henoch-Schonlein purpura

Presentation of kernicterus

- Seizures - Hearing loss - Hypotonia - Choreoathetosis

How is CAH diagnosed at birth?

- Serum electrolytes - ↑ 17-OH progesterone levels

Presentation of Truncus arteriosus

- Single S2 is heard - Systolic ejection murmur - Bounding peripheral pulses - Severe dyspnea - Early/frequent respiratory infections

Pathophysiology of transient hyperbilirubinemia

- Splenic breakdown of excess RBCs that carry Hgb F - ↑ Hgb → ↑ bilirubin

Most common organisms associated w/ late onset sepsis

- Staphylococcus - E. coli - Group B Streptococcus

Presentation of Fifth disease/Erythema infectiosum

- Starts w/ fever + URI - Progresses to rash w/ "slapped cheek" appearance

Diagnostic tests for diarrhea and gastroenteritis

- Stool for blood and leukocyte count - Stool cultures and ova/parasite exam - Sigmoidoscopy

Upper GI series findings associated w/ pyloric stenosis

- String sign - Shoulder sign - Mushroom sign - Railroad track sign

Common pediatric risk factors for vitamin D-*deficient* rickets

- Sunless environment - Low milk intake

What is indicated for all patients w/ symptomatic ASD?

- Surgery - Transcathether closure

What are the TORCH infections?

- T: toxoplasmosis - O: other infections such as Syphilis - R: rubella - C: cytomegalovirus - H: herpes simplex virus

Cyanotic heart defects dependent on PDA

- TGV - Hypoplastic left heart syndrome

Effect of hypocalcemia in an IDM

- Tetany - Lethargy

Set of congenital defects seen w/ VACTERL syndrome

- Vertebral anomalies - Anal atresia (imperforate anus) - Cardiovascular anomalies - Tracheoesophageal fistula - Esophageal atresia - Renal anomalies - Limb anomalies

Presentation of esophageal atresia + TEF

- Vomiting w/ first feeding - Coughing/choking/cyanosis due to TEF - History of polyhydramnios - Recurrent aspiration pneumonia

2 forms of Total anomalous pulmonary venous return (TAPVR)

- W/ obstruction of venous return - Without obstruction of venous return

Diagnostic features of Whooping cough

- Whooping inspiration - Vomiting - Burst blood vessels in eyes

Presentation of Vitamin K deficiency

- ↑ PT/INR - S/Sx of mild-severe bleeding - Analogous to warfarin therapy

Lab values correlated to Scarlet fever

- ↑ antistreptolysin O titer - ↑ ESR/CRP

Most common complications of PDA later in life

- ↑ occurrence of respiratory infxns - Infective endocarditis

Rationale for squatting in Tetralogy of Fallot patients

- ↑ preload & SVR - ↓ right-to-left shunting - ↑ pulmonary blood flow - ↑ blood oxygen saturation

Classic findings w/ Cystic fibrosis

- ↑ sweat chloride - Presence of mutations in CFTR gene - Abnormal functioning in at least 1 organ system

Treatment of G6PD

- ↓ oxidative stress - Specialized diets

What is the maximum single dose of Atropine that should be given to a 6 year old child

0.5 mg

Incidence of congenital hypothyroidism

1 in 4000 infants

Typical duration of infectious diarrhea caused by *Norwalk virus*

1-2 days

Treatment of Epiglottitis

1. *Intubate* in OR 2. *Ceftriaxone* 7-10 days 3. *Rifampin* to all close contacts

3 forms of CAH

1. 21-hydroxylase (90% of cases) 2. 17-hydroxylase 3. 11-β-hydroxylase

Management of necrotizing enterocolitis

1. Antibiotics 2. NPO, IV fluids 3. NGT 4. Surgery (w/ failure of medical management)

Name the stages of Whooping cough

1. Catarrhal stage 2. Paroxysmal stage 3. Convalescent stage

2 major forms of brachial palsy

1. Duchenne-Erb 2. Klumpke

Newborns make three major physiological adaptations at birth that are necessary for survival

1. Emptying fluids from their lungs & beginning ventilation 2. Changing their circulatory pattern 3. Maintaining body temperature

Management of duodenal atresia

1. IV fluids 2. NG tube 3. Surgical duodenostomy

Most important initial steps in management of intussussception

1. IV fluids + electrolytes 2. NG tube

Management of intussussception

1. IV fluids + electrolytes 2. NG tube 3. Barium enema 4. Observation 5. Emergent surgical intervention (w/ failure of barium enema)

3 types of skull fractures in the newborn

1. Linear 2. Depressed 3. Basilar

3 etiologies of holosystolic murmurs

1. Mitral regurgitation 2. Tricuspid regurgitation 3. VSD

7 newborn reflexes

1. Suckling reflex 2. Grasping reflex 3. Babinski reflex 4. Rooting reflex 5. Moro reflex 6. Stepping reflex 7. Superman reflex

3 main etiologies of rickets

1. Vitamin D-*deficient* 2. Vitamin D-*dependent* 3. X-linked hypophosphatemic rickets

Why are patients w/ intussussception typically observed for 24 hours after therapy?

10% recur within 24 hours

Q023. gross motor skills at 4-5 mo

A023. rolls front to back and back/front; sits supported

Q024. gross motor skills at 6 mo

A024. sits unsupported

Q025. gross motor skills at 9 mo

A025. crawls; cruises; pulls to stand

Q026. gross motor skills at 12 mo

A026. walks alone

Q027. gross motor skills at 15 mo

A027. walks backwards

Q028. gross motor skills at 18 mo

A028. runs

Q029. gross motor skills at 24 mo

A029. walks well up and down stairs

Q030. gross motor skills at 3 yrs

A030. rides tricycle; throws ball overhand

Q031. gross motor skills at 4 yo

A031. alternates feet going down stairs; skips

Q032. fine motor skills at 1 mo

A032. follows eyes to midline

Q033. fine motor skills at 3 mo

A033. hands open at rest

Q034. fine motor skills at 4-5 mo

A034. grasps with both hands together

Q035. fine motor skills at 6 mo

A035. transfers hand to hand, reaches with either hand

Q036. fine motor skills at 9 mo

A036. pincher grasp; finger feeds

Q037. fine motor skills at 12 mo

A037. throws, releases objects

Q038. fine motor skills at 15 mo

A038. builds 2 block tower

Q039. fine motor skills at 18 mo

A039. feeds self with utensils

Q040. fine motor skills at 24 mo

A040. removes clothing; builds 5 block tower

Q041. fine motor skills at 3 yrs

A041. draws circle

Q042. fine motor skills at 4 yrs

A042. catches ball; dresses alone

Q043. fine motor skills at 5 yrs

A043. ties shoes

Q044. Simple febrile seizure

A044. between 6mo - 6yr; tonic clonic; associated with fever &gt;100.4; seizure lasts &lt;15 mins; only 1 seizure in 24 hrs; minimal post-ictal state

Q045. complex febrile seizure

A045. 6 mo - 6 yrs; focal seizure; &gt;15 mins; &gt;1 seizure/24 hrs

Q046. management of simple febrile seizure

A046. determine the source of the fever, otherwise, no other w/u is needed

Q047. management of complex febrile seizure

A047. full w/u should be done, but no anti-epileptics, no eeg needed; if patient is &lt;18 mo, LP

Q048. eeg abnormality associated with infantile spasm

A048. hypsarrythmia

Q049. definition of recurrent abdominal pain

A049. &gt;3x in 3 mo

Q050. #1 cause of abdominal pain

A050. gastroenteritis

Q051. mesenteric lymphadenitits

A051. persistent pain following an infection

Q054. complication of Henoch-Schonlein Purpura

A054. intussusception; kidney probs

Q055. clinical features of kawasaki

A055. CRASH and BURN; Conjunctivitis; Rash (on trunk mostly); Aneurysm (coronary); Skin peels off, Strawberry tongue; Hands/Feet edema; BURN = FEVER (x 5d)

Q056. phases of kawasaki disease

A056. acute; subacute (aneurysm formation); convaslescent; takes 2-3 months to resolve

Q057. tx of kawasaki

A057. ASA; IVIG

Q058. when is colicky pain associated with

A058. constipation

Q059. what type of stool is seen with bacterial enterocolitis

A059. bloody, mucinous stool

Q060. when will an appendix perforate in appendicitis

A060. w/i 36 hrs

Q061. which infection can clinically mimic appendicitis

A061. yersinia; campylobacter

Q062. what imaging study for appendicitis

A062. ct

Q063. meds used to tx of perforated appendix

A063. amp, gent, flagyl

Q064. clinical presentation of intussusception

A064. currant jelly stools; lethargy; palpable tubular mass; paucity of gas on xr or evidence of obstruction

Q065. imaging most specific for intussusception

A065. barium (or air) enema; is also therapeutic

Q066. most common location for intussusception

A066. ileocolic; can also develop at meckel's divertic

Q067. major complication of intussusception

A067. there is impaired venous return so bowel edema develops -- &gt; ischemia, necrosis --&gt; perforation

Q068. etiology of intussusception

A068. ileum invaginates into colon at ileocecal valve; a previous viral infection --&gt; hypertrophy of the peyer's patches... this can develop into a lead point; Henoch-Schonlein Purpura can be association with an ileal- ileal intussusception

Q069. tx of intussusception

A069. must do fluid resusc first, if needed; hydrostatic reduction with air/barium

Q070. recurrence rate of intussusception

A070. 0.15

Q071. at what age would a patient present with pyloric stenosis

A071. 1-3 mo

Q072. which medicaation can be associated with pyloric stenosis

A072. erythromycin

Q073. best imaging for pyloric stenosis

A073. u/s; will also see a string sign ugi study

Q074. tx of pyloric stenoSIS

A074. MUST correct fluids and lytes first!; then pyloromyotomy

Q102. complications of developmental dysplasia of hip

A102. Avascular necrosis; degenerative arthritis of hip

Q1020. Next step:; Foreign Body aspiration removal

A1020. RIGID bronchoscopy

Q1021. Define:; Meconium Ileus

A1021. Failure to pass meconium within the first 24 hours of birth

Q1022. MCC of high fever and chills or sepsis in Sickle cell patient

A1022. Strep Pneumococcus; (encapsulated organism that cannot be handled well due to asplenia)

Q1023. Dx:; Sickle cell patient with pain in the hip including restriction of abduction and internal rotation

A1023. Avascular necrosis; (common in sickle cell adolescent)

"Q1024. Dx:; premature neonate with ""increased gastric residues"""

A1024. Necrotizing Entercolitis

Q1025. Dx:; child born normal and asymptomatic until teens, when he began to have delayed muscle relaxation, progressive muscle weakness, thinning cheeks and atrophy of thenar and hypothenar eminences

A1025. Myotonic Muscular Dystrophy

Q103. metatarsus adductus

A103. dorsiflexion and plantarflexion are UNRESTRICTED (diff from clubfoot);; heels go out and toes go in

Q104. tx of metatarsus adductus

A104. stretching or a brace; surgery not usualy needed

Q343. Pyloric Stenosis - Dx

A343. Abdominal US - diagnostic; barium studies - string sign, pyloric beak; hypochloremic, hypokalemic; metabolic alkalosis

Q183. what meds are associated with pseudotumor cerebri

A183. tetracyclines; corticosteroids

Q184. define encephalopathy

A184. generalized cerebral dysfxn, ms change, disorientation

Q185. risk factors for developing retinopathy of prematurity

A185. Body weight &lt;1250 g; age &lt;32 weeks; mechanical ventilation; need for supplemental o2

Q186. causes of leukocoria

A186. retinoblastoma (--&gt; death and visceral metastasis in all cases); cataracts (most common cause); retinopathy of prematurity

Q187. amblyopia

A187. visual impairment not corrected by glasses and not due to an ocular lesion; often from strabismus

Q188. tx of amblyopia

A188. occlusion of better seeing eye forces development of affected eye and visual cortex of affected eye

Q189. at what age will tx not be successful in ambylopia

A189. after 8yo

Q190. tx of retinoblastoma

A190. enucleation (removal of the affected eye); radiation tx; chemo

Q191. complications of neonatal cataracts

A191. if not remoed by 3-4 months, there is irreversible amblyopia

Q192. course of retinopathy of prematurity

A192. most regress spontaneously; if not, cryotherapy can be performed to reduce progression; are still at risk for amblyopia even if treated

Q244. what effect does phenobarb have on jaundice

A244. it improves it by increasing gluocoronyl transferase

Q245. cancer that is associated with (germline) retinoblastoma

A245. osteosarcoma is most common; melanoma; squamous cell ca

Q246. conditions associated with aniridia

A246. congenital glaucoma; surge-weber; marfan; neurofibromatosis

Q247. presentation of malrotation

A247. obstruction; typically normal for first few days of life than malrotation worsens --&gt; abdominal fullness, especially in ruq --&gt; bilious vomiting --&gt; ischemia and necrosis

Q248. #1 malrotation

A248. volvulus

Q298. what effect does sepsis have on bilirubin

A298. disrupts the BBB so it can cause diffusion of bili into the brain

Q299. what effect does temp have on bili

A299. cold temperature can --&gt; bili dissociation from albumin

Q344. Pyloric Stenosis - Tx

A344. First - hydration; correct acid-base &amp; electrolyte abnormalities; NG tube - possible; longitudinal pyloromyotomy

Q339. Intussusception - Dx

A339. Abdominal XR; abdominal US; air contrast barium enema; CBC

Q340. Intussusception - Tx

A340. Correct volume &amp; electrolytes; check CBC; air-contrast barium enema - diagnostic and therapeutic; surgical reduction or resection (if gangrenous)

Q341. Pyloric Stenosis - What is it

A341. Hypertrophy of pyloric sphincter; 1st-born males more affected

Q342. Pyloric Stenosis - History/PE

A342. 1st 2 wks - 4 mos. of life; nonbilious emesis =&gt; projectile emesis after each feeding; so, babies feed well initially =&gt; malnutrition &amp; dehydration; palpable olive-shaped, mobile, NT epigastric mass; visible gastric peristalsis

CXR findings associated w/ Hypoplastic left heart syndrome

Globular-shaped heart w/ pulmonary edema

Infants and children have a limited store of ____ and ____

Glycogen; glucose

Cause of Wilms tumor

Hemihypertrophy of one kidney due to its increased vascular demands

Characteristics of G6PD deficiency

Hemolytic crisis

What is another name for vitamin K deficient bleeding?

Hemorrhagic disease of the newborn

Most likely cause of conjunctivitis after 3 weeks or more

Herpes

Etiology of Roseola

Herpesvirus types 6 and 7

Treatment of Caput succedaneum & Cephalohematoma

Gradual improvement without treatment over weeks-months

Findings w/ manometry for Hirschsprung disease

High pressures in anal sphincter

Presentation of Diphtheria

Gray highly vascular pseudomembranous plaques on the pharyngeal wall

Vital signs in newborn are always higher or lower?

Higher

How do you do the Barlow maneuver?

Hip adduction + posterior force through knee

Positive Barlow maneuver

Hip dislocation

Why must you ultrasound the contralateral testicle w/ varicocele?

It is a bilateral disease

16-month-old boy, bright red blood in diaper, mild mass palpated in middle left quadrant. What is the most accurate test for this condition?

Meckel's scan [technetium-99m pertechnetate (99mTc) scan]

What are some of the anatomical differences between children and adults?

Larger tongues with smaller and narrower airways

What side do Bochdalek diaphragmatic hernias most commonly occur?

Left side

Condition in which there is a congenital lack of innervation of the distal bowel by the Auerbach plexus

Hirschsprung disease

Important risk factor for Epiglottitis

History of vaccination delinquency

The most common physiology seen in neonates with congenital heart disease involves a(n)

Left to right shunt

How does right ventricular hypertrophy occur in the setting of VSD?

Left-to-right shunt

What is the most likely diagnosis in: a 12-year-old boy found unconscious, awoke on ride to hospital without confusion, and history of uncle who died from "heart condition"?

Long QT syndrome

A 14 year old adolescent is anxious and has tremors and chest pain after smoking crack cocaine. His vital signs are BP 180/100 mm Hg, P 130/min, and RR 20/min. What drug is indicated

Lorazepam

Physical examination finding of Epiglottitis

Hot cherry-red epiglottis

Are 17ɑ-hydroxylase deficiency patients hyper- or hypotensive?

Hypertensive

Metabolic imbalance associated w/ pyloric stenosis

Hypochloremic, hypokalemic metabolic alkalosis

What life threatening condition may be found in a child after ingestion of alcohol

Hypoglycemia

Electrolyte abnormalities associated w/ 17ɑ-hydroxylase deficiency

Hypokalemia

Syndrome consisting of left ventricular hypoplasia, mitral valve atresia, and aortic valve lesions

Hypoplastic left heart syndrome

Reasoning for sending stool in for cultures and ova/parasite exam in the setting of gastroenteritis

Identify the causative agent

Most common cause of pyloric stenosis

Idiopathic

The patient may be very dry. An antidote is a cholinergenic

If a patient ODs on a anticholinergic EX) Atropine , what may they present as?

Best initial diagnostic test for toxoplasmosis

IgM

Why are newborns at most risk for Vitamin K deficient bleeding?

Immature livers do not utilize vitamin K to develop clotting factors

Treatment of gastroschisis

Immediate surgical intervention w/ gradual introduction of bowel and silo formation

What is the best treatment for Duchenne Erb paralysis?

Immobilization

Q376. Atrial Septal Defect (ASD) - Dx

A376. Echo with color flow Doppler - diagnostic; ECG - right-axis deviation; CXR - cardiomegaly and increased pulmonary vascular markings

______ usually becomes evident within the first week of life, with a primary presenting sign of emesis that is filled with bile

Intestinal malrotation

Source of lethargy in Vitamin K deficient bleeding

Intracranial bleeding

Q474. X-linked Agammaglobulinemia; (Bruton's Disease) - What is it

A474. B-cell defect; boys only; may present &lt; 6 mos. of age; at risk for life-threatening Pseudomonas infections

Age x 2 + 70

Lowest Systolic BP

Q475. X-linked Agammaglobulinemia; (Bruton's Disease) - Dx

A475. No B cells; low levels of all Ab classes

Q476. X-linked Agammaglobulinemia; (Bruton's Disease) - Tx

A476. IVIG; prophylactic Antibiotics

Q477. Common Variable Immunodeficiency - What is it

A477. Ig levels drop in 2nd-3rd decade of life; increased risk of lymphoma; increased risk of autoimmune dis.

Q478. Common Variable Immunodeficiency; Dx

A478. Ig levels; antibody titers

Q479. Common Variable Immunodeficiency; Tx

A479. IVIG; prophylactic Antibiotics

Q480. IgA Deficiency - What is it

A480. MC immunodeficiency; usually asymptomatic; may have recurrent infections

Q514. With what immunodeficiency will patients have severe, frequent bacterial infections; chronic candidiasis, and opportunistic organisms?

A514. Severe combined immunodeficiency

Q515. What is the treatment for SCID?

A515. PCP prophylaxis,; bone marrow transplant or stem cell transplant,; IVIG for antibody deficiency

Q516. With what immunodeficiency will patients present with eczema, increased IgE/IgA, decreased IgM, and thrombocytopenia?

A516. Wiskott-Aldrich syndrome

Q517. What is the classis presentation of Wiskott-Aldrich syndrome (3)

A517. Bleeding,; eczema,; recurrent otitis media

Q518. With what immunodeficiency will patients present with anemia, lymphadenopathy, hypergammaglobulinemia, and infections with catalase-positive organisms?

A518. Chronic granulomatous disease

Q519. Pts with chronic granulomatous disease require daily treatment with what antibiotic?

A519. TMP-SMX

Q520. With what immunodeficiency will patients present with oculocutaneous albinism, neuropathy, neutropenia, and increased incidence of overwhelming infections with S. pyogenes, S. aureus, and Pseudomonas?

A520. Chediak-Higashi syndrome

Q521. What is the defect in Chediak-Higashi syndrome?

A521. Neutrophil chemotaxis (Autosomal recessive)

Q522. What is the diagnostic test for chronic granulomatous disease?

A522. Nitroblue tetrazolium test

Q523. What immunodeficiency presents with recurrent episodes of angioedema lasting 2-72 hours and provoked by stress or trauma

A523. C1 esterase deficiency (autosomal dominant)

Q524. What type of immunodeficiencies tend to present at 1-3 months?

A524. T-cell deficiencies

Q525. With what immunodeficiency are patients unable to form membrane attack complexes

A525. Terminal compliment deficiency (C5-C9)

Q526. With what immunodeficiency will patients get recurrent miningococcal or gonococcal infections?

A526. Terminal compliment deficiency (C5-C9)

Q527. What type of immunodeficiencies are characterized by mucous membrane infections, gram-negative enteric organisms, and delayed umbilical cord separations?

A527. Phagocyte deficiencies

Etiology of Diphtheria

Membranous inflammation of the pharynx due to Corynebacterium diphtheriae

Q528. What type of immunodeficiencies are characterized by recurrent bacterial infections with encapsulated organisms

A528. Complement deficiencies

Q529. What are the subacute-phase manifestations of Kawasaki disease? (2)

A529. Thrombocytosis and increased ESR

-size of small finger -Braslow tape - (age in years +16) / 4

Methods for figuring out tube size

Most common location of volvulus in children

Midgut, specifically the ileum

stridor audible with stethoscope

Mild croup presents with....

13-15 GCS

Mild to GCS

9-12 GCS

Moderate GCS

Brachial palsy is most commonly seen in

Macrosomic infants of diabetic mothers

Reasoning for lifelong steroid administration in CAH patients

Maintain adequate levels of mineralocorticoid & glucocorticoid levels

What are some ways to improve communication with pediatric patients?

Make eye contact providing reassurance

Hirschsprung disease is more common in males or females?

Males (4:1 ratio)

Best initial diagnostic test for rubella

Maternal IgM status + clinical diagnosis

Most accurate diagnostic test for Rubeola/Measles

Measles IgM antibodies

Purpose of APGAR

Measures need and effectiveness of resuscitation

What must be assessed before beginning a focused assessment on a pediatric trauma patient?

Mechanism of injury and mental status

Only true congenital diverticulum in which vitelline duct persists in the small intestinal tract

Meckel's diverticulum

What must you consider when presented w/ painless red blood per rectum in a male child under age 2?

Meckel's diverticulum

Reflex where arms spread symmetrically when the baby is scared

Moro reflex

What does the APGAR score not predict?

Mortality

Q631. Dx:; newborn has a mass on the right anterior, superior chest and crepitus on exam; Tx?

A631. Clavicle fracture (MC newborn fracture); Tx: None (will heal on its own)

Q632. MC birthing brachial plexus injury?; what nerve roots?; another name for this injury?

A632. Erb-Duchenne; roots: C5-C6 (C4: if ipsilateral diaphragmatic paralysis also); another name: "waiter's tip" - wrist flexed

Q661. When should Ampicillin be added to the treatment of meningitis?; (2); When should Vancomycin be added?

A661. Ampicillin:; 1. Listeria (as bug); 2. Neonates; Vancomycin: Pneumococcus

Q662. Dx:; Lymphocytic meningitis, rash on wrists and ankles moving centrally; Tx?

A662. Rocky Mountain Spotted Fever; Tx: Doxycycline (and Ceftriaxone for meningitis prophylaxis)

Q663. Differential for Lymphocytic meningitis and next test for each; (3)

A663. Test: IgG and IgM Serology; 1. Rocky Mt. Spotted Fever; 2. Lyme disease; Test: India Ink (or cryptococcal antigen); 3. Cryptococcus

Q664. Treatment for meningitis caused by Cryptococcus

A664. Amphotericin; (then Fluconazole to take home)

Q665. What are the causes of CSF with very high protein and very low glucose?; (2)

A665. 1. TB; 2. fungal

Q666. What should be given prophylactically for a sneaker puncture of a pin that enters the foot?; For what bug?

A666. TMP-SMX; for: Pseudomonas

Q690. What is the specific bug that produces a positive on throat swab culture?; Tx? (3 possible); (2) possible complications?

A690. Group A Strep (A not B); Tx:; 1. Penicillin; 2. Erythromycin; 3. Clindamycin; complications:; 1. Rheumatic Fever; 2. Glomerulnephritis

Q821. What GI complaint can HENOCH-SCHONLEIN PURPURA lead to causing sevre morbidity?

A821. Intussusception

Q822. What is the Tx for a sickle cell patient who presents with anemia and an enlarged spleen?

A822. Blood Transfusion

Q823. What (5)* medications can cause Thrombocytopenia?

A823. PT CCS:; 1. Phenytoin; 2. TMP-SMX; 3. Chloramphenicol; 4. Carbamazepine; 5. Sulfonamides

Q825. How many mL/kg water loss is considered Mild, Moderate and Severe?

A825. Mild = &lt; 50mL/kg; Moderate = 50 - 99mL/kg; Severe = &gt; 100mL/kg

What mechanism of injury accounts for the largest number of trauma deaths in children

Motor vehicle crashes

Source of bright red blood per rectum in Vitamin K deficient bleeding

Mucosal bleeding

What can occur w/ gastroschisis?

Multiple intestinal atresias

Activity in APGAR measures

Muscle tone

Sign observed on upper GI series in pyloric stenosis characterized by *hypertrophic pylorus against the duodenum*

Mushroom sign

What closes a PDA?

NSAIDs (specifically, Indomethacin)

Pedi's have what size airway than adults

Narrow airways

Where do aortic lesions radiate to?

Neck

Condition seen in premature infants where bowel undergoes necrosis and bacteria invade the intestinal wall

Necrotizing enterocolitis

Most likely cause of conjunctivitis from days 2-7

Neisseria Gonorrhoeae

For the first 28 days of life, children are most correctly referred to as a(n)

Neonate

Jaundice usually occurs in

Neonates

Sepsis usually occurs in

Neonates

Are antibiotics effective in treating Diphtheria?

No

Are endoscopy, CT scan, or repeating hemoglobin useful in Meckel's diverticulum?

No

Can an NG tube be passed into the stomach of a patient w/ esophageal atresia?

No

Is a low score on APGAR associated with future cerebral palsy?

No

Is renal agenesis compatible w/ life?

No

Is vomiting a presenting symptom of volvulus, intussusception, biliary atresia, or pyloric stenosis?

No

Are 1-week-olds typically affected by CVAs, Crohn's disease, or Meckel's diverticulum?

No, too early

Hallmark feature of pyloric stenosis

Nonbilious projectile vomiting

Level with the surface or slightly sunken. It may also pulsate

Normal anterior fontanelle

Effect of 11β-hydroxylase deficiency on sex development of boys

Normal at birth

Effect of 17ɑ-hydroxylase deficiency on sex development of girls

Normal at birth

Effect of 21-hydroxylase deficiency on sex development of boys

Normal at birth

Epidemic cause of viral infectious diarrhea

Norwalk virus

Treatment of subconjunctival hemorrhage

Not indicated

Age group affected by slipped capital femoral epiphysis

Obese adolescents

Infants are ___ ___ breathers

Obligated Nose

A great deal of a child's physical exam can be done by

Observing the child's behavior

Which region of a child's head is most prominent?

Occipital

Too little fluid because fetus cannot urinate

Oligohydramnios

Sign that delineates a palpable mass the size of an olive felt in the epigastric region highly associated w/ pyloric stenosis

Olive sign

Defect in which intestines and organs form beyond the abdominal wall w/ a sac covering

Omphalocele

When is congenital hip dysplasia typically discovered?

On newborn exam screening

Why is a single S2 heard w/ Truncus arteriosus?

Only one semilunar valve is present

Most likely process underlying the following patient's stroke: 17-year-old boy who flew from Australia to NY presenting w/ facial droop, altered mental status, left-sided paralysis, took diphenhydramine, and swollen left calf muscle

Paradoxical emboli from deep leg veins

Etiology of Mumps

Paramyxovirus

Etiology of Rubeola/Measles

Paramyxovirus

Etiology of Fifth disease/Erythema infectiosum

Parvovirus B19

Treatment of congenital hip dysplasia

Pavlik harness

Thin layer under their back to raise their airway in a neutral inline position

Pedi less than 3 years old

A folded sheet or towl under the occiput to obtain a sniffing position.

Pedi over 3 years old

Pulse respiration, blood pressure, and temp a BP should cover 2/3 of arm

Pedi vital signs

smaller. This results in a higher potential for hypoxia.

Pediatric O2 reserve is...

Appearance: focus on the mental status and muscle tones Breathing: respiratory rate and respiratory effort Circulation: uses skin signs and color as well as capillary refill as indicators of the patient's circulatory status

Pediatric assessment triangle

18

Pediatric burn index: Head

18 per side

Pediatric burn index: Trunk

9

Pediatric burn index: whole arm

13.5 each

Pediatric burn index: legs

Generally larger with smaller faces and flat noses.

Pediatric head

Peds can maintain BP well and still be at risk for shock. Hypotension is a late sign.

Peds BP in shock

Peds have proportionally higher blood volume

Peds blood volume.

Presentation of Vitamin B3 (niacin) deficiency

Pellagra (diarrhea, dermatitis, dementia, death)

Treatment of syphilis

Penicillin

What does surgical intervention for choanal atresia involve?

Perforate membrane, reconnect pharynx to nostrils

Most likely diagnosis w/ *pear-shaped* cardiac x-ray findings

Pericardial effusion

Q907. A 2-yo boy undergoes end-to-end repair of an aortic coarctation. Seven years later he has HTN symptoms again. Tx?

A907. Angioplasty via balloon; (TOC for re-coarctation)

Q908. Dx:; Marfan features + Thromboembolic events; enzyme deficiency?

A908. Homocystinuria; enzyme: Cystathionine Synthase

Q909. Dx:; Onset of HA with focal neurological sx soon after acute otitis media?

A909. Brain Abscess

Q910. Dx:; A &lt;1-mo child with bilious vomiting, abdominal distention and passage of bloody stools

A910. Midgut Volvuvus

Q911. MCC of Jaundice in premature infant?

A911. TPN-induced jaundice

Q912. Dx:; a child presents with a wide-based gait, decreased vibratory and position sense in lower extremities, absent ankle jerk bilaterally, atrophy of cervical spinal cord and T-wave inversions

A912. Friedreich Ataxia; (Trinucleotide repeats; Autosomal Recessive)

Q913. Dx:; severe dehydration in neonate with HypoN, HyperK and HypoG plus Metabolic Acidosis

A913. Congenital Adrenal Hyperplasia; (21-hydroxylase deficiency)

Q914. A premature infant on a vent has signs of pneumonia. MC bug?

A914. Oxacillin-Resistant Staph Aureus; (MRSA would also cause skin pustules of periumbilical and diaper areas)

Q915. What is the difference in presentation of ABO incompatability and Rh incompatability?

A915. ABO: Fetal anemia and Jaundice; Rh: Hydrops (anemia, edema, ascites, cariomegaly and hepatomegaly)

Q916. What is the difference in Tx wth Ewings Sarcoma and Osteosarcoma?

A916. Ewings: Surgery, Chemo, Radiation; Osteosarcoma: Surgery &amp; Chemo only!

Q917. Aside from blood lead level, what else is increased in the blood with lead poisoning?

A917. Erythrocyte Protoporphyrin; (usually seen w/ &gt; 55mg/dL of lead)

Q918. What is the best way to test for acid-fast bacilli of TB?

A918. Early-morning Gastric Aspiration

Q919. Dx:; A 6-yo female child with constant leaking of urine

A919. Low implantation of Ureter; (into vagina)

Q920. What is the age of precocious puberty in boy and girl?; What is the best Initial test for Evaluation of Precocious puberty?

A920. Boy: &lt; 9-yo; Girl: &lt; 8-yo; test: Radiograph of Hand and Wrist to determine Bone Age

Q921. Tx:; Hereditary Angioedema

A921. Danazol (and Epi if needed)

Q922. Dx:; Superoxide deficiency in Macrophages

A922. Chronic Granulomatous Disease

Q923. Dx test:; Chronic Granulomatous Disease

A923. Nitroblue Tetrazolium

Q924. Dx:; Defect in neutrophil chemotaxis

A924. Chediak-Higashi syndrome

Q925. Dx:; MENTAL RETARDATION with rocker-bottom feet and clenched fist

A925. Edward's syndrome; (Trisomy 18)

Q926. Dx:; MENTAL RETARDATION with midline defects, cleft lip/palate

A926. Patau syndrome; (Trisomy 13)

Q927. Dx:; Tall, infertile, gynecomastia, microtestes

A927. Klinefelter's syndrome; (XXY)

Q928. Age:; Social smile

A928. 1 - 2 months

Q929. Age:; Rolls onto Back

A929. 4 months

Q930. Age:; Rolls onto Stomach

A930. 5 months

Q931. Age:; Walks alone

A931. 15 months

Q932. Age:; Copies Circle

A932. 3 years

Q933. Age:; Copies Cross or Square

A933. 4 years

"Q934. Tanner stage:; Penile ""growth spurt"""

A934. Tanner stage 3

Q935. Dx:; Vaccination that cannot be taken if allergic to eggs

A935. Influenza A

Q936. Dx:; Place of foreign body aspiration in a child that was standing

A936. Right Bronchi in Superior Segment of Lower Lobe

Q937. Dx:; Place of foreign body aspiration in a child that was supine or is an epileptic

A937. Right Bronchi in Posterior Segment of Lower Lobe

Q938. Dx:; possible Kidney problem in Turner's Disease

A938. Horseshoe kidney

Q939. Dx test:; Duodenal Atresia

A939. AXR; (Double bubble)

Q940. Dx test:; Rotavirus

A940. Immunoassay

Q941. Dx test:; steathorrhea plus protozoal cysts in stool

A941. Duodenal Aspirate

Q942. Bacterial Diarrhea:; Ulcerative colitis-like Sx plus Arthritis

A942. Campylobacter; (MCC of infectious diarrhea)

Q943. Bacterial Diarrhea:; Child with acute onset of RLQ pain (like appendicitis) plus diarrhea

A943. Yersinia

Q944. Bacterial Diarrhea:; Daycare outbreak plus child with diarrhea and new onset seizure

A944. Shigella

Q945. Dx test:; Tuberous Sclerosis

A945. CT scan of head; (Calcified periventricular tubers)

Q946. Tx:; West syndrome (Infantile spasms); (2)

A946. ACTH and Prednisone

Q947. Dx:; Anaphylaxis after blood trasnfusion

A947. IgA deficiency

Q948. Dx:; very low T- and B-cells (lymphopenia) with bacterial and fungal infections

A948. SCID

Q949. Prophylaxis needed for SCID

A949. PCP

Q950. Tx:; SCID

A950. Bone Marrow Transplant; (or stem cell transplant with IgG)

Q951. Dx:; baby less then 2-yo presents with tachypnea and wheezing

A951. Bronchiolitis (RSV)

Q952. Dx test:; Bronchiolitis (RSV)

A952. ELISA nasal washings

Q953. Bug:; Assoc with Guillain-Barre

A953. Campylobacter Jejuni

Q954. Dx:; Corneal clouding, cataracts, early renal failure

A954. Fabry's Disease

Q955. Dx:; Optic atrophy, spasticity, Globoid bodies in brain

A955. Krabbe's Disease

Q956. Dx:; Inspiratory stridor, barking cough

A956. Croup (Parainfluenza)

Q957. Tx:; Croup

A957. Steroids and humidified air

Q958. Dx:; Hearing loss, cataracts, microscopic hematuria

A958. Alport's syndrome

Q959. Dx:; salmon-colored rash with daily fever spikes, knee pain, leukocytosis, thrombocytosis, Inc ESR

A959. Still's Disease; (Systemic juvenile RA)

Q960. Dx:; Surfactant deficiency in preterm infant

A960. Respiratory Distress Syndrome; (RDS)

Q961. Dx:; Heart disorder assoc with DiGeorge syndrome

A961. Transposition of the Great Vessels

Q962. Dx:; short, Inc calcium, developmental delay, overly friendly, supravalvular aortic stenosis

A962. Williams syndrome

Q963. Dx:; Heart problem with Kartagener's syndrome

A963. Dextrocardia

Q964. Next step:; positive PPD

A964. Chest X-Ray; (before medicine)

Q965. First test:; Nasal polyps found in child

A965. Pilocarpine Sweat Test; (polyps in child = CF)

Q966. Dx:; Infant gets repeated pneumonias at beginning of life due to exhaustion of maternal IgG in system

A966. Transient Hypogammaglobinemia

Q967. Dx:; child with increased AA in urine, photosensitivity, ataxia and neuro problems

A967. Hartnup Disease

Q968. Dx:; 6-yo boy presents with dark pubic hair, enlarged penis/testis, growth spurt and acne

A968. Hypothalamic tumor; (without growth spurt may be 21-hydroxylase deficiency)

Q969. Dx:; Hereditary eye problem with progressive night blindness, field constriction and loss of acuity

A969. Retinitis Pigmentosa

Q970. First step:; Emergency airway in ER on patient &lt; 12-yo

A970. Needle Cricothyroidotomy

Q972. Dx:; Acanthocytes (spiny RBC)

A972. Abetalipoproteinemia

Q973. Child Brain Tumor:; Originates from cerebellar vermis and grows to the fourth ventricle

A973. Medulloblastoma

Q974. Child Brain Tumor:; Cyst and mural nodule assoc with vonHipple-Lindau

A974. Hemangioblastoma

Q975. Child Brain Tumor:; Cyst with a mural nodule in the cerebellum

A975. PiloCYSTIC Astrocytoma; (similar to Hemangioblastoma)

Q976. Dx:; Delayed passage of meconium, chronic constipation, FAILURE TO THRIVE, air in bowel

A976. Hirschprung's Disease

Q977. Dx:; High fever for 3 days, then macular rash

A977. Roseola

Q978. Virus assoc with Roseola

A978. HSV-6

Q979. another name for Measles

A979. Rubeola; (not Roseola)

Q980. First test:; Infant with seizures

A980. Serum chemistries

Q981. Dx:; BRBPR in infant 1 week to 3 months old

A981. Food Allergy-induced Colitis

Q982. Dx:; slight fever, sore throat, suboccipital and posterior auricular LN enlargement, rash starts on face and moves down body

A982. Rubella

Q983. MC genetic defect in CF

A983. CFTR-508 Deletion on chromosome 7

Q984. Dx:; Sickle cell patient with acute severe anemia without reticulocytes, but with normal platelets and WBC

A984. Aplastic Anemia

Q985. First step:; child is cyanotic from congenital diaphragmatic hernia

A985. Orogastric Tube placement; (with continuous suction to prevent bowel from decompressing lung)

Q986. Dx:; 2-week-old infant presents with jaundice. He is exclusively breastfed, his stool has a lighter color, has hepatomegaly and direct bilirubin of 4

A986. Biliary Atresia; (light stool, hepatomegaly and inc direct bilirubin)

Q987. An increase in what lab in a 2-week-old infant would lead to breastfed jaundice conclusion?

A987. Increased Indirect Bilirubin; (as high as 10 - 30mg/dL)

Q988. Next step:; after confirming Dx of Septic Arthritis

A988. Emergent Surgical Drainage

Q989. Dx:; Jaundice within 24 hours of life and an increased direct and indirect bilirubin, anemia, pallor, HSM and reticulocytosis

A989. Erythroblastosis Fetalis

Q990. Dx:; Jaundice appearing after the 5 days of life, but within the first week

A990. Neonatal Sepsis; (causing Jaundice)

Q991. Dx:; Child of 13-yo with femoral head off the metaphysis, with new bone formation

A991. Slipped Capital Femoral Epiphysis

Q992. First step:; Slipped Capital Femoral Epiphysis

A992. Emergency Surgery

Q993. Tx:; Kawasaki Disease

A993. IV immunoglobulins and Aspirin

Q994. Dx:; IgG, IgM and IgA are all very low, very low B-cells, bacterial infections in the first 5 years of life

A994. X-linked Agammaglobinemia

Q995. Dx:; low IgG, IgM and IgA levels, normal B-cells and T-cells; Sx begin at 15 - 35 years old.

A995. Common Variable Immunodeficiency; (CVID)

What are some causes of increased work of breathing in a pediatric patient who has suffered a traumatic event?

Airway, chest, and abdominal injuries

What drug may initially relieve respiratory distress in a child with bronchiolitis

Albuterol

What may be indicated for the management of severe respiratory distress associated with bronchiolitis

Albuterol, 0.15 mg/kg by inhalation

What is released in response to the hypovolemia associated w/ pyloric stenosis?

Aldosterone

Cyanotic heart defects w/ right-to-left shunts

All of them (Tetralogy of Fallot, TGV, Hypoplastic left heart syndrome, Truncus arteriosus, TAPVR)

What is an appropriate intervention for a child in whom epiglottitis is suspected

Allow the child to assume a comfortable position

0.2 mg/kg min:0.1 max:0.5

Atropine dose

PKU inheritance pattern

Autosomal recessive

Another name for Legg-Calvé-Perthes disease

Avascular necrosis of femoral head

Where do mitral lesions radiate to?

Axilla

Treatment of unconjugated hyperbilirubinemia

Phototherapy

Are more common in pediatric patients who have been subjected to major trauma

Pulmonary contusions

CXR findings associated w/ TAPVR w/ obstruction

Pulmonary edema

Most severe sequela of Truncus arteriosus

Pulmonary hypertension

Pulse in APGAR measures

Pulse rate

What are reliable indicators of cardiac output in pediatric patients?

Pulse rate and quality, skin color and condition, and capillary refill

- Arterial pulse waveform showing alternating strong and weak beats - Sign of *left ventricular systolic dysfunction*

Pulsus alternans

- Relatively normal QRS complexes, each followed by a smaller, abnormal one - Sign of *hypertrophic obstructive cardiomyopathy (HOCM)*

Pulsus bigeminus

Treatment of toxoplasmosis

Pyrimethamine and sulfadiazine

Treatment of moderate-severe Croup

Racemic epinephrine

Although occur less frequently in children they are not uncommon in abuse,

Rib fracture

CXR findings for coarctation

Rib notching

CXR findings associated w/ coarctation of the aorta

Rib notching + "3" sign

Disorder caused by lack of vitamin D, calcium, or phosphate leading to softening and weakening of the bones, making them more susceptible to fractures

Rickets

Presentation of Vitamin D deficiency

Rickets

What is the most likely diagnosis in: a 2-year-old girl who walks abnormally, falls a great deal, does not like milk, withdrew from breastfeeding & cow's milk early, physical exam revealing unsteady gait, bowing of the tibia, and x-rays revealing beading of the ribs & genu varum?

Rickets

What is the most likely finding on EKG in: a 3-month-old female infant that will not eat anymore, has a loud pansystolic murmur on PE, and appears small for her age?

Right ventricular hypertrophy due to VSD

RR > 60 HR > 180 or < 80 under 5 years old HR > 180 or < 60 over 5 years old Cyanosis Fever with petechiae Rispiratory distress Trauma Burns ALOC Seizures

Risks for cardiopulmonary arrest in a pediatric

Reflex where touching a baby's cheek, will result in the baby turning to that side

Rooting reflex

How do you stimulate crying and breathing?

Rubbing/stimulating heels

1-6 months

SIDS usually occurs during what months?

Presentation of Vitamin C deficiency

Scurvy (ecchymoses, bleeding gums, petechiae)

Pathophysiology of bleeding associated w/ Meckel's diverticulum

Searing of nearby small bowel tissue due to gastric acid secretion by ectopic tissue

Helps minimize injury when it happens

Secondary prevention

Most common type of ASD

Secundum defect

Treatment of prune belly

Serial Foley catheter placements

Less than or 8 GCS

Severe GCS

Features of the Catarrhal stage of Whooping cough

Severe congestion and rhinorrhea

Features of the Paroxysmal stage of Whooping cough

Severe coughing episodes with extreme gasp for air (inspiratory whoop) following by vomiting

Cyanosis, loud stridor, high anxiety.

Severe croup presents with....

Warm IV fluids. Rewarm until pt is at 95 degrees or a return of spontaneous circulation.

Severe hypothermia actions

86 degrees or less

Severe hypothermia is...

Severe hypothermia

Shallow breathing, and extreme disorientation or non responsiveness are environmental signs of...

Newborns lack the ability to ____ thus always prevent heat loss and cover the patients head.

Shiver

Why was the technetium-99m pertechnetate (99mTc) scan termed the Meckel scan?

Because of its accuracy

A child is experiencing signs and symptoms of hypoxia while on a home ventilator. On arrival, you should immediately perform what?

Begin ventilation with a bag valve device

For what drug is glucagon given as an antidote

Beta blockers

6-12

Between what months can an infants start to stand and walk?

Prototypic finding of duodenal atresia

Bilious vomiting on the 1st day of life, typically within 12 hours after birth

When should exchange transfusion be considered in the setting of hyperbilirubinemia?

Bilirubin ↑ >20-25 mg/dL

Components of immediate evaluation of sepsis

Blood and urine cultures

Sign observed on upper GI series in pyloric stenosis characterized by *filling defect in the antrum due to prolapse of muscle inward*

Shoulder sign

Surgical connection than runs from the brain to the abdomen to drain CSF and prevent ICP.

Shunt definition:

CSF Hearing loss seizure battle signs

Signs of a basilar skull fracture (4)

Retraction Nasal flaring Head bobbing Grunting Wheezing Gurgling Stridor

Signs of increased respiratory effort

Vitamin K deficient bleeding prophylaxis

Single IM dose of vitamin K

HR of 180

Sinus tach in children is...

HR greater than 220

Sinus tach in infants is..

What injuries should be considered the result of possible abuse

Bruises or burns in unusual patterns

Least common type of ASD

Sinus venosus defect

Menengitis

Bulging Fontanels and nuchal rigidity are signs of....

What is a sign of increasing intracranial pressure unique to an infant

Bulging fontanelle

Presentation of Vitamin B5 (panthotenic acid) deficiency

Burning feet syndrome

Motor vehicle collisions, falls, and drowning are common mechanisms of injury for children. What are some other common mechanisms of injury for children?

Burns, intoxication, and choking

Appearance in APGAR measures

Skin color/complexion

Children have thinner ____ and less ___ than adults, thus having having a larger BSA to weight ratio. Because of this they lose fluid and heat much quicker than adults.

Skin; Subcutaneous fat

Not always a sign of improvement i could mean the patient is tired and will soon not be able to compensate.

Slower breathing rate

Condition associated w/ an IDM that is characterized by a congenitally smaller descending colon leading to distention from constipation

Small left colon syndrome

Treatment of small left colon syndrome

Small, frequent feeds

CXR findings associated w/ TAPVR without obstruction

Snowman or figure 8 sign

Most accurate diagnostic test for PDA

Cardiac catheterization

Most accurate diagnostic test for coarctation of the aorta

Cardiac catheterization

Most definitive diagnostic test for ASD

Cardiac catheterization

hypotension

Cardiac tamponade may only present with...

CXR findings associated w/ ASD

Cardiomegaly + ↑ vascular markings

CXR findings associated w/ Truncus arteriosus

Cardiomegaly w/ ↑ pulmonary markings

A 4 year old child has fatigue, difficulty breathing, and peripheral edema. Crackles are audible in the bases of both lungs. Which illness do you suspect

Cardiomyopathy

0.5-1j/ kg

Cardiovert dose is...

Reflex where baby will automatically suck on a nipplelike object

Suckling reflex

First thing done for a newborn

Suction mouth and nose

A child who has a tracheostomy is dyspneic. The tube appears to be partly obstructed. What is your first intervention?

Suction the Tracheostomy

Reflex when held facing the floor, arms go out

Superman reflex

Infection CNS disorder Increased ICP

Causes of a fever (3)

Location of secundum type ASD

Center of atrial septum

Associate the *Murphy sign* w/ its associated condition

Cessation of breathing due to RUQ tenderness

Most likely cause of conjunctivitis at day 1

Chemical irritation

6 years

Child must be greater than * for an IO

Highest risk group for rickets

Children 6-24 months of age

Who receives Hepatitis B immunoglobulin + Hepatitis B vaccine?

Children w/ HBsAg+ mothers

Most likely cause of conjunctivitis after more than 7 days postdelivery

Chlamydia trachomatis

What is done when medical management of necrotizing enterocolitis fails?

Complete or partial colectomy

Treatment of Fifth disease/Erythema infectiosum

Supportive

Treatment of Roseola

Supportive

Treatment of rubella

Supportive

Treatment of Rubeola/Measles

Supportive treatment

Treatment of bronchitis

Supportive treatment

Treatment of Varicella

Supportive treatment w/ topical ointments

Management of imperforate anus

Surgery

Treatment of TAPVR w/ obstruction

Surgery

Treatment of TAPVR without obstruction

Surgery

Treatment of cyanotic heart defects

Surgery

Treatment of epispadias

Surgical correction

What actually causes the Doughnut/target sign?

Concentric alternating echogenic (mucosa) and hypoechogenic (submucosa) bands

What must you evaluate a patient w/ epispadias for?

Concomitant bladder exstrophy

Treatment of hypospadias

Surgical correction

What does the 1 minute APGAR score evaluate?

Conditions during labor and delivery

Condition in which there is an inherited defect of steroid synthesis

Congenital adrenal hyperplasia

Umbilical hernia is associated w/?

Congenital hypothyroidism

The lack of innervation to the distal bowel seen in Hirschsprung disease causes what in regards to bowel muscle tone?

Constant contracture

Most common surgical procedure and definitive treatment for duodenal atresia

Surgical duodenostomy

Definitive treatment for choanal atresia

Surgical intervention

Only definitive therapy for Tetralogy of Fallot

Surgical intervention

Treatment of omphalocele

Surgical reintroduction of contents

Management of Meckel's diverticulum

Surgical removal

Treatment of coarctation of the aorta

Surgical resection of the narrowd segment

80% of amniotic fluid is a filtrate of the mother's plasma. How does the baby produce the remaining 20%?

Swallowing, absorbing, filtering, and urinarting

Best initial test for Cystic fibrosis

Sweat chloride

Presenting symptoms of volvulus & intussusception

Symptoms of obstruction (distention, failure to pass flatus/stool)

A fever

Tachycardia and Tachypnea might be present with...

Often the first manifestation of respiratory distress in infants.

Tachypnea

What clinical finding may be present in a child who has ingested a large amount of aspirin

Tachypnea

Congenital heart defect associated w/ exercise intolerance and squatting

Tetralogy of Fallot

Most common cyanotic heart defect after the neonatal period

Tetralogy of Fallot

Most likely diagnosis w/ *boot-shaped* cardiac x-ray findings

Tetralogy of Fallot

Hole in the diaphragm that allows the abdominal contents to move into the thorax?

Diaphragmatic hernia

What is the most likely diagnosis in: a premature infant born at 28 weeks in respiratory distress, w/ grunting, nasal flaring, use of accessory muscles, bowel sounds heard upon auscultation of the back, and CXR showing air fluid levels in the chest?

Diaphragmatic hernia

10 stools a day.

Diarrhea is defined as...

Ultrasound findings associated w/ varicocele

Dilatation of the vessels of the pampiniform plexus > 2 mm

Conservative treatment for VSD

Diuretics and digoxin

What should not be done in patients w/ Diphtheria?

Do not scrape pseudomembranes

You are doing an interfacility transport of a pediatric patient whose brain has expanded into a linear crack in his skull widening the fracture. What is this condition known as?

Growing skull fracture

PEEP

Grunting increases what??

Pathophysiology of development of aspiration pneumonia in newborn w/ esophageal atresia w/ tracheoesophageal fistula

Food contents aspirated via fistula in the respiratory system

HR > 180 or < 60

HR for cardiopulmonary arrest over 5 years old

HR >180 or <80

HR for cardiopulmonary arrest under 5 years old

Mainstay of diagnosis for Hirschsprung disease

Full thickness biopsy

Indications for newborn nasogastric tube placement

GI decompression

Omphaloceles result from failure of what?

GI sac retraction at 10-12 weeks gestation

Rare genetic disorder that precludes normal metabolism of galactose

Galactosemia

Treatment of CMV

Ganciclovir w/ signs of end-organ damage

Pedi's are more prone to ____ _____.

Gastric distension

Inflammation of the GI tract secondary to microbiologic infiltrate and spread

Gastroenteritis

Most appropriate next step in management of: a 4-year-old child w/ extreme irritability, refusal to eat, refusal to lean back, muffled speech, extremely ill appearance, drooling, positive thumbprint sign on CXR?

Intubate

Most crucial next step in management of Epiglottitis

Intubate

What is the most likely diagnosis in: a 1-year-old w/ purple jelly stool, recent history of irritability, complaints of pain, episode of vomiting, lethargy, & firm-sausage-shaped mass on PE?

Intussusception

Associate the *doughnut sign* w/ its associated condition

Intussussception

Condition in which part of the bowel telescopes into another segment of bowel distal to it

Intussussception

Pulse associated w/ atrial fibrillation

Irregularly irregular pulse

Yes. It effects children under 2 years.

Is bronchiolitis contagious? What age group does it usually effect?

Wall defect lateral to midline w/ intestines and organs forming beyond the abdominal wall w/ no sac covering

Gastroschisis

Most accurate test for Cystic fibrosis

Genetic analysis of the CFTR gene

Cause of Tetralogy of Fallot

Genetic and environmental factors

Under the car. Head and spine are commonly injured

Where do peds usually go when hit by a car?

C-1 and C-2

Where do spine fractures usually happen in peds?

Under 3- under shoulders Over 3- under head

Where do you put a towel for under and over the age of three years?

Form of bronchitis caused by Bordetella pertussis

Whooping cough

Small purple spots resulting from skin hemorrhages

petechiae

Pathophysiology behind notched appearance of ribs in coarctation of the aorta

↑ pressure in subcostal vessels erodes ribs

Regardless of surgical intervention, cryptorchidism is associated w/ what?

↑ risk of malignancy

Aldosterone's effect on postassium

↑ urinary excretion of K+

Mechanism behind ↓ Na+, Cl-, glucose, & ↑ K+ seen w/ CAH

↓ aldosterone & cortisol production

Effect of asymmetric septal hypertrophy in an IDM

↓ cardiac output

Why are children more vulnerable to liver and splenic injuries

The abdominal musculature is minimal and does not protect these organs

Why is surgery done on both hips in patients w/ Legg-Calvé-Perthes disease?

The contralateral unaffected hip eventually undergoes necrosis

Congenital condition in which there is no venous return b/w pulmonary veins and the left atrium

Total anomalous pulmonary venous return (TAPVR)

What is a bacterial infection of the upper airway & subglottic trachea that occurs during or after Croup

Tracheitis

Something to be considered if you see recurrent aspiration pneumonia

Tracheoesophageal fistula

Common respiratory findings in newborns delivered via cesarean birth

Transient tachypnea of the newborn (excess fluid in the lungs, hypoxia)

Condition characterized by an aorta that orginates from the right ventricle and pulmonary artery that comes from the left ventricle

Transposition of the great vessels

Most common cyanotic lesion during the neonatal period

Transposition of the great vessels

Pressers and Forosemide

Treatment for cardiomyopathy

Ventilation. If not increase of HR, intubate. Consider Epi down the ET tube.

Treatment for unstable bradycardia

A benefit of immediately drying a newborn is that the act of drying also provides gentle stimulation, which may initiate respirations

True

In addition to being the organ of exchange between the mother and fetus, the placenta becomes becomes a temporary endocrine gland, secreting estrogen and progesterone

True

In infants and young children the chief support for the chest wall comes from muscles rather than bones

True

Neonates are obligate nose breathers during the first month of life

True

When using a bulb syringe to suction the nose and mouth of a newborn, it is preferable to suction the mouth first to prevent aspiration in case the infant gasps and begins to breathe when the nose is cleared

True

Congenital heart defect where a single trunk emerges from both right and left ventricles and gives rise to all major circulations

Truncus arteriosus

A hydrocele is a remnant of what?

Tunica vaginalis

Presenting features that are clinically diagnostic for choanal atresia

Turn blue when feeding, pink when crying

Frequent association w/ coarctation of the aorta

Turner syndrome

1st-Chemical mediated release of histamine. 2nd- Inflammation of bronchioles

Two phases of Asthma

Best initial diagnostic test for Herpes

Tzanck smear

Best initial diagnostic test for Varicella

Tzanck smear showing multinucleated giant cells

Treatment of prune belly carries a high risk of what?

UTIs

Most accurate test for varicocele

Ultrasound of the scrotal sac

W/ varicocele, what must you always do on ultrasound?

Ultrasound the other testicle

Congenital weakness of the rectus abdominis muscle which allows for protrusion of vessels and bowel

Umbilical hernia

You are caring for a teenager who was "huffing" toluene. What effects could this produce

Visual disturbances

What is the most likely diagnosis in: a 1-week-old newborn who received no routine newborn care presenting w/ bright red blood per rectum, lethargy, unequal pupils, and gross red blood in his diaper?

Vitamin K deficient bleeding

What does a succussion splash sound similar to?

Waves on a beach

Liver and spleen. Organs are closer together

What 2 organs may most be effected in blunt trauma?

Cholinergenic. (they make you into liquid) Atropine is an antidote.

What are organophosphates and what do you use as an antidote?

The growth plate

What could an IO disrupt?

use it to breath

What do infants do with their diaphragm?

Ethel alchohol ETOH

What is an antidote for antifreeze?

Scolding

What is the most common burn for peds?

Cricoid

What is the narrowest part of the pediatric airway?

Tight or bulged and pulsating might stop

What might fontanelles look like with ICP??

Epiglottis

What presents with drooling?

Sunken

What will fontanelles look like in dehydration?

Toddler 1-3 years

When can you start asking the patients some questions other than just the caretaker?

Anterior 9-18 months Posterior 4 months

When do anterior and posterior fontanelles close?

Preschool 3-5 years

When may a child want to help you with treatment?

Q813. What are (4) developmental milestones at 9 months old?

"A813. ""Pinches Ma-ma's furniture Bye-Bye to cruise""; 1. Pincher grasp; 2. says: Mama or Dada; 3. waves Bye-Bye; 4. Cruises around furniture"

Hallmark findings on imaging for duodenal atresia, volvulus, & intussussception

- *Duodenal atresia*: double bubble sign - *Volvulus*: "bird beak" apperance - *Intussussception*: doughnut/target sign

CAH's general effect on sexual development in boys

- No initial abnormalities - Loss of defining sexual features w/ ↑ age

Treatment of Scarlet fever

- Penicillin - Azithromycin - Cephalosporins

WAGR syndrome

- Wilms tumor - Aniridia - GU malformations - Retardation

Q123. nursemaids elbow

A123. subluxation of radial head

Q169. tx of Meckel's diverticulum

A169. surgical resection

Q314. onset of chlamydia conjunctivitis

A314. DOL 5-14

Q348. Child Abuse - Tx

A348. Document injuries; notify child protective services; hospitalize if necessary

Q380. Ventric Septal Defect (VSD) - Dx

A380. Echocardiogram - diagnostic; EKG - may show RVH or LVH; normal with small VSDs

Q405. Febrile Seizures - Dx

A405. Find source of infection; LP - if signs of CNS infection; No labs if presentation consistent with febrile seizure; Atypical presentation - electrolytes, glucose, BC, UA, CBC with diff, EEG &amp; MRI - complex seizures

Q466. Meckel's Diverticulum - Tx

A466. Surgery

Q498. Terminal Complement Deficiency; (C5-C9) - Tx

A498. Meningococcal vaccine; appropriate Antibiotics

Q549. What congenital defect often presents in childhood with asymptomatic hypertension?

A549. Coarctation of the aorta

Q550. What is the most common childhood malignancy?

A550. ALL

Q554. hyponasal speech plus mouth breathing, think?

A554. NASAL POLYP

Q659. What is the first step in a suspected meningitis of any age?

A659. IV Ceftriaxone

Q692. Dx:; High fever, sore throat and ulcers scattered on the soft palate, tonsils and pharynx; cause?; Tx?

A692. Herpangina; cause: Coxsackievirus; Tx: None...will go away

Suicidal Gestures usually occurs in

Adolescents

20mL/kg

Dose for fluids

Presentation of Herpes in the 3rd week

Encephalitis

When did it start? What is the current temp? Did you try any remedies?

Fever questions to ask

EKG findings associated w/ PDA

LVH 2° to high SVR

Most common extracranial solid malignancy

Neuroblastoma

Contrast enema findings for Hirschsprung disease

Retention of barium > 24 hours

A 10 month old child "didn't wake up from his nap." He is unconscious and has vomited. What other physical finding may indicate abuse

Retinal Hemorrhage

W/ a Morgagni diaphragmatic hernia, where is the defect?

Retro- or parasternal

Signs and symptoms of TAPVR without obstruction

Right heart failure & tachypnea in a 1-2 year-old

For which of the 3 major causative agents of viral infectious diarrhea is there a vaccine available?

Rotavirus

Most common cause of viral infectious diarrhea in the winter

Rotavirus

Which of the 3 major causative agents of viral infectious diarrhea present w/ a viral prodrome?

Rotavirus & Adenovirus

Effect of hypoglycemia in an IDM

Seizures

If tachypnea lasts > 4 hours, what is it considered?

Sepsis

How is perforation excluded in the setting of necrotizing enterocolitis?

Serial AXRs

Presentation of Herpes in the 1st week

Shock and DIC

CXR findings w/ Epiglottitis

Thumbprint sign

Reasoning for sending stool in for blood and leukocyte count in the setting of gastroenteritis

To detect presence of invasive toxins

During your assessment, when should you begin using the Pediatric Assessment?

Upon visual contact

Most accurate test for pyloric stenosis

Upper GI series

Defect in the wall dividing the left and right ventricles of the heart

VSD

D25W -50% dextrose solution with sterile water or saline

treatment for hypoglycemia

Levels of 11-DOC characteristic of *11β-hydroxylase*?

Why does oligohydramnios cause flat facies?

↑ atmospheric pressure → compression of fetus normally buffered by amniotic fluid

Q100. dx of developmental dysplasia of hip

"A100. u/s: see ""false acetabulum"" in lateral ileum"

Q240. clinical presentation of meconium ileus

"A240. vomiting; abdominal distension; distended bowel loops; ""soap bubble"" on XR"

Q325. Neuroblastoma - History/PE

"A325. Can occur anywhere; Sx vary with location; nontender abdominal mass; (may cross midline); Horner's syndrome; HTN; cord compression; anemia; FAILURE TO THRIVE; fever; site-specific metastasis can cause proptosis; periorbital bruising; subq tumor nodules; bone pain with pancytopenia; opsoclonus/myoclonus; ""dancing eyes, dancing feet"""

Q350. Epiglottitis - History/PE

"A350. Sudden-onset high fever; dysphagia; drooling; muffled voice; soft stridor; cyanosis; ""sniffing dog"" position; ""tripod"" position; insist on sitting up in bed; untreated - life-threatening"

Q777. What Non-Hodgekin's Lymphoma presents as an abdominal mass and is associated with EBV?; what is seen on LN bx?; Chromosomal anomaly and gene?

"A777. Burkitt Lymphoma; LN Bx: ""Starry sky"" pattern; Chromosomes: t(8;14); Gene: c-myc"

Q786. Dx:; A child of 2 - 7 months; presents with Recurrent mixed Flexor-Extensor spasms lasting only a few seconds, but repeat more then 100 times in a row; Looks like multiple Startle or Moro response; Dx Test &amp; Result?; Tx? (2 together)

"A786. Infantile spasms (""West syndrome""); Dx Test: EEG: HYPSARRTHYMIA; Tx:; 1. ACTH; 2. prednisone"

Q791. Dx:; Infant with spinal muscular atrophy, Fasciculations, frog-leg posture; Dx Test/results? (3); Tx?

"A791. Werdnig-Hoffman Disease (""Floppy-Baby"" syndrome); Dx Test:; 1. EMG: Fibrillations; 2. Muscle bx: denervation; 3. Nerve bx: slowed conduction; Tx: Supportive"

PE findings for varicocele

"Bag of worms" sensation

Major cardiac abnormality associated w/ an IDM

*Asymmetric septal hypertrophy* due to obliteration of the LV lumen

Presentation of cerebellar injury

*Ataxia* rather than simply an unsteady gait

Specific characteristics in the setting of bilious vomiting on the 1st day of life that point to a diagnosis of duodenal atresia

*Down syndrome-like characteristics* (oblique eye fissures, simian crease, congenital murmur)

Tetralogy of Fallot is characterized by

*Mnemonic*: PROVe - Pulmonary stenosis - Right ventricular hypertrophy - Overriding aorta - Ventricular Septal Defect (VSD)

Best treatment of duodenal atresia, volvulus, & intussussception

- *Duodenal atresia & volvulus*: surgery - *Intussussception*: air enema

Typical onset of duodenal atresia, volvulus, & intussussception

- *Duodenal atresia*: within 1st day - *Volvulus & intussussception*: within 1st year

Presentation of Tetralogy of Fallot

- *Holosystolic murmur* best heart at LLSB - Cyanosis of lips and extremities - Squatting after exertive actrivities

Treatment of Whooping cough

- *Isolation + erythromycin or azithromycin* (ineffective in Paroxysmal stage) - *Macrolides* for close contacts

APGAR scoring for Respiration

- 0: Absent - 1: Weak or irregular - 2: Strong

Signs of pathologic jaundice in the newborn

- Appears on the 1st day of life - Bilirubin ↑ > 5 mg/dL/day - Bilirubin ↑ > 19.5 mg/dL in a term child - Direct bilirubin ↑ > 2 mg/dL at any time - Hyperbilirubinemia persists after the 2nd week of life

What are the most common *wrong* answers for diagnostic testing of imperforate anus?

- Barium study - Rectal manometry

Presentation of Croup

- Barking cough - Coryza - Inspiratory stridor

Presentation of Vitamin B1 (thiamine) deficiency

- Beriberi - Wernicke's encephalopathy

Complications of volvulus

- Bowel necrosis w/ perforation - Life-threatening sepsis

Cyanotic heart defects in which a VSD is present

- Tetralogy of Fallot - Truncus arteriosus

Complication of overly aggressive surgical reintroduction of the bowel in gastroschisis

- Third spacing - Bowel infarction

With regards to the hearing test, why is early detection necessary?

- To maintain speech patterns - Assess the need for cochlear implantation

Treatment of Wilms tumor

- Total nephrectomy for unilateral - Partial nephrectomy for bilateral - Chemotherapy + radiation (based on staging)

Presentation of rickets

- Ulnar/radial bowing - Waddling gait due to tibial/femoral bowing (characteristic sign)

Antibiotics of choice in treatment of necrotizing enterocolitis

- Vancomycin - Gentamicin - Metronidazole

Presentation of non-inflammatory diarrhea

- Vomiting - Crampy abdominal pain - Watery diarrhea

Sodium Bicarbonate

1 mEq/kg per dose

Lidocaine

1 mg/kg

After the delivery, the paramedic should assign an apgar score at ______ minute(s) and ____ minute(s) to evaluate in the infant

1, 5

Normal urine output

1-2 ml/kg/hr

Fetal heart sounds can be auscultated beginning at _____ weeks of gestation

12

Normal newborn HR

120-160 BPM

How many different varietes of long QT syndrome are there?

13

Education, Environmental modification, Enforcement, Engineering.

4 E's of prevention

The current belief is that a blood glucose measurement less than ____ mg/dl in the infant indicates hypoglycemia

40

The period during which the fetus grows and develops within the uterus (gestation) averages ____ weeks from the time of fertilization to delivery of the newborn

40

A 5 year old, 44 lb child is in ventricular fibrillation. What is the correct initial energy level for defibrillation

40 joules

Normal newborn respiratory rate

40-60 BrPM

Q052. clinical features of HENOCH-SCHONLEIN PURPURA

A052. condition preceded by uri; non-thrombocytopenic palpable purpura; hematuria (good prognosis); proteinuria (poor prognosis); spasmodic abdominal pain; ileus, n/v; ugi/lgi bleed

Q053. tx of Henoch-Schonlein Purpura

A053. steroids

Q084. findings in classic hemophilia

A084. bleeding problems + hemarthrosis

Q134. Tx of CF

A134. chest physiotherapy; exercise; frequent cough; recombinant human dnase given through nebs to break down thick mucous complexes

Q136. mesenteric lymphadenitis leads to

A136. persistent pain after infection

Q196. when is it normal for there not to be tears

A196. in the first few weeks of life

Q197. age of onset of gonococcal ophthalmia neonatorum

A197. 2-4 days

Q198. age of onset of chlamydia ophthalmia neonatorum

A198. 4-10 days

Q199. clinical features of gonococcal ophthalmia neonatorum

A199. eyelid edema; chemosis; purulent d/c

Q200. complications of gonococcal ophthalmia neonatorum

A200. sepsis; meningitis; arthritis; corneal ulceration; blindness

Q234. clinical presentation of acute arsenic poisoning

A234. n/v/d, abdominal pain; 3rd spacing; hemorrhage; hypovolemic shock; Ventricular tachycardia; QT prolongation

Q235. narcosis

A235. deep stupor, unconsciousness

Q236. clinical presentation of cerebral palsy

A236. postural hypotonia; failure ot reach for toys; gross and fine motor delay; cognitive and sensory deficits

Q274. Patau's or Edwards syndrome; narrow forehead

A274. Edwards

Q275. Patau's or Edwards syndrome; microphthalmia

A275. Patau

Q276. Patau's or Edwards syndrome; cutis aplasia

A276. Patau

Q317. effect of propanolol in utero

A317. IUGR; decreased ability of asphyxiated newborn to increase heart rate and CO; associated with hypoglycemia and apnea

Q318. when is surgery for cleft lip done

A318. 3 mo

Q355. Croup (Laryngotracheobronchitis) - Dx

A355. Clinical; XR - steeple sign

Q389. Coarctation of the Aorta - Tx

A389. Surgery or balloon angioplasty; endocarditis prophylaxis

Q404. Febrile Seizures - History/PE

A404. Most are simple seizures; simple - high fever, fever onset within hrs of seizure; generalized seizure, lasts &lt; 15 min., 1 in a 24-hr period; complex - low-grade fever, fever for several days before seizure onset, seizure has focal features, can have postictal paresis, lasts &gt; 15 min., &gt; 1 in a 24-hr period

Q406. Febrile Seizures - Tx

A406. Simple - aggressive antipyretics; tx underlying illness; complex - thorough neuro exam, chronic anticonvulsants may be necessary

Q407. Febrile Seizures - Complications

A407. Febrile Seizure will recur in 30%; no increased risk of epilepsy, developmental, intellectual or growth abnormality; those with complex seizures - 10% risk of developing epilepsy

Q417. Double Y males - What is it

A417. XYY; phenotypically normal; very tall; severe acne; antisocial behavior

Q605. presentation of transposition?

A605. blue baby with low sat, no change with 02 given, otherwise looks in no distress; egg on string (narrow mediastinum) chest x-ray...

"Q606. a typical, physiological ""flow"" murmur?"

A606. Still's murmur - heard at left lower sternal border

Q607. systolic murmur heard in ASD is really what?

A607. increased flow across pulmonic valve and tricuspid regurgitation

Q608. systole ejection murmur heard best at upper sternal border but radiating to axilla and back? vs carotids?

A608. axilla and back = peripheral pulmonic stenosis (due to flow past sharp angle of pa and its branches = resolves in 6 mo; carotids = aortic stenosis/bicuspid aortic valve

Q609. Most common congenital heart lesions?

A609. VSD

Q610. what's a positive PPD?

A610. 5mm if CLOSE CONTACTS w/known or suspected cases; 10mm if greater risk for disseminated disease ie &lt;4y/o's, DM, CKD, malnourishment, lymphoma, or at greater risk of exposure to TB, ie born in, parents born in high prevalent regions, travel to these regions, exposure to adults at high risk, (ie HIV infected, homeless, drug abusers); 15mm in normal, age over 4 w/o any risk factors

Q611. What are the 5 portions of the APGAR score?

A611. Appearance; Pulse (HR); Grimace; Activity; Respirations

Q612. Dx:; A lump on the face after a foreceps delivery

A612. Subcutaneous Fat Necrosis

Q613. Dx:; rash that starts after the first day of life as a flat reddish area with a central papule that becomes a pustule and persists for a couple of weeks; Test?

A613. Erythema Toxicum; test: Wright stain of vesicular fluid will show Eosinophils

Q614. Dx;; Strawberry rash of newborn with difficulty breathing

A614. Epiglottic Hemangioma

Q615. Dx:; sharply demarcated bruise-like appearance on sacral area of newborn

A615. Mongolian spots; (do not confuse w/ Abuse, which is not sharply demarcated)

Q616. Dx:; orange/yellow warty lesion on the scalp in newborn that displaces hair growth; Tx?; what can it lead to?

A616. Nevus sebaceous; Tx: leave alone until Adolescence (should go away); leads to: 15% chance to become malignant

Q617. Dx:; scalp swelling that crosses suture lines and goes away in a few days

A617. Caput

Q618. Dx:; scalp swelling that does not cross suture lines and goes away in a couple of months; cause?

A618. Cephalohematoma; cause: Subperiosteal bleed

Q619. When do anterior fontanels normally open and close?; Posterior?

A619. Anterior: opens: birth; closes: 9 - 18 months; Posterior: opens: sometimes at birth; closes: 4 - 5 months

Q620. what is possible diagnosis if:; 1. both fontanels are enlarged?; 2. One fontanel only is enlarged?

A620. 1. Hydrocephalus; 2. Hypothyroidism

Q621. Newborn eye:; White reflex

A621. Retinoblastoma; (another source: MCC is Congenital Cataracts)

Q633. birthing injury where baby has hand extended and fingers flexed; what nerve roots?; another name for this injury?

A633. Klumpke; roots: C8-T1; another name: "claw hand"

Q634. What are the (3) 'diseases' tested for during a newborn screening?

A634. 1. PKU; 2. Galactosemia; 3. Hypothyroidism

Q635. Dx:; baby born with Mental retardation, Eczema, musty odor, fair hair, fair skin and blue eyes

A635. PKU

Q636. Dx:; baby born with Jaundice, Hypoglycemia, and Cataracts; genetic inheritance?; what is contraindicated in this baby?

A636. Galactosemia; (Autosomal Recessive); Contraindication: Breastfeeding

Q637. Dx:; Newborn with Jaundice, mottled, constipation, Large tongue, Umbilical hernia; what is seen on blood test?

A637. Hypothyroidism; blood test: high TSH, low T4

Q638. Dx:; pre-term baby with ground glass appearance on CXR; Tx? (2 in order)

A638. Respiratory Distress Syndrome Tx:; 1. Surfactant; 2. Ventilation

Q639. Dx:; baby delivered by C-section has a Rapid respiratory descent but after 72 hours of Tachypnea, then gets better; CXR shows possible fluid in the fissure

A639. Transient Tachypnea of newborn; (when delivered by C-section that do not get a vaginal squeeze and then don't expel amniotic fluid); patient does not need respiratory support; (a main way to differentiate)

Q640. What are the 3 differences b/t physiologic and pathologic jaundice?

A640. Physiologic:; 1. Appears &gt;24 hours of age; 2. peaks at or below 12.9 by 3 days; 3. resolves by 1 week Pathologic:; 1. Within first day of life; 2. Level &gt;13; 3. Lasts &gt; 1 week

Q641. MC type of pathologic Jaundice?; Tx?

A641. Hemolysis (Rh or ABO incompatibility); Tx: Phototherapy

Q642. Dx:; newborn with Jaundice within 8 hours of birth, a Direct bilirubin increase; Acholic (or clay-colored) stools

A642. Biliary Atresia

Q643. Aside from certain meds, what are the (6)* main contraindications of offering newborn breast milk?

A643. "A Healthy Toddler Gladly Sucks Vitamins":; AIDS;; Herpes if breast lesions;; TB (active, untreated);; Galactosemia;; Syphilis;; Varicella

Q644. How many calories are in baby formula?; When can Whole milk be started?; When should Iron supplementation begin?

A644. Formula: 20 cal/oz; whole milk: &gt; 1 year-old; Iron: &gt; 6 months old

Q645. What test can be done in newborn to test for Turner's syndrome?

A645. Buccal smear; (to test for Barr Bodies)

Q646. definition:; child is short but expect a growth spurt at a later age

A646. Constitutional Delay

Q647. MCC for childhood obesity?

A647. take in too many calories and don't burn enough off; (don't be tricked and look for a pathologic cause)

Q648. when do most reflexes disappear?; which (2) don't at this time?; what time do they disappear?

A648. reflexes disappear: 4 - 6 months; except: Babinski: until 18 months; Parachute: never

Q649. Rule for Cube stacking in child; (give 3 milestones)

A649. 3 cubes at 15 months; 4 cubes at 18 months; 7 cubes at 24 months

Q650. MCC for Enuresis; First Tx?

A650. Primary Nocturnal (usually a male child that is just a deep sleeper); Tx:; 1. Give it Time; 2. Alarms; 3. Meds (Imipramine; DDAVP)

Q651. In the severely immunocompromised, what vaccination do you not give?

A651. MMR

Q652. what 3 vaccinations are contraindicated with an allergy to neomycin?

A652. 1. IPV; 2. MMR; 3. Varicella

Q653. which vaccination do you give:; child &gt; 2 yo who has not been vaccinated if they have immunodeficiency of any kind, are asplenic or lack splenic function (sickle cell)

A653. Pneumococcus spp.

Q654. which vaccination is contraindicated in patient who has streptomycin allergy?

A654. IPV; (also CI if they have neomycin allergy)

Q655. what 2 additional vaccinations are required specifically if patient is asplenic?

A655. 1. Meningococcal; 2. Pneumococcus spp.

Q656. MCC of Meningitis in:; Neonate

A656. Group B Strep; (from mother)

Q657. MCC of Meningitis in:; Patient &gt; 2 months old

A657. Pneumococcus

Q658. MCC of Meningitis in:; College or Military; (sexually active)

A658. N. Meningococcus

Q660. MCC of Meningitis in:; Elderly, HIV+, steroid user; (immunocompromised)

A660. Listeria

Q667. Bug causing septic arthritis from a dog or cat bite?

A667. Pasturella

Q668. Pediatric patient without immunization history has coughing spasms, inspiratory whoop, facial petechiae and leucocytosis with lymphocytosis. (2) possible confirmatory tests; Tx?

A668. Pertussis; tests:; 1. Positive cultured Nasopharyngeal washings; 2. Rapid Fluorescent antibody stain; Tx: Erythromycin

Q669. What can Fifth's disease lead to?; what can it cause in unborn child if mother has it?

A669. leads to: Aplastic Anemia; in unborn child: Hydrops

Q670. Dx:; three-day rash and lymphadenopathy

A670. Rubella

Q671. what 2 test should be run if suspecting acetaminophen toxicity?

A671. 1. Acetaminophen level in blood; 2. LFTs

Q672. A child swallowed a bottle with no lable and has Sx of:; Hypernea/Tachypnea, Fever, N/V, Dehydration, Seizures, agitation, tinnitus; Fastest diagnostic test?

A672. Aspirin toxicity; fastest test: Urine Ferric Chloride Test

Q673. what acid/base problems does Aspirin toxicity cause in a child?

A673. Respiratory Alkalosis w/ Metabolic Acidosis; (high pH, low pCO2 and bicarb); Tx: Charcoal

Q674. A child swallowed a bottle with no lable and has Sx of:; Tachycardia, HTN to Hypotension, confusion, drowsiness, dilated pupils, Seizures, Widened QRS; Ventricular arrhythmia; Tx? (2)

A674. Tricyclic Antidepressant toxicity; Antidote/Tx:; (1) Gastric Lavage/Charcoal; (2) Sodium Bicarbonate (to alkalinize blood for arrhythmias)

Q675. What is the first test if you suspect a TCA toxicity?

A675. ECG; (will show Widened QRS, Ventricular arrhythmia)

Q676. A child presents with swallowing kerosene. What is the primary objective?; What are the first (2) tests?; What are (2) Tx?

A676. objective: Prevent Aspiration resulting in Chemical Pneumonitis; tests:; 1. CXR; 2. Blood Gas; Tx:; 1. Prevent aspiration; 2. avoid gastric lavage

Q677. a child swallows something from an unlabled bottle:; diarrhea, excessive urination, miosis, bradycardia; Test?; Tx?

A677. Organophosphate (insecticide) toxicity; test: Plasma Cholinesterase activity; Tx: Atropine w/ Pralidoxime chloride

Q678. a child swallows an unlabled bottle of pills:; drowsey, delirius, hallucinate, N/V, DRY state, dilated pupils; Test?; Tx?

A678. Antihistamines; Test: Drug screen; Tx: Physostigmine (if severe)

Q679. a child swallows an unlabled bottle of pills:; signs of hemorrhagic gastroenteritis (bloody diarrhea), N/V, diarrhea, acute liver failure, shock, coma; Test?; Tx?

A679. Iron pill overdose; test: Serum Iron level; Tx: Deferoxamine

Q680. MCC of conjunctivitis in newborn (less then 24 hours-old)

A680. Chemical; (from Silver Nitrate drops)

Q681. MCC of infectious conjunctivitis in newborn

A681. Chlamydia

Q682. Medical term:; Conjunctivitis within the first month of life

A682. Ophthalmia Neonatorum

Q683. Difference in presentation b/t viral vs. bacterial conjunctivitis

A683. Viral: Crusting of eyelid; Clear discharge; Bacterial: Mucopurulent discharge

Q684. At what age is strabismus still normal?; What can it lead to if it is prolonged?

A684. Normal: up to 4 months old; leads to: Amblydopia

"Q685. Dx:; 2 yo child has a ""lazy eye""; Dx tests? (2); Tx?"

A685. Amblydopia; tests:; 1. Hirschberg test (shine light directly into eye and look to see if it falls at an angle); 2. Cover test (cover good eye and lazy eye will move straight ahead); Tx: Patch good eye (causes patient to use lazy eye; must be done early in life)

Q686. Child presents with a painful buldging eye; eyeball does not move when testing muscles; Dx test?; Tx? (2 possible)

A686. Orbital Cellulitis; Dx test: CT Scan; Tx:; 1. Cefuroxime with Clindamycin, or; 2. Ampicillin/Sulbactam

Q687. Child presents with swelling of eyelids and surrounding tissue; eye moves when checking ROM

A687. Preorbital cellulitis

Q688. Tx for a large Retinoblastic tumor of the eye

A688. Enucleation

Q689. Definition:; Otitis Media w/ Osteomyelitis of the bony canal or mastoid; Bug?; what Disease is it associated with?

A689. Malignant Otitis Media; bug: Pseudomonas; Assoc Disease: Diabetes

Q709. Dx:; CHF early in life including a feeding difficulty, sweat while feeding and tachypnea; loud harsh pansystolic murmur heard in the Lower left sternal boarder; What is it Associated with?

A709. Endocardial cushion defect; (Subclass of VSD and ASD; also known as "Common AV canal"); Assoc w: Down's Syndrome

Q710. Dx:; Weak/delayed/absent lower extremity pulses, HTN; What is seen on CXR?; What Disease is it assoc with?

A710. Coarctation of the Aorta; CXR: Rib notching; Disease: Turner's Syndrome

Q712. Dx:; Cyanosis; Single second heart sound; Murmur Heard in Upper left sternal border; What is seen on CXR?

A712. Tetralogy of Fallot; CXR: Boot-shaped heart

Q714. Dx:; CHF w/i weeks of birth; Systolic Ejection murmur in the Left sternal border; Single S2; Wide Pulse Pressure; bounding arterial pulses; What Disease is it assoc with?

A714. Truncus Arteriosus (Single arterial vessel arising from the base of the heart that goes to the coronary, systemic AND pulmonary circulation); Assoc with: DiGeorge Syndrome (22q11 microdeletion)

Q715. Dx:; single S2 pansystolic murmur with a CXR showing a Heart that looks like Snowman or Figure 8

A715. Total Anomalous Pulmonary Venous Return; (TAPVR)

Q716. Dx:; Loud, palpable S2 without murmur; Common cause of heart failure in first week of life. What can Patient develop?

A716. Hypoplastic Left Heart; (Underdevelopment of left cardiac chambers, atresia or stenosis of aortic and/or mitral orifices, and hypoplasia of the aorta); Patient develops Metabolic Acidosis

Q717. What are the (5) parts of the Major criteria for Rheumatic Fever?; What is Ailment is it assoc with?

A717. Major (JONES) Criteria: (2 of these = Rheumatic fever); J = Joints; O = carditis (O looks like a heart); N = Nodules (subcutaneous); E = Erythema marginatum; S = Sydenham's chorea; Association with: Strep throat (GAS)

Q718. MCC of HTN in infants or children?

A718. Renal disease

Q719. MCC of Diarrhea in children

A719. Rotavirus

Q729. MCC of GI blood from neonate?; Dx Test?

A729. Anal Fissure; Dx test: Apt test (to confirm Dx: tells if fetal Hb versus mother Hb)

Q730. If a neonate has a positive Apt test, what is the cause of the GI blood?

A730. Swallowing maternal blood

Q731. Dx:; Premature infant with Low Apgar and when fed the baby has bloody stools and abdominal distention; What is seen on Abdominal x-ray?

A731. Necrotizing Enterocolitis; Abd x-ray: Pneumatosis intestinalis (Air in bowel wall)

Q733. Dx:; Painless rectal bleed in 2 yo that can cause intussusception; Dx test?

A733. Meckel's diverticulum; (disease of 2's: 2% of population, 2:1 males, 2 years of age, 2 types of tissue, 2 feet from ileocecal valve, 2 cm in size); Dx Test: Technetium scan (Lights-up gastric tissue)

Q734. Dx:; Hematuria, Edema, Hypertension, Azotemia and Oligouria; What infection does it follow?; Dx Test? (2 plus results)

A734. Acute Glomerulonephritis; follows: Group A Strep infection; Dx Test:; 1. Urine: Red cell Casts; 2. Blood: C3 decreased

Q735. Dx:; Renal failure in child with hearing loss and possible cataracts; Genetics?

A735. Alport's Syndrome; (Mutation coding type IV collagen; Problem in basement membrane); genetics: X-linked Dominant

Q736. Dx:; Periorbital Edema / Ascites, High Proteinuria, Hypoalbuminemia, Hyperlipidemia

A736. Nephrotic Syndrome; (MCC in children: Minimal Change Disease)

Q744. Dx:; child who has painful bones that wake him from sleep but are relieved by aspirin

A744. Osteoid Osteoma

Q751. Patient develops anaphylaxis after IgG exposure. What is the first possibility?

A751. IgA Deficiency; (MC primary immunodeficiency)

Q752. Dx:; recurrent respiratory, GU and GI infections; IgG subclass 2 and IgA are low; eveything else is normal

A752. IgA Deficiency

Q753. What is the deficiency in most cases of Severe Combined Immunodeficiency (SCID)?; The incidence of what cancer is greater in this patient?

A753. Adenosine Deaminase deficiency; cancer: Lymphoma

Q754. Dx:; a patient has a specific component of the complement system that is low and continues to get Neisseria infections.

A754. Complement deficiency in any complement from C5 - C9

Q755. What type of juvenile RA occurs in a few large joints (like knee, hip shoulder)?

A755. Pauciarticular JRA

Q756. What is positive and what sex is predominant in JRA pauciarticular arthritis type 1 versus type 2?

A756. Type 1: ANA+, Females; Type 2: HLA B27+, Males

Q760. Another name for Kawasaki disease?

A760. Mucocutaneous LN syndrome

Q771. What (2) coagulopathies present with:; Normal PT,; High PTT,; Normal Bleeding time; What medication also presents this way?; What else in blood work differentiates it?

A771. Coags:; Hemophilia A and Hemophilia B (X-linked recessive); Med: Heparin; Difference: Thrombocytopenia

Q772. What coagulopathy presents with:; High PT,; High PTT,; High Bleeding Time

A772. DIC; (will also see low platelet count)

Q773. What coagulopathy presents with:; Normal PT,; High PTT,; High Bleeding time

A773. vonWillebrand's Disease; (Autosomal dominant)

Q774. What coagulopathy presents with:; High PT,; Normal PTT,; Normal Bleeding Time

A774. Warfarin toxicity

Q775. Dx:; a boy presents with a high PTT and Hemarthrosis; Tx?

A775. Hemophilia A; Tx: replace Factor VIII

Q776. Dx:; A 3 - 5 yo child w/ an acute onset of Anorexia, pallor, fever, bone pain in 1/4 of patients, Pancytopenia. Dx Test?; Tx? (3); MC places for Relapse? (2)

A776. Acute Lymphocytic Leukemia (MC childhood cancer); Dx test: Bone Marrow biopsy; Tx:; 1. Chemotherapy,; 2. Radiation,; 3. Transplant; Relapse:; 1. TESTES; 2. CNS

Q778. Dx:; Persistant cervical lymphadenopathy, night sweats, otherwise normal adolescent; Dx test?

A778. Hodgkin Lymphoma; Dx test: LN biopsy (Reed-Sternberg cells)

Q779. How is a Neuroblastoma and Wilms Tumor distinguished by CT scan?

A779. Neuroblastoma:; 1. on Adrenal gland; 2. Calcifications seen; Wilm's Tumor:; 1. in Kidney; 2. no calcifications

Q780. Dx:; child with morning Headache, vomiting (causing HA to go away); Ataxia, nystagmus, head-tilt, intention tremor; Specific site?

A780. Cerebellar Astrocytoma (MC childhood brain tumor); site: Infratentorial (posterior fossa)

Q781. Dx:; Child with short stature, Bitemporal hemianopsia and endocrine abnormalities; Dx test?; What can occur post-op?

A781. Craniopharyngioma(Supratentorial); Dx test: X-ray of skull: Large Cella Tursica; Post-op: Diabetes Insipidus

Q782. Dx:; A 3 - 5yo child presents with obstructive hydrocephalus, ataxia and CSF metastasis; Specific site?

A782. Medulloblastoma; site: Infratentorial

Q783. Dx:; Child stares into space in middle of sentence, then 20 seconds later he resumes his sentence; Dx test results?; Tx?

A783. Absence seizure; Dx test: EEG showing 3-Hz Spike and Wave pattern; Tx: Ethosuximide

Q784. MC type of seizure in pediatrics; Tx?

A784. Febrile Seizure; Tx: Acetaminophen (for fever) (check for underlying cause of seizure)

Q785. Dx test for Neonatal seizures; Tx?

A785. test: Continuous Bedside EEG; Tx: Phenobarbital

Q787. Difference in presentation with Niemann-Pick Disease and Tay-Sachs Disease?

A787. Both have Cherry-red spot on Macula; NP: Hepatosplenomegaly; TS: No hepatosplenomegaly

Q788. Dx:; child who constantly clears his throat

A788. Tourette's syndrome

Q789. Dx:; MENTAL RETARDATION, spasticity, aggressive behavior, self-mutilation, Gout arthritis

A789. Lesch-Nyhan

Q790. What are the (2) disorders where infants have Hypotonia or Flaccidity?

A790. 1. Werding-Hoffmann Disease; 2. Infant Botulism

Q792. Dx:; A child presents following a viral illness with Ascending weakness, Paralysis, Loss DTR, may progress to affect Diaphragm (breathing); Tx? (3 possible)

A792. Guillain-Barré; Tx:; (1) Supportive (intubation may be needed); (2) Plasmapheresis; (3) IVIG

Q793. Dx:; A boy 3 - 7 yo with Muscle and Hip girdle weakness and pseudohypertrophy of the calves; Genetics?; Dx Test?; Confirmatory test?

A793. Duchenne muscular dystrophy (MC inherited neuromuscular disease); genetics: X-linked recessive (only boys get it); Dx test: Blood shows greatly elevated Creatine Kinase; Confirmatory test: Muscle Bx

Q794. Name the sign:; a patient w/ DMD is walking their hands up their legs to sit/stand properly

A794. Gower's sign

Q795. Dx:; Café-au-lait spots, axillary/inguinal freckling, Learning disorders, Renovascular HTN, Scoliosis, Lisch nodules (seen on slit lamp); Genetics?; what chromosome has the problem?

A795. Neurofibromatosis (von Recklinghausen); genetics: Autosomal Dominant, chromosome: 17; (Type 2 - Bilateral Acoustic neuromas is chrom: 22)

Q796. Dx:; A child presents with pale hypopigmented areas, facial sebaceous adenomas, areas of abnormal skin thickening, MENTAL RETARDATION and Seizures are common; Genetics?; Dx Test/results?

A796. Tuberous Sclerosis; genetics: Autosomal Dominant; (Ash leaf spot - pale hypopigmented area); (Shagreen patch - areas of abnormal skin thickening); Dx Test: CT scan of Head: periventricular calcified tubers

Q797. Dx:; Facial nevus (port wine stain) with trigeminal distribution, MENTAL RETARDATION, seizures difficult to control, visual impairment; Dx Test/results?

A797. Sturge-Weber Disease; Dx Test: CT Scan of Head: intracranial calcifications

Q798. First sign of Puberty in:; 1. Females; 2. Males

A798. Females: Breast buds; Males: Testicular enlargement

Q799. What is the cause of Hypoglycemia in a newborn from a mother with DM?

A799. Fetal Hyperinsulinemia

Q800. What is the highest risk for a child to be born with developmental displasia of the hip?; What is the confirmatory test for DDH?

A800. Female Breech birth; test: Ultrasound of hip

Q801. Why does a mother with type O blood cause hemolysis in newborn if the child has type A or B?

A801. Type O people have Antibodies to blood types A and B

Q802. When should a child have their electrolytes checked if you suspect dehydration?

A802. Moderate or Severe dehydration; (not Mild dehydration)

Q803. Tx for Mild, Moderate and Severe Dehydration in children?

A803. Mild: Oral rehydration; Moderate/Severe: IV rehydration

Q804. What is considered Moderate dehydration percentage-wise?; Severe?

A804. moderate: 5-10% dehydrated; severe: &gt; 10% dehydrated

Q805. Dx:; Baby is tired when feeding, diaphoretic and breathing strangely

A805. CHF in newborn; (tired when feeding is #1 sign)

Q806. what are the (2) criteria for a Dx of failure to Thrive in a newborn?

A806. 1. failure to regain birth weight by 3 weeks of age; 2. continuous weight loss after 10 days of life

Q807. when a child has a decrease in respirations, what does it suggest?; increase in respirations?

A807. Dec: CNS depression; Inc: Infection

Q808. What are the (5) Tanner stages for Pubic hair?

A808. 1. None; 2. Scant, Fine hair; 3. Curly, extending laterally; 4. Adult-like hair, NOT on medial thigh; 5. Adult hair on medial thigh

Q814. What is the MC malignant tumor in infancy?

A814. Neuroblastoma

Q828. Pediatrics are usually started on 3 insulin injections per day. What is the dosing pattern?

A828. 1. 2/3 of total dose in morning (1/3 rapid or short-acting, 2/3 intermediate-acting); 2. 1/6 of total at dinner (as rapid or short-acting); 3. 1/6 of total before bed (as intermediate-acting)

Q829. A child has a seizure following the immunization. What is the cause?; What is done on the next immunization?

A829. Pertussis; Do NOT ever give that immunization again!

Q830. When a child is bit by a snake, how do you determine the venom dose?

A830. Based on the childs symptoms; (amount of envenomization)

Q831. Dx:; A 6-month-old infant presents to the emergency department with the new onset of weak cry, decreased activity, and poor feeding. The mother also states that the infant has been constipated for the past 2 days. On physical examination, the infant has a very weak cry, poor muscle tone, and absent deep tendon reflexes. Tx?

A831. Botulism; Tx: Supportive care (Airway PRN)

Q832. Dx:; A 16-year-old boy presents with a temperature of 38.4 C (101 F) and low back, wrist, and knee pain. He had a sore throat 1 month earlier. His arthritis is diffuse. Pea-sized swellings are noted over the skin on his knees. He has a serpiginous erythematous area on his anterior trunk. His blood and throat cultures are negative, and his CBC is unremarkable. His antistreptolysin-O (ASO) titer is high. Tx? (2 together)

A832. Rheumatic Fever; Tx:; 1. Penicillin; 2. Aspirin

"Q838. Dx:; A 9-month-old girl has had one serious infection after another since about 3 months of age, including thrush, pneumonias, and diarrhea. The baby is small for age. An older brother died at age 2 of pneumonia. Immunologic evaluation demonstrates lymphopenia and very low gamma globulin levels. Both T and B cell numbers are very low. Radiologic studies demonstrate ""frayed"" long bones, abnormally thick growth arrest lines, and dysplasia of the costochondral junctions"

A838. Severe Combined Immunodeficiency (SCID); (an Adenosine deaminase deficiency)

"Q839. Dx:; A 17-year-old girl presents to the office with a 5-day history of a malodorous vaginal discharge. She is sexually active and uses condoms for sexual intercourse. On examination, a thin, white discharge is seen. A ""fishy"" odor is produced when KOH is added to the discharge. The vaginal fluid has a pH of 5. What is the most likely finding on a microscopic examination of the vaginal fluid?"

A839. Bacterial Vaginosis; micro: Clue Cells

Q840. A 7-yo patient presents with Lyme disease. Tx?

A840. oral Amoxicillin; (doxycycline is not given to kids under age 8)

Q841. In a patient diagnosed with vesicoureteral reflux, what is the first Dx step?; What is the Dx/confirmatory Test (2nd step)?

A841. first: Renal Ultrasound; Dx/confirm Test:; Voiding Cystourethrogram

Q842. Dx:; a 12-yo girl presents with a fever and skin rash of 2 days duration. She complained of a sore throat and mild neck pain several days ago. The PE shows lymphadenopathy of the posterior occipital, retroauricular and cervical LN, plus an erythematous macules on the soft palate and a rash on the face, chest and proximal extremities.

A842. Rubella

Q843. Tx for Candida-induced oral thrush?

A843. Nystatin therapy

"Q844. A child presents with ""cat-scratch Disease.""; what is the bug?; what is the Dx/confirmatory test?"

A844. Bartonella henselae; test: Serum antibody titers

Q845. what is the bone complication of NF-1?

A845. Thinning of the Long Bone Cortex

Q846. what is the difference in RDW levels b/t thalassemia and Iron-deficiency anemia?

A846. RDW:; normal in Thalassemia; High in Iron-deficiency anemia

Q855. Dx:; a child with strep pharyngitis is given penicillin. The next day he develops a fine, papular rash over his body, which is accentuated in his axilla and groin.

A855. Scarlet Fever; (a rash from the penicillin would be urticarial in nature)

Q856. What are the (11)* criteria (of which 4 or more need to be present) to diagnose SLE?

A856. SOAP BRAIN MD:; 1. Serositis (pleuritis/pericarditis); 2. Oral ulcers; 3. Arthritis; 4. Photosensitivity; 5. Blood (any can be low); 6. Renal problems; 7. ANA+; 8. Immunological (ds-DNA+); 9. Nero Sx (psych, seizures); 10. Malar Rash; 11. Discoid Rash

Q857. what is seen in the serum of someone with a pneumonia caused by Mycoplasma?

A857. Cold Agglutinins

Q858. A 13-yo girl presents with lethargy, fever, severe HA and a stiff neck. On exam she has a unilateral fixed, dilated pupil and papilledema. What is the initial step in management?

A858. Intubation and Hyperventilation

Q859. what is antibiotic used for the initial therapy of acute sinusitis?

A859. Amoxicillin-clavulanic acid

Q860. In a child, if the monospot test comes back negative and you think the Dx is Mono, what is the next step?

A860. EBV titer; (heterophil test has a poor response in children)

Q861. How does a child with Breast milk Jaundice present?

A861. Prolonged unconjugated hyperbilirubinemia in first weeks/months of life

Q862. A 4-yo child presents with a platelet count of 30K and you diagnose her with ITP. what is the most appropriate Tx at this time?

A862. No specific therapy; (when platelets go below 20K start on oral prednisone; a splenectomy is done for those with no response to prenisone or those with recurrent ITP)

Q863. Dx:; A 12-yo athelete presents with chest pain and dyspnea on exertion that resolves with rest. First step in Tx?

A863. Asthma (exercise-induced); (&lt;5% of kids have heart problems, aside from congenital anomalies); Tx: Trial of Albuterol inhaler

Q871. A 3000g female is born at 38 weeks to a mother that is positive for GBS. What is the next step in managing the baby?

A871. CBC and blood culture; (in addition to 48 hours of observation in the hospital)

Q872. An infant is crawling around on the floor at home and suddenly become cyanotic. what is the first step?

A872. Back blows followed by chest thrusts; (never perform a blind finger sweep in a child, b/c it can push the aspirated object further)

Q873. why does a CF patient present with Metabolic Alkalosis?

A873. Potassium is lost in urine

Q874. MC presentation of Strep throat?; (4)*

A874. FEAT:; 1. Fever; 2. Exudate on Tonsils; 3. Anterior Cervical lymphadenopathy; 4. Throat pain

Q875. A full-term newborn is born vaginally after a pregnancy complicated by gestational diabetes that was poorly controlled. He is born with scleral icterus. Three hours after birth the baby is plethoric, irritable and cyanotic. Six hours after birth he has a seizure. What hematological abnormality is most likely seen on his CBC?

A875. Polycythemia; (a known complication in the infant of a diabetic mother)

Q876. Dx:; A previously healthy 3-mo term boy presents to ED with tachypnea, poor feeding and pallor for 24 hours. He is tachycardic with a HR of 240 with narrow QRS complexes. Vagal maneuvers are attempted and unsuccessful. What drug should be given?

A876. Supraventricular Tachycardia (SVT); drug: Adenosine (blocks AV conduction and SA-node pacemaker activity; DOC for SVT; if unavailable, can Cardiovert)

Q877. What is the most important risk factor for a PE in children?

A877. Central Venous catheter; (followed by immobility)

Q878. What hormone is responsible for growth plate fusion?

A878. Estradiol

Q879. Dx:; A patient presents with mild MENTAL RETARDATION, prominent lips and a long philtrum, a systolic ejection murmur and hypercalcemia.

A879. Williams syndrome

Q881. What is the MCC of retinopathy in a premie?

A881. Too much Oxygen saturation; (should be &lt;95%)

Q882. What occurs if you correct the following problems too fast?; 1. DI; 2. DKA; 3. SIADH

A882. 1. DI -&gt; Seizures; 2. DKA -&gt; Cerebral edema -&gt; Herniation; 3. SIADH -&gt; CPM

Q883. What additional (3) things are needed in formula for premature infants?

A883. 1. Extra Calories (22kcal/oz); 2. Extra Calcium; 3. Extra Phosphorus

Q884. What is a normal systolic BP in the following ages of children?; 1. Premie; 2. up to 1mo; 3. 1mo - 1yo; 4. &gt; 1yo

A884. 1. Premie = meanBP &gt; gestational age at birth; 2. up to 1mo = &gt;60; 3. 1mo - 1yo = &gt;70; 4. &gt; 1yo = 70 + (2 x age)

"Q885. Diarrhea that is caused by ""chitlins"" at a picnic"

A885. Vibrio

Q886. Dx:; Patient presents with absent LN, tonsils, lymphopenia, no thymus, recurrent infections of oral candida, diarrhea, pulmonary infections and viral infections

A886. SCID; (Severe Combined Immunodeficiency Disease)

Q887. MC heart defect in congenital Rubella

A887. PDA

"Q888. Dx:; An infant presents with noisy breathing and on laryngoscopy the epiglottis is seen ""rolling from side-to-side""; Tx?"

A888. Laryngomalacia; Tx: hold infant upright for 30 minutes after feeding; (most disappear by 2 yo)

Q889. What type of Pneumonia is seen in Hyper-IgM syndrome?

A889. PCP

"Q891. Dx:; a child presents with a slowly developing back pain and neuro degeneration with a ""step-off"" palpated at lumbosacral area"

A891. Spondylolisthesis

Q892. Dx:; an infant presents with Cyanosis, Left Axis Deviation and VSD; what does EKG show?

A892. Tricuspid Atresia; EKG: LVH (and hypoplastic RV)

Q893. Dx:; a cyanotic newborn with a CXR showing cardiomegaly, increased vascular markings and right aortic arch. what does the EKG show?

A893. Truncus Arteriosus; EKG: Biventricular Hypertrophy

Q894. what are the (4)* main features of Beckwith-Wiedmann syndrome?

A894. HOMO:; 1. Hypoglycemia &amp; Hyperinsulinemia; 2. Omphalocele; 3. Macroglossia &amp; Macrosomia; 4. Organomegaly

Q895. If a child presents with asthma and other allergic disorders (allergic rhinitis or eczema), what is the best long term medication?

A895. Mast Cell Stabilizers; (inhaled Cromolyn)

Q896. Dx;; A 12-mo with foul-smelling, non-bloody stool 7-8 times/day, irritable, pre-tibial edema, erythematous vesicles on extensor surfaces of elbows and knees and microcytic anemia

A896. Celiac Disease; (begins at age 12 - 15 mo)

Q897. MCC of Meningitis in following ages:; 1. Newborn; 2. 1-mo to 2-yo; 3. 2-yo to 18-yo; 4. &gt; 18-yo

A897. 1. Newborn = GBS; 2. 1-mo to 2-yo = S. Pneumonia; 3. 2-yo to 18-yo = N. Meningitis; 4. &gt; 18-yo = S. Pneumonia

Q898. Dx:; a few days after birth a mother realizes that her child cannot move his hand on the right side and also has right lid ptosis

A898. Klumpke Paralysis; (birth injury to CN-7,8 and T1)

Q899. What reflex is affected by a child born with E-D palsy?

A899. Moro relex is absent

Q900. What is the test of choice in evaluating a child for Hyperlipidemia?

A900. Screening Total Cholesterol level; (not Fasting Lipid Profile)

Q901. What are the nodular swellings of the irises in NF-1 patients?

A901. Hammartomas

Q902. When does Physiologic Jaundice begin?; Breast milk Jaundice?

A902. Physiologic Jaundice: starts 2nd or 3rd day of life; (resolves in several weeks); Breast milk Jaundice: starts after 1 week-old

Q903. If an infant presents with a fever and no other Sx, what is the best first test?

A903. Catheterized UA with culture

"Q971. Dx:; ""sea-blue"" histiocytes"

A971. Niemann-Pick

Best initial test for duodenal atresia

AXR

What does obstruction refer to in the setting of Total anomalous pulmonary venous return?

Angle at which the veins enter the sinus

Most valuable clue in the diagnosis of Wilms tumor

Aniridia

How do you do the Ortolani maneuver?

Anterior force through greater trochanter + hip abduction

What is the next best step in the management of: a 3-day-old preterm female neonate w/ ↑ gastric residual volume, abdominal distention, heme positive stool, lactate 2.9 mg/dL, AXR showing air in bowel wall?

Antibiotics (when evidence confirms necrotizing enterocolitis)

Color of light used for phototherapy

Blue-green

Subperiosteal hemorrhage that *does not* cross suture lines

Cephalohematoma

What happens to the vast majority of ASDs?

Close spontaneously

What are some causes of abnormal appearance in a pediatric patient who has suffered a traumatic event?

Closed head injury, hypoxia, and pain

Congenital narrowing of the aorta in the area of the ductus arteriosus

Coarctation of the aorta

Most likely diagnosis w/ *"3"-like appearance or rib notching* cardiac x-ray findings

Coarctation of the aorta

What findings are diagnostic for imperforate anus?

Complete failure to pass meconium w/ no anus on PE

Congenital hypothyroidism can result in?

Cretinism

The narrowest part of a young child's airway

Cricoid Ring

Absence of one testicle in the scrotum

Cryptorchidism

There is a decreased incidence of Whooping cough in what population?

DTaP vaccine recipients

Reasoning for NG tube placement in duodenal atresia

Decompress bowel

Features of Convalescent stage of Whooping cough

Decrease of frequency of coughing

Rationale for administering racemic epinephrine in patient w/ RSV

Decrease swelling to prevent asphyxiation & tracheostomy

An early sign of shock.

Decreased skin perfusion

Abdominal x-ray findings associated w/ duodenal atresia

Double bubble sign

Ultrasound findings in a patient w/ intussussception

Doughnut/target sign

Definitive diagnosis of TAPVR w/ obstruction is made by?

Echocardiography

How is diagnosis of TAPVR without obstruction made?

Echocardiography

Less invasive, effective diagnostic test for ASD

Echocardiography

What can a Meckel's diverticulum contain?

Ectopic gastric tissue

Esophagus that ends blindly

Esophageal atresia

Third trimester bleeding occurs in 4% of all pregnancies and is considered normal

False

Meconium staining is a sign of

Fetal defecation

90% of normal infants pass their first meconium within?

First 24 hours

What causes the normalization of hemoglobin in transient polycythemia of the newborn?

First breath increases O2 → ↓ erythropoeitin

When do symptoms of Truncus arteriosus typically present?

First few days of life

What does the Pediatric Assessment Triangle help you with when assessing a pediatric patient?

First impression

Physical features associated w/ oligohydramnios

Flat facies

Stool study findings for necrotizing enterocolitis

Frank or occult blood

Brain tissue decrease. CSF decrease. Blood volume decrease.

How will the brain compensate for increased ICP?

Painless, swollen fluid-filled sac along the spermatic cords within the scrotum that transilluminates upon inspection

Hydrocele

Mechanism behind acidosis seen w/ CAH

Hydrogen ion retention

Your 7 year old patient is lethargic and has a blood pressure of 70/50 mm Hg and a pulse of 138 beats/min. Respirations are 40 breaths/min, and his breath smells fruity. His mother says he has been losing weight and has had increased urination and thirst for several weeks. What condition should be considered

Hyperglycemia

Are 11β-hydroxylase deficiency patients hyper- or hypotensive?

Hypertensive

Metabolic imbalance worsened by aldosterone released in response to hypovolemia associated w/ pyloric stenosis

Hypokalemia

Opening of the urethra found on the ventral surface of the penis

Hypospadias

A late/ominous sign of shock. A pedi PT maybe in shock at normotensive.

Hypotension

Are 21-hydroxylase deficiency patients hyper- or hypotensive?

Hypotensive (salt-wasting shock)

Most common presentation of CAH

Hypotensive child w/ severe electrolyte abnormalities

95 degrees or less

Hypothermia is a body temp of...

Volume status associated w/ pyloric stenosis

Hypovolemia

Pathophysiology of transient polycythemia of the newborn

Hypoxia during delivery stimulates erythropoeitin → ↑ circulating RBCs

Abnormal interictal pattern, consisting of high amplitude and irregular waves and spikes in a background of chaotic and disorganized activity seen on electroencephalogram (EEG)

Hypsarrhythmia

Best first step in management of duodenal atresia, volvulus, & intussussception

IV fluids

Most appropriate next step in management of severe dehydration

IV fluids + electrolytes

What is the most appropriate next step in management of: an 11-month-old girl, in the ED for severe diarrhea, fever of 103°F, lethargy/listlessness/drowsy, not eating, several episodes of diarrhea, last episode was bloody w/ mucus, skin tenting, marked leukocytosis, ↑ BUN & creatinine, ↓ HCO3-, ↑ hematocrit?

IV fluids + electrolytes

What must you differentiate a hydrocele from?

Inguinal hernia

How is Legg-Calvé-Perthes disease diagnosed?

Joint effusion & widening on XRs

Electrolyte often low from vomiting in patients w/ duodenal atresia

K+

Condition associated w/ infertility & situs inversus

Kartagener syndrome

Most serious complication of pathologic jaundice in the newborn where deposition of bilirubin in the basal ganglia occurs

Kernicterus

Cause of vitamin D-*deficient* rickets

Lack of enough vitamin D in child's diet

What does full thickness biopsy of a patient w/ Hirschsprung disease show?

Lack of ganglionic cells in the submucosa (Auerbach plexus)

MVAs

Leading cause of traumatic pediatric death.

When treating a child, it is important to gain their trust. One way to accomplish this is by sitting or lying on the floor with them. What some other techniques you can use to help develop trust?

Let them play with a stethoscope or pen light, and by playing games

Most common type of skull fracture in the newborn

Linear

Most likely diagnosis in a patient w/ hearing loss, syncope, normal vitals/exam, family history of sudden cardiac death

Long QT syndrome

Temp Bark cough stridor

Main presentation of croup (3)

Adrenal medulla tumor similar to pheochromocytoma but w/ fewer cardiac manifestations

Neuroblastoma

Statistically most common cancer in infancy

Neuroblastoma

Pressers and stabilize injury.

Neurogenic shock treatment

When are lumbar puncture w/ CSF analysis and culture done in newborns?

Neurological signs (irritability, lethargy, temperature irregularity, and feeding problems)

Presentation of duodenal atresia

Onset of bilious vomiting within 12 hours of birth

Which two major fuels will NOT be delivered to the brain when there is inadequate profusion?

Oxygen and glucose

Most accurate diagnostic test for Herpes

PCR

Most accurate diagnostic test for toxoplasmosis

PCR

Failure of spontaneous closure of the ductus arteriosus

PDA

Normal cardiac finding in the first 12 hours of life

PDA

Best initial test for varicocele

PE

When does a PDA usually close?

PO2 ↑ 50 mm Hg

Presentation of Legg-Calvé-Perthes disease

Painful limp

sub-q

Peds have less **** fat

Painless, bright red bleeding, with or without uterine contraction, is most suggestive of

Placenta previa

Makes it so it never happened

Primary prevention

What keeps a PDA open?

Prostaglandin E1

Lack of abdominal muscles, so unable to bear down and urinate

Prune belly

Effect of 17ɑ-hydroxylase deficiency on sex development of boys

Pseudohermaphroditism

BG>200 mg/DL

Pt is hyperglycemic at...

- Sign where, on palpation of the pulse, a double peak per cardiac cycle can be appreciated - In *aortic regurgitation*

Pulsus bisferiens

- An abnormally large decrease in systolic blood pressure (> 10 mm Hg) and pulse wave amplitude during inspiration - In *cardiac tamponade* and *tension pneumothorax*

Pulsus paradoxus

- Slow-rising pulse and anacrotic pulse; is a sign where, upon palpation, the pulse is weak/small, and late relative to its expected characteristics - In *aortic stenosis*

Pulsus tardus et parvus

Associate the *string sign* w/ its associated condition

Pyloric stenosis

Condition in which a hypertrophic pyloric sphincter prevents proper passage of GI contents from the stomach into the duodenum

Pyloric stenosis

Projectile vomiting + palpable abdominal mass are characteristic of

Pyloric stenosis

Cholinergenic

SLUDGE is an acronym for what OD?

Where does oxygenated blood instead return in Total anomalous pulmonary venous return?

SVC

Infection characterized by diffuse erythematous eruption that is concurrent w/ pharyngitis caused by erythrogenic toxin

Scarlet fever

Two key areas of development during the _____ years include an increased ability to cocentrate and learn quckly and the onset of puberty. Their psychosocial development varies by individual, but as a rule however, self-concept and moral traits and behavior begin to emerge during thse years

School age

CXR findings w/ RSV

Steeple sign

Reflex characterized by walking-like maneuvers when toes touch the ground

Stepping reflex

Because pediatrics are more susceptible to hypothermia, you should check capillary refill at ____ or _____.

Sternum or forehead.

Treatment of mild Croup

Steroids

Organism that causes Scarlet fever

Streptococcus pyogenes

Sign observed on upper GI series in pyloric stenosis characterized by *thin column of barium leaking through the tightened muscle*

String sign

What is the most likely finding on upper GI series in: a 1-month-old child presenting w/ forceful nonbloody, nonbilious vomitus after feeding that winds up across the nursery and a palpable abdominal mass on PE?

String sign

Minute hemorrhages present in the eyes of infant due to rapid rise in intrathoracic pressure as chest is compressed while passing through birth canal

Subconjunctival hemorrhage

What does auscultation reveal in a patient w/ pyloric stenosis?

Succussion splash

Treatment of Mumps

Supportive

Often the first sign of shock in pediatrics

Tachycardia

Improves outcome after accident

Tertiary prevention

Why are children 6-24 months of age at the highest risk for developing rickets?

Their bones are rapidly growing

Abdominal ultrasound findings associated w/ pyloric stenosis

Thickened pyloric sphincter

Pneumonia

This lower airway disease presents with grunting, crackles or wheezing and anorexia.

Bronchiopulmonary Dysplasia

This respiratory disease is common with premature kids.

Best initial imaging study for Wilms tumor

Ultrasound

Best initial test for intussussception

Ultrasound

Prognosis of smaller VSDs

Usually close in first 1-2 years

4j/kg

V-fib/pulseless V-tach shock

Commonly tested syndrome associated w/ imperforate anus

VACTERL syndrome

Best initial diagnostic test for syphilis

VDRL/RPR

Condition that presents w/ holosystolic murmur and symptoms of failure to thrive

VSD

Varicose vein in the scrotal veins causing swelling of pampiniform plexus and increased pressure

Varicocele

What findings would be your first indication that an infant needs a fluid bolus

Very dry mucous membranes & tachycardia

Levels of Vitamin K in breast milk

Very low

Most accurate diagnostic test for Varicella

Viral culture

Croup Laryngotracheobronchitis

Viral infection that effects child (3 months - 6 years) in fall and winter season.

Effect of 11β-hydroxylase deficiency on sex development of girls

Virilized

Bowel obstruction in which a loop of bowel has twisted on itself abnormally

Volvulus

On the USMLE, hypochloremic, hypokalemic metabolic alkalosis is almost always caused by

Vomiting (vast loss of hydrogen ions)

A significant volume loss can be caused by these

Vomiting and Diarrhea

How is slipped capital femoral epiphysis diagnosed?

Widening of joint space on XRs

Most common abdominal mass in children

Wilms tumor

glucose

With ICP, you should avoid...

When does pulmonary hypertension develop in the setting of Truncus arteriosus?

Within 4 months

Typical duration for resolution of hydrocele

Within 6 months

G6PD deficiency inheritance pattern

X-linked recessive

Best diagnostic test when suspecting clavicular fracture

XRs

Child Abuse usually occurs in

Young Infant, Infant, Toddler, School-Age children, & Adolescent

You are treating an 11-month-old child that has an epidural hematoma. What can occur in a child this age that cannot occur in a child 11 years of age with the same injury?

a. Blood loss creating shock b. Blurred and double vision c. Decreased respiratory rate

CXR findings associated w/ VSD

↑ vascular markings

Treatment of PKU

↓ phenylalanine diet for first 16 years (at least)

Term neonate

≥ 38 weeks

Atropine Sulfate

0.02 mg/kg

Epinephrine (1:10,000)

0.1 mL/kg

Adenosine

0.1 mg/kg

CXR findings associated w/ Transposition of the great vessels

"Egg on a string"

Appearance associated w/ Duchenne Erb paralysis

"Waiter's tip" appearance

Positive Ortolani maneuver

'Clunk' heard as femoral head relocates into acetabulum

About ____ of newborns require some assistance to begin breathing at birth

0.1

Q338. Intussusception - History/PE

"A338. History - colicky abdominal pain in apparently healthy kids; n/v; too young to talk - cry; draw knees up to chest; dyspnea with pain; advanced signs - red ""current jelly"" stool; lethargy; fever; PE - abdominal tenderness, positive stool guaiac, palpable ""sausage-shaped"", RUQ abdominal mass"

Q367. Kawasaki Disease; (Mucocutaneous Lymph Node Syndrome) - History/PE

"A367. ""CRASH and Burn""; Conjunctivitis - b/l; Rash; Adenopathy - cervical lymph; Strawberry tongue; Hands and feet - swollen, red, desquamation; fever &gt; 40C for &gt; 5 days"

Q388. Coarctation of the Aorta - Dx

"A388. EKG - LVH; echocardiography; color-flow doppler; CXR - ""3"" sign: rib notching; aortography - diagnostic (cardiac catheterization)"

Q392. Transposition of the; Great Arteries - Dx

"A392. Echocardiography; CXR - ""egg-shaped sihouette"", ""egg on a string"", ""apple on a string"""

Q421. Gaucher's Disease - (lysosomal storage disease); What is it

"A421. AR; deficiency of B-glucocerebrosidase; glucocerebroside accums in; brain, liver, spleen, bm; Gaucher's cells - ""crinkled paper"" enlarged; cytoplasm; Type I - more common, compa-; tible with normal life span"

Q430. Erythema Infectiosum - (Fifth Disease); Cause; Characteristics

"A430. Cause - parvovirus B19; prodrome none; fever absent or low-grade; ""slapped cheek"" erythematous; pruritic; maculopapular rash; goes to arms; spreads to trunk and legs; worse with fever and sun"

Q439. Varicella - Cause; Characteristics

"A439. VZV; prodrome - mild fever; anorexia; malaise; precedes rash by 24 hrs; generalized, pruritic,; ""teardrop"" vesicular rash; starts on trunk; spreads to periphery; lesions often at different; stages of healing; infectious from 24 hrs; before eruption til; lesions crust over"

Q441. Varicella Zoster - Cause; Characteristics

"A441. Prodrome - reactivation of Varicella; infection; starts as pain along affected; sensory n. pruritic ""teardrop""; vesicular rash; in dermatomal distribution; uncommon unless immunocomp"

Q711. (4) heart defects of the Tetralogy of Fallot; MC presenting Sx?

"A711. ""PROVe""; 1. Pulmonary stenosis; 2. RV hypertrophy; 3. Overriding Aorta; 4. VSD; MC presenting Sx: Cyanosis"

Q713. Dx:; MCC of cyanosis in the first 24 hours of life; loud single S2; What is seen on CXR?; Tx (aside from airway) until surgery?

"A713. Transposition of Great Vessels; CXR: ""Egg on a string""; (ECG will show right axis deviation and RVH); Tx: Prostaglandin (to keep PDA open until surgery)"

Q732. Dx:; a child between the ages of 6 - 18 months presents with crampy abdominal pain, Currant jelly-like stool, Sausage- shaped abdominal mass; Dx test?

"A732. Intussusception; Dx test: Barium Enema (will see ""Coil spring sign""; also treats problem)"

Q738. Dx:; infant with Toes AND heel deviated inward; Cannot place in neutral, dorsiflex or plantarflex; Tx?

"A738. Talipes equinovarus (""Club Foot""); Tx: Serial Casting (only intoeing needing treatment)"

Q742. Dx:; Toddler with sudden traction to arm causing it to hang limp; Tx?

"A742. Radial head subluxation (""nursemaid injury""); Tx: gentle Supination pops it back into place"

Q812. What are (4) developmental milestones at 6 months old?

"A812. ""Six Babbling Strangers Switch Sitting at 6 months"":; 1. Babbles; 2. Stranger recognition; 3. Switches hands (transfers objects); 4. Sitting unsupported"

Q820. What is the difference b/t HUS and TTP?; (2)

"A820. No Fever and Neurological signs in HUS; HUS = ""RAT"" (Renal failure, Anemia, Thrombocytopenia); TTP = ""FAT RN"" (Fever, Anemia, Thrombocytopenia, Renal problems, Neuro problems)"

Q824. In assessing dehydration, what are the (7)* main findings to evaluate?; How many findings equal &lt;5% dehydration?; More then 10% dehydration?

"A824. having Dehydration is ""BUM NEWS"":; 1. BP and HR; 2. Urine output; 3. Mucous membranes and Tears; 4. Neurological status; 5. Eye and Fontanels; 6. Weight; 7. Skin turgor/color and Capillary refill; &lt;5% = &lt;3 findings; &gt;10% = &gt;6 findings"

Q880. What are the erruption times of permanent teeth?; (8)*

"A880. ""Mama Is In Pain, Papa Can Make Medicine"":; 1st Molar: 6 years; 1st Incisor: 7 years; 2nd Incisor: 8 years; 1st Premolar: 9 years; 2nd Premolar: 10 years; Canine: 11 years; 2nd Molar: 12 years; 3rd Molar: 18-25 years"

Q890. what are the (4)* MC congenital problems in infant of a diabetic mother?

"A890. ""A Diabetic Causes Trouble"":; 1. Ancephaly / NT defects; 2. Duodenal Atresia / small left colon; 3. Caudal Regression syndrome; 4. Transposition of the Great Vessels"

Q904. what is the first step in Tx for a patient with Allergic Rhinitis?

"A904. Antihistamines; (do be fooled with ""environmental change"")"

Rule of 2s for Meckel's diverticulum

- 2% of population - 2 feet from ileocecal valve - 2 types of ectopic tissue (gastric and pancreatic) - 2X ↑ prevalence in males - < 2 years of age - 2% of patients are symptomatic - 2 inches long

Treatment of Hypoplastic left heart syndrome

- 3 separate, high-mortality surgeries - Heart transplant

What does APGAR stand for?

- A: Appearance - P: Pulse - G: Grimace - A: Activity - R: Respiration

Best first & initial step in management of choanal atresia

- ABCs - NG tube

Screening for omphalocele

- AFP levels - Ultrasound

Diagnostic tests done for necrotizing enterocolitis

- AXR, CT - Stool studies

W/ Duchenne Erb paralysis, the infant is unable to carry out what motions?

- Abduction - External rotation - Supination

Presentation of Hypoplastic left heart syndrome

- Absent pulses w/ single S2 - ↑ RV impulse - Gray cyanosis (rather than bluish)

Signs of severe dehydration

- Acute renal failure (↑ BUN & creatinine) - Skin tenting - Hemoconcentration

Findings on upper GI series assocated w/ voluvlus

- Air fluid levels - "Bird beak" appearance (at site of rotation)

CT findings for perforated necrotizing enterocolitis

- Air in the portal vein - Dilated bowel loops - Pneumoperitoneum

Levels of aldosterone, cortisol, & sex hormone characteristic of *17ɑ-hydroxylase deficiency*?

- Aldosterone: ↑ - Cortisol: ↓ - Sex hormone: ↓

Levels of aldosterone, cortisol, & sex hormone characteristic of *11β-hydroxylase*?

- Aldosterone: ↓ - Cortisol: ↓ - Sex hormone: ↑

Levels of aldosterone, cortisol, & sex hormone characteristic of *21-hydroxylase*?

- Aldosterone: ↓ - Cortisol: ↓ - Sex hormone: ↑

CAH's general effect on sexual development in girls

- Ambiguous genitalia/virilization - Menstrual abnormalities - Inappropriate facial hair

Treatment of neonatal sepsis

- Ampicillin and gentamicin - Cefotaxime

Presentation of Vitamin B2 (riboflavin) deficiency

- Angular chelosis - Stomatitis - Glossitis

Duodenal atresia associations

- Annular pancreas - Down syndrome

Diseases screened for prior to discharge of neonate

- Biotinidase - β-thalassemia - CAH - Cystic fibrosis - Galactosemia - Hypothyroidism - Homocysteinuria - PKU

CXR findings associated w/ Tetralogy of Fallot

- Boot-shaped heart - ↓ pulmonary vascular markings

When does hypertrophy of the pylorus become pronounced?

- By 1st month of life - Not commonly found at birth

Components of a full sepsis evaluation

- CBC w/ differential - Urinalysis & urine cultures - CXR - Lumbar puncture

Complications of untreated VSD

- CHF - Endocarditis - Pulmonary hypertension

How is the diagnosis of esophageal atresia + TEF made?

- CXR - CT - Esophagram

Levels of calcium, phosphate, 1,25(OH)2, and 25(OH) characteristic for X-linked hypophosphatemic

- Calcium: normal - Phosphate: ↓ - 1,25(OH)2: normal - 25(OH): normal

Levels of calcium, phosphate, 1,25(OH)2, and 25(OH) characteristic for vitamin D-*deficient* rickets

- Calcium: normal or ↓ - Phosphate: ↓ - 1,25(OH)2: ↓ - 25(OH): ↓

Levels of calcium, phosphate, 1,25(OH)2, and 25(OH) characteristic for vitamin D-*dependent* rickets

- Calcium: ↓ - Phosphate: normal - 1,25(OH)2: ↓ - 25(OH): normal

Effect of laying down on RSV patients

- Causes difficulty breathing - May show signs of hypoxia

Causes of conjunctivitis in the newborn

- Chemical irritation - Neisseria gonorrhoeae - Chlamydia trachomatis - Herpes simplex

Presentation of toxoplasmosis

- Chorioretinitis - Hydrocephalus - Multiple ring-enhancing lesions on CT

Pathophysiology of stroke from paradoxical emboli from deep leg veins in 17 y/o

- Clot formed in setting of venous stasis - Traveled to brain through patent ASD

Set of congenital defects seen w/ CHARGE syndrome

- Coloboma of the eye, CNS anomalies - Heart defects - Atresia choanae - Retardation of growth/development - GU, genital defects (hypogonadism) - Ear anomalies, deafness

Presentation of Rubeola/Measles

- Cough, coryza, conjunctivitis - Koplik spots (grayish macule on buccal surface)

Hypospadias is associated w/?

- Cryptorchidism - Inguinal hernias

Characteristics of PKU

- Deficiency in phenylalanine hydroxylase - Leads to mental retardation

Presenting features of Hirschsprung disease

- Delayed (>48 hours) or failure to pass first meconium - Extreme constipation - Large bowel obstruction - Extremely tight sphincter - Failure to pass flatus

Indications for treatment of varicocele

- Delayed growth of testes - Evidence of testicular atrophy

Treatment of Transposition of the great vessels

- Maintenance of PDA w/ prostaglandin E1 - 2 separate surgeries

What is the diagnosis and most common complication in: a 1-day-old child who begins to choke and exhales milk bubbles from her nose upon her first feeding, appears to be in significant respiratory distress, CXR reveals an air bubble in the upper esophagus, no gas pattern in the remainder of the GI tract, and a coiled NGT?

- Diagnosis: esophageal atresia w/ tracheoesophageal fistula - Most common complication: aspiration pneumonia

Diagnostic tests done for suspected pathologic jaundice in the newborn

- Direct & indirect bilirubin levels - Blood type of infant & mother (for ABO & Rh incompatibility) - Peripheral smear & reticulocyte count

Findings on plain x-ray for Hirschsprung disease

- Distended bowel loops - Lack of air in rectum

3 conditions associated w/ bilious vomiting

- Duodenal atresia - Volvulus - Intussussception

Future complications associated w/ ASDs

- Dysrhytmias - Paradoxical emboli from DVTs

Types of esophageal fistulas

- EA + distal TEF - Isolated EA w/o TEF - EA + proximal & distal TEFs - H-type TEF

How is asymmetric septal hypertrophy in an IDM diagnosed?

- EKG - Echocardiography

Presentation of Transposition of the great vessels

- Early/severe cyanosis - Single S2 is heard

Other symptoms associated w/ infectious diarrhea caused by *Norwalk virus*

- Explosive - Cramping, pain

Major findings on exam w/ imperforate anus

- Extreme constipation - Absence of anal orifice

Presentation of Epiglottitis

- Fever - "Hot potato" voice - Drooling in the tripod position - Refusal to lie flat

Presentation of inflammatory diarrhea

- Fever - Abdominal pain - Possible bloody diarrhea

Other symptoms associated w/ infectious diarrhea caused by *Rotavirus & Adenovirus*

- Fever - Emesis - No blood

Presentation of Mumps

- Fever - Parotiditis - Orchitis

Presentation of Scarlet fever

- Fever - Pharyngitis - Sandpaper rash over trunk & extremities - Strawberry tongue - Cervical lymphadenopathy

Presentation of bronchitis

- Fever - Productive coughing (7-10 days)

Presentation of Roseola

- Fever + URI - Progressing to diffuse rash

Presentation of pharyngitis

- Fever above 104℉ - Cervical adenopathy - Petechiae - Other URI symptoms - Acute rheumatic fever - Glomerulonephritis

Findings in an IDM suggestive of renal vein thrombosis

- Flank mass - Bruit - Hematuria - Thrombocytopenia

Treatment of esophageal atresia

- Fluid resuscitation - Antibiotic coverage for anaerobes - 2-step surgical repair

Treatment of CAH

- Fluids + electrolytes - Lifelong steroids - Psychiatric counseling

Without vitamin K-dependent clotting factors, newborn will likely bleed from where?

- GI tract - Belly button - Urinary tract

CXR findings associated w/ esophageal atresia + TEF

- Gastric air bubble + esophageal air bubble - Coiling of NG tube

Treatment of facial nerve palsy

- Gradual improvement over weeks-months - Surgical nerve repair w/ failed recovery

Most common organisms associated w/ early onset sepsis

- Group B Streptococcus - E. coli - Listeria

Most common organisms causing sepsis

- Group B Streptococcus - E. coli - S. aureus - L. monocytogenes

Presentation of VSD

- High-pitched holosystolic murmur over LLSB - Loud pulmonic S2 - Dyspnea w/ respiratory distress

Presentation of Vitamin D toxicity

- Hypercalcemia - Polyuria - Polydipsia

Effect of hypomagnesemia in an IDM

- Hypocalcemia - PTH decrease

Metabolic findings in an IDM

- Hypoglycemia - Hypocalcemia - Hypomagnesemia - Hyperbilirubinemia

Electrolyte abnormalities associated w/ 21-hydroxylase deficiency

- Hyponatremia - Hypochloremia - Hyperkalemia

Electrolyte abnormalities observed w/ CAH

- Hyponatremia - Hypocholoremia - Hypoglycemia - Hyperkalemia - Acidosis

Highly tested hallmark findings associated w/ neuroblastoma

- Hypsarrhythmia (on EEG) - Opsomyoclonus - ↑ VMA & metanephrines (diagnostic)

Treatment of asymmetric septal hypertrophy in an IDM

- IV fluids - β-blockers

Treatment of pyloric stenosis

- IV fluids + electrolytes (esp. *K+*) - NG tube (for decompression) - Surgical myotomy

Effect of hyperbilirubinemia in an IDM

- Icterus - Kernicterus

Causes of intussussception

- Idiopathic (most common) - Polyp - Hard stool - Lymphoma - Viral origin

Treatment of clavicular fracture

- Immobilization - Splinting - Physical therapy

Causes of polyhydramnios

- Inability to swallow (e.g., Werdnig-Hoffman disease) - Atresias (e.g., intestinal atresias)

Type of esophageal fistula present in ≤20% of cases

- Isolated EA w/o TEF - EA + proximal & distal TEFs - H-type TEF

Presenting features of an IDM

- Macrosomia - Plethora - Jitteriness

Findings in infants of diabetic mothers (IDM)

- Macrosomia - Small left colon syndrome - Cardiac abnormalities - Renal vein thrombosis - Metabolic abnormalities

If respirations are inadequate (evidenced by low SpO2 readings) or if the heart rate remains less than ____ beats per minute 30 seconds after administering the initial steps following birth, positive-pressure ventilation should be initiated

100

A fetal heart rate that remains above 160 or below ___ beats/min for more than 60 seconds may be an early sign of fetal distress

110

Duration of Convalescent stage of Whooping cough

14 days

Duration of the Catarrhal stage of Whooping cough

14 days

Duration of the Paroxysmal stage of Whooping cough

14-30 days

Q019. rash pattern for erythema infectiosum

A019. slapped cheek rash;; lacy, reticular

Most likely diagnosis w/ *jug-handle appearance* cardiac x-ray findings

1° pulmonary artery hypertension

Dopamine Hydrochloride

2 to 20 mcg/kg/min

Ophthalmia neonatorum prophylaxis

2 types of antibiotic drops: - Erythromycin or tetracycline ointment - Silver nitrate solution

Spontaneous abortion is the nontherapeutic termination of pregnancy from any cause before ____ weeks of gestation

20

A 20 kg child is lethargic & tachycardic and has dry mucous membranes after a 3 day history of flu. Medical direction asks for a fluid bolus of normal saline. How much will you administer initially

400 mL

≥ 90% of CAH cases are due to deficiency in what enzyme?

21-hydroxylase deficiency

ASD more common in men or women?

2X in women

Children get defribillation at

2j/kg then 4j/kg

Duration of Scarlet fever

3-6 days

Management of Hirschsprung disease

3-stage surgery procedure

Mortality % of necrotizing enterocolitis

30%

Late preterm neonate

34-37 weeks

Drownings

4th leading cause of death from 1-4 years

Amiodarone

5 mg/kg

Mortality rate associated w/ surgical therapy for Transposition of the great vessels

50%

% of neuroblastoma cases presenting w/ metastases

50-60%

What % of amniotic fluid is a filtrate of the mother's plasma?

80%

Pyloric stenosis can present as late as

6 months after birth

Typical duration of infectious diarrhea caused by *Rotavirus & Adenovirus*

< 7 days

After how many hours of life is a PDA considered pathologic?

> 24 hours

> what % of newborn infants are jaundiced?

> 60%

When compared with an adult, how is a child's tongue different?

A child's tongue is larger

An incomplete brake in the boken.

A greenstick fracture is...

Classic migraine

A migraine with an aura is a....

9-12

A moderate brain injury has a GCS of...

scarlet fever

A rash all over the body might be...

8 or less

A severe head injury has a GCS of...

A Fontanelle during dehydrations

A sunken fontenalle

Q020. VZV rash pattern

A020. begins on trunk,; followed by head, face, and extremities

What causes a child to have an increased need for oxygen when injured or ill?

A. Decreased oxygen use** B. Increased metabolism C. Decreased temperature

When using the Pediatric Assessment Triangle, what kind of treatment approach should be used?

A. During transport B. Treat as you go** C. Very aggressive

Q021. gross motor skills at 1 mo old

A021. raises head

Q022. gross motor skills at 3 mo

A022. holds head up

Q017. rash pattern in rubella

A017. rash on face that spreads to the rest of the body

What developmental difference causes children to have increased internal organ damage when suffering blunt chest-wall trauma?

A. Thicker chest wall B. Softer chest wall C. Smaller surface area **

Q018. rash pattern in measles

A018. rash starts at the head and spreads downwards and disappears in the same manner

Q001. vaccinations given at birth

A001. hepatitis B

Q002. vaccinations given at 1 month

A002. hep B, only if not given at birth

Q003. vaccinations given at 2 months

A003. Pediarix (HBV, DTaP, IPV); Prevnar (pneumococc); Rotavirus; Hib

Q004. what is in pediarix

A004. HBV; DTaP; IPV

Q005. vaccinations given at 4 months

A005. Pediarix; Prevnar; Hib; Rotavirus

Q006. vaccinations given at 6 months

A006. Pediarix; Prevnar; Hib; Rotavirus

Q007. vaccinations given at 12 months

A007. DTaP; Hib; MMR; VZV; Prevnar; Influenza; Hep A

Q008. vaccinations given at 24 months

A008. none

Q009. vaccinations given at 4-6 yrs

A009. DTaP; IPV; MMR; Influenza

Q010. vaccinations given at 11 yo

A010. Tdap; Meningococcus vaccine

Q011. what is the schedule for well visits starting from birth

A011. 1 mo; 2 mo; 4 mo; 6 mo; 9 mo; 12 mo; 15 mo; 18 mo; 24 mo; then annually

Q012. why shouldn't babies drink water

A012. because their kidneys aren't mature enough to handle the extra fluid so the babies will become hyponatremic since they can't excrete the water, and then they can have seizures

Q013. plagylocephaly

A013. mishapen head

Q014. sunsetting sign

A014. increased ICP, eyes are half closed secondary to the increased ICP on the cranial nerves

Q015. craniosynostosis

A015. premature closure of the fontanelles

Q016. rash pattern in rocky mountain spotted fever

A016. palms and soles, then spreads to trunk; petechial

Q075. describe malrotation

A075. small intestines rotate abnormally in utero, so there is an abnormal fixation posteriorly to the mesentary; it can twist on its vascular supply --&gt; volvulus

Q076. clinical presentation of malrotation

A076. bilious emesis; possibly abdominal distention/shock; + guiac test = bowel ischemia, poor prognostic sign

Q077. tx of malrotation

A077. surgery ASAP

Q078. tx for scd induced priapism

A078. sedation

Q079. disease that scd can mimic if htere is abdominal pain

A079. appendecitis

Q080. when is frontal bossing seen?

A080. beta-thal or some other hemolytic process that requires rapid hematopoesis

Q081. tx for beta-thal

A081. serial transfusion + chelation therapy (desferoxamine) b/c of fe overload

Q082. consequence of fe overload

A082. hemochromatosis

Q083. complications of g6pd deficiency

A083. rbcs are destroyed but there is increased amounts of hb liberated in the process --&gt; hb-uria

Q085. battle's sign

A085. basilar skull fracture that leads to bleeding/bruise behind the ear

Q086. mechanism behind bell's palsy in neonates

A086. forceps deliver... usually resovles

Q087. social milestones at 5 yo

A087. competetive games; understands rules and abides by them

Q088. social milestones at 4 yo

A088. imaginative play

Q089. social milestones at 3 yo

A089. group play; shares

Q090. social milestones at 2 yo

A090. parallel play

Q091. social milestones at 18 mo

A091. plays around other children

Q092. social milestones at 12 mo

A092. comes when called; cooperates with dressing

Q093. social milestones at 9 mo

A093. pat-a-cake

Q094. social milestones at 6 mo

A094. recognizes strangers

Q095. social milestones at 4-5 mo

A095. enjoys observing environment

Q096. social milestones at 3 mo

A096. reaches for familiar objects/ppl

Q097. social milestones at 2 mo

A097. recognizes parent

Q098. social milestones at 1 mo

A098. fixes on face

Q099. developmental dysplasia of the hip

A099. abnormal relationship between head of femur and acetabulum --&gt; instabilility and dislocation of hip joint; develops secondary to lack of contact of acetabulum and femur during intrauterine development

Q1000. Dx:; Alpha-Galactosidase deficiency

A1000. fAbry's Disease; (fAAAAAbry's)

Q1001. Dx:; Hyperinsulinemia, omphalocele, macroglossia, organomegaly

A1001. Beckwith-Wiedemann syndrome; (HOMO)

Q1002. Dx:; short child with hypoglycemia, lactic acidosis, hyperuricemia and hyperlipidemia. Enlarged liver and kidneys and possible seizures

A1002. Von-Gierke's syndrome; (G6P deficiency)

Q1003. Dx:; G-6-P deficiency

A1003. Von-Gierke's syndrome

Q1004. Dx:; Floppy baby with macroglossia and heart failure

A1004. Pompe's syndrome; (Maltase deficiency)

Q1005. Dx:; Maltase deficiency

A1005. Pompe's syndrome

Q1006. Age:; walks alone, speaks 2 words, throws objects

A1006. 12 months old

Q1007. Age:; walks up and down stairs without help and has 2 - 3 word phrases

A1007. 24 months old

Q1008. First step:; Pyloric Stenosis

A1008. Rehydrate and correct Electrolytes

Q1009. When does stranger anxiety start and peak?

A1009. starts: 6 - 8 months; peaks: 12 - 15 months

Q101. tx of developmental dysplasia of hip

A101. pavick harness (keeps hip abducted and flexed), or body casting on older pts

Q1010. Dx:; IVP shows focal renal parenchymal scarring and blunting of calices

A1010. Chronic Pyelonephritis

Q1011. Dx:; Bilateral ptosis, difficulty getting up from chair and muscle weakness

A1011. Myasthenia Gravis

Q1012. Dx:; palpable purpura, glomerulonephritis, decreased complement, arthralgias, HSM, peripheral neuropathy and positive HCV

A1012. Mixed Essential Cryoglobinemia; (hCv = Cryoglobinemia)

Q1013. MC Sx of mixed Sickle cell trait

A1013. painless Hematuria

Q1014. When is the best time to give VZ-Ig to child who was playing with others that had chickenpox

A1014. within 3 - 5 days of exposure; (otherwise tell parents he will get Sx)

Q1015. Dx:; Non-immunized mother brings 2-week-old in b/c of spasms involving whole body and erythematous, tender umbilical cord and poor suckling

A1015. Tetanus

Q1016. Dx:; newborn lung problem due to right-to-left shunt from PDA or foramen ovale

A1016. Persistent Pulmonary HTN of Newborn

Q1017. Deficient enzyme:; FAILURE TO THRIVE, bilateral cataracts, jaundice and hypoglycemia at birth

A1017. Galactose-1-phosphate

Q1018. Tx:; prevents painful episodes of Sickle cell crisis

A1018. Hydroxyurea

Q1019. MC child brain tumor in cerebrum

A1019. Benign Astrocytoma

Q105. idiopathic talipes equinovarus (aka); appearance

A105. congenital clubfoot; medial rotation of tibia, fixed plantar flexion, inversion of foot, forefoot adduction; CANNOT DORSIFLEX (unlike metatarsus adductus)

Q106. tx of clubfoot

A106. bracing; serial casting

Q107. legg-calve-perthes disease

A107. avascular necrosis of femoral head; ischemic bone is eventually resolved and re-ossification occurs; --&gt; limp, pain referred to thigh/knee

Q108. what movements are restricted in legg-calve-perthes disease

A108. abduction; flexion; internal rotation

Q109. tx of legg-calve-perthes disease

A109. bracing; surgery; observation

Q110. complications of legg-calve-perthes disease

A110. collapse of femoral head

Q111. SLIPPED CAPITAL FEMORAL EPIPHYSIS

A111. gradual or acute separation of proximal femoral growth plate; fem head slipps off of femoral neck and rotates inf-post postition

Q112. etiology of slipped capital femoral epiphysis

A112. common during puberty, could be hormonal

Q113. presentation of slipped capital femoral epiphysis

A113. limp, pain in hip and groin, pain referred to knee

Q114. dx of slipped capital femoral epiphysis

A114. plain film frog-leg, lateral position

Q115. tx of slipped capital femoral epiphysis

A115. goal is to prevent further misalignment; pin fixation is done acutely; chronic cases require osteotomy

Q116. Osgood Schlatter disease

A116. inflammation, swelling and tenderness over tibial tuberosity secondary to tendonitis of distal insertion of infrapatellar tendon

Q117. when does Osgood Schlatter disease occur

A117. During growth spurt, in teens

Q118. tx of Osgood schlatter

A118. conservative, supportive management

Q119. complication related to achondrodysplasia

A119. small foramen magnum seen in homozygotes --&gt; brainstem compression

Q120. what is achondrodysplasia

A120. disorder of cartilage calcifications and remodeling

Q121. why do children have an increased risk of fracture

A121. tendons and ligaments are stronger than bones so in kids injuries often lead to fracture when they would only cause sprain in adult

Q122. types of fracture

A122. spiral (twisting forces on tibia during fall); epiphyseal fracture (use salter classification); stress fracture (hairline crack from repeated activity); torus fracture (at metaphysis)

Q124. define upper airway

A124. nose --&gt; carina

Q125. sx of upper airway disease

A125. inspiratory stridor; tachypnea; respiratory distress

Q126. choanal atresia

A126. most proximal abnormality of airway; bony or membranous septum between 1 or both nasal passages and pharynx, preventing airflow through nose; life threatening if b/l (most young infants are obligate nose breathers); can't pass NG tube

Q127. complications of long-term intubation

A127. subglottal stenosis

Q128. laryngeal or tracheomalacia

A128. floppiness that closes off airway

Q129. how to confirm dx of laryngeal or tracheomalacia

A129. bronchoscopy

Q130. ddx for wheezing and respiratory distress

A130. asthma; bronchiolitis; foreign body aspiration; gerd; TE fistula; vascular sling

Q131. pathophysiology of sx in CF

A131. CFTR is abnormal --&gt; altered cl channel; cl stays in cells and Na/water enter the cell to maintain osmotic balance --&gt; viscous secretions

Q132. GI effects in CF

A132. pancreatic insufficiency; bowel obstruction; rectal prolapse; DM; cirrhosis; large bulky smelly stools; later in life stools --&gt; distal obstruction

Q133. Pathognomonic finding in CF

A133. meconium ileus

Q135. other than respiratory infection, what are some other complications of CF

A135. hemoptysis (&gt;500 cc/d = emergency) tx with embolization; spontaneous ptx (1/2 will recur unless sclerosis is performed, but if that is done then transplant is very difficult)

Q137. GI complication from Henoch-Schonlein Purpura

A137. ileal-ileal intussusception

Q138. Sx of Henoch-Schonlein Purpura

A138. non thrombocytopenic palpable purpura in dependent areas; an IgA mediated vasculitis involving GI, skin, joints, and kidneys

Q139. what is Henoch-Schonlein Purpura usually preceded by

A139. URI

Q140. GI sx of Henoch-Schonlein Purpura

A140. spasmodic pain, ileus, vomiting, ugi/lgi bleed

Q141. Tx of Kawasaki disease

A141. iv Ig; ASA

Q142. 3 phases of kawasaki

A142. acute; subacute (aneurysms form); convalescent (resolution over 2-3 mo)

Q143. which bacterial infection can mimic appendicitis

A143. campylobacter; Yesinia

Q144. sx of intussusception

A144. BOWEL OBSTRUCTION; currant jelly stools; lethargy; palpable tubular mass; paucity of gas on XR or evidence of obstruction

Q145. dx of intussusception

A145. barium (or air) enema

Q146. progression of complications of intussusception

A146. impaired venous return; bowel edema; ischemia; necrosis; perforation

Q147. pathophys of intussusception

A147. most are ileocecal, and occur when the ileum invaginates into colon at ileocecal valve

Q148. lead points in intussusception

A148. hypertrophy in Peyer's patches (often p viral infection); Meckel's divertic; intestinal polyp; lymphoma; foreign body; Henoch-Schonlein Purpura

Q149. at what age would you expect pyloric stenosis

A149. 1-3 months

Q150. what medication if given is associated with pyloric stenosis

A150. erythromycin

Q151. dx of pyloric stenosis

A151. u/s, see hypertrophic pyloris; UGI study shows string sign

Q152. embryology behind omphalocele

A152. when midgut loop fails to return to abdominal cavity; see light gray shiny sac protruding from base of umbilical cord

Q153. embryology behind malrotation of midgut

A153. midgut undergoes partial rotation and --&gt; abnormal position of abdominal viscera, can be assoc with volvulus --&gt; compromised blood flow and gangrene

Q154. gastroschisis

A154. weakness in abdominal wall --&gt; herniation of bowel through the rectus muscle, usually to the right of the umbilicus

Q155. clinical features of malrotation

A155. bilious emesis; possible abdominal distension; XR shows gas in stomach, but no gas in intestines

Q156. risk factors for developing GERD in babies

A156. prematurity; esophageal disease; obstructive lung disease; overdistension of stomach from overeating; meds (theophylline)

Q157. dx of GERD

A157. pH probe placement in esophagus or ugi endoscopy; barium swallow to confirm normal anatomy

Q158. tx of GER/GERD

A158. small frequent feedings; keep head up for 20mins p eating; thicken feeds with cereal; metoclopramide (increases gastric motility); h2 blocker or PPI; last resort: nissen fundlopication (fundus of stomach is wrapped around distal esophagus to increase les pressure)

Q159. tx of diarrhea in children

A159. if no serious complications, feed through the diarrhea --&gt; decreased denudement and faster return to normal stooling patterns; don't give antidiarrheals b/c --&gt; toxic megacolon, unless salmonella, Shigella, c diff, or parasites

Q160. when to do w/u for diarrhea in a child

A160. in infant &lt;3mo, do blood culture; 0-12 mo, do stool culture; do blood and stool culture if &gt;5d of enterocolitis or salmonella exposure; any infant with + stool dx looking toxic or + blood culture should be evaluated for pyelo, meningiits, PNEUMONIA, osteo

Q161. when should a patient with diarrhea be admitted to hospital

A161. &gt;5% dehydration and can't rehydrate effectively at home

Q162. constipations and complications

A162. no bowel movement, if after neonatal pd, #1 cause is voluntary withholding; can be caused by pain, on defecation --&gt; fear of defecation - -&gt; further retention; voluntary holding increases distension of rectum, decreases rectal sensation, requiring increased amounts needed to receive urge

Q163. sx of Hirchsprung's

A163. h/o diarrhea, fecal spotting alternating with constipation

Q164. how to tx functional constipation

A164. increase fluid,; decrease junk food,; increase fiber,; increase ingestion of undiluted juice

Q165. pathophys of Hirchsprung's

A165. failure of ganglion cells of myenteric plexus to migrate down colon in utero; therefore distal colon is tonically contracted and there is obstruction; usually limited to rectosigmoid colon

Q166. when to suspect Hirchsprung's

A166. in any infant who doesn't pass meconium, then --&gt; bilious vomiting, abdominal distension, and poor feeding

Q167. tx of Hirchsprung's

A167. diverting colostomy with bowel that contains ganglion cells; aganglionic segment is removed by pulling ganglionic segment through rectum

Q168. sx of Meckel's diverticulum

A168. remnant of vitilline duct w/i 2 inches of ileocecal valve; bleeding from diverticulum; melena, obstruction (from intussusception), diverticulitis

Q170. dx of Meckel's diverticulum

A170. technetium-99 scan p h2 antagonist to locate hemorrhagic cells

Q171. encephalocele

A171. projection of cranial content through bony skull defect in occiput --&gt; severe mental retardation, seizure, movement disorder

Q172. myelomeningocele

A172. protrusion of neural and meningeal tissue

Q173. meningocele

A173. meninges protrude

Q174. spina bifida II is associated with

A174. Chiari II malformation

Q175. complications of spinal bifida

A175. caudal end of cord is tethered to distal spine and can't ascend to adult position --&gt; scoliosis, sphincter dysfxn, LE deformities

Q176. non-communicating hydrocephalus

A176. block in exiting ventricles;; above blockage, ventricles are big

Q177. causes of non-communicating hydrocephalus

A177. secondary to narrowing at 4th ventricle/aqueduct or malformation at posterior fossa; Chiari II malformation; spina bifida occulta

Q178. communicating hydrocephalus

A178. subarachnoid villi are dysfunctional/obliterated

Q179. sx of motor cerebral palsy

A179. fixed lesion in immature brain -&gt; nonprogressive disorder of movement and posture; spasticity is #1 type from injury to motor tracts in brain; pts are hypotonic in early months then later become spastic; CONDITION IS NOT PROGRESSIVE

Q180. sx of extrapyramidal cerebral palsy; etiology

A180. from basal ganglia damage --&gt; choreoathetoid movement, postural ataxia, spasticity; kernicterus, there is usually some sort of brain insult

Q181. what gcs is required for a head ct to be obtained

A181. &lt;12

Q182. #1 cause of ICH in kids

A182. Arteriovenous malformation

Q193. dacryostenosis

A193. congenital nasolacrimal duct obstruction; causes overflow tearing - 6% of neonates; secondary to failure of distal membranous end of nasolacrimal duct to open

Q194. tx of dacryostenosis

A194. probing of nasolacrimal dut at 12-15 mo; although most resolve spontaneously by 1 yo (in 96% of infants)

Q195. ophthalmia neonatorum

A195. conjunctivitis occurring in first month of life; p/w eyelid edema, conjunctival hyperemia and ocular d/c

Q201. complications of chlamydia ophthalmia neonatorum

A201. corneal scarring; PNEUMONIA

Q202. tx of chlamydia ophthalmia neonatorum

A202. oral and topical erythromycin; tx parents; tx with erythromycin despite the risk of developing pyloric stenosis

Q203. tx of gonococcal ophthalmia neonatorum

A203. topical erythromycin; IV cefotaxime; tx parents

Q204. when should steroid containing eye drops not be given in conjunctivitis

A204. if hsv-1 is suspected etiology; can make disease worse

Q205. hordeolum

A205. acute infection of sebacous tarsal glands; staph aureus is usually cause

Q206. tx of hordeolum

A206. warm compresses

Q207. chalazion

A207. area of sterile lipogranulomatous reaction within meibomian glands that can enlarge; can be chronic and recurrent

Q208. is periorbital or orbital cellulitis an emergency

A208. orbital cellulitis

Q209. sx of periorbital cellulitis

A209. skin around eye is indurated, warm, and tender; no eye pain; may have si/sx of sinus infection

Q210. sx of orbital cellulitis

A210. severe pain with eye movement; proptosis; vision changes; decreased ocular mobility

Q211. dx of orbital cellulitis

A211. ct scan

Q212. organisms to cover with orbital cellulitis

A212. strep; H. flu; M. cat

Q213. tx of periorbital cellulitis

A213. iv antibiotics; can --&gt; meningitis, tx aggressively with vanco, PCN, 1st generation cephalosporin

Q214. orgs to tx in periorbital cellulitis

A214. strep; h flu; m cat

Q215. features of fragile x syndrome

A215. hyperactive; mental retardation; large body; long face; prominent jaw and ears; thickened nasal bridge; large testes; +/- autism

Q216. facial features of 47 XYY

A216. long asymmetric ears; increased length: breadth in hands, feet, and cranium

Q217. nutritional deficiencies in goat's milk

A217. decreased vit d, iron, folate, b12

Q218. infectious disease associated with drinking raw cow's milk

A218. brucellosis

Q219. what supplements should moms receive if they are vegan

A219. b12 to prevent buildup of mma

Q220. effects of excess vit D

A220. hypercalcemia; azotemia; poor growth; n/v/d

Q221. vit E deficiency in premies --&gt;

A221. hemolytic anemia

Q222. b1 deficiency --&gt;

A222. beriberi; (neuritis, edema, CHF); hoarseness; anorexia

Q223. b2 deficiency --&gt;

A223. photophobia; cheilosis; glossitis; corneal vascularity

Q224. b3 deficiency --&gt;

A224. pellagra (dermatitis, dementia, diarrhea)

Q225. pathophys of primary hypophosphatemia

A225. defective po4 resorption; no conversion of 25-vit D --&gt; 1,25-vit D in proximal tubules

Q226. lab studies in primary hypophosphatemia; ca; po4; alkaline phosphatase

A226. low/normal; low; high

Q227. clinical findings in primary hypophosphatemia

A227. smooth LE bowing (not angular, as is seen in Ca deficiency); intraglobulin dentin deformities (ca deficiency --&gt; enamel defects); coarse trabecular bone and fraying

Q228. somogyi phenomenon

A228. nocturnal hypoglycemia manifested as night terrors, early am sweating, then later has hyperglycemia, ketonuria, glucosuria (sugars increase because of glucagon release)

Q229. what is an absolute contraindication for DTaP

A229. if first dose --&gt; encephalopathy or encephalitis

Q230. clinical presentation of pb poisoning

A230. emotional lability; abdominal pain; achy bones; intermittent vomiting and constipation

Q231. at what BLL should tx be initiated

A231. &gt;45

Q232. clinical presentation of acute hg poisoning

A232. GI pain; fever; chills; HA; visual changes; cough; cerebral palsy

Q233. clinical presentation of chronic hg poisoning

A233. gingivostomatitis; tremor; neuropsych features

Q237. complications of cerebral palsy

A237. seizure; mental retardation

Q238. differences in etiology for quadriplegia and paraplegia

A238. quadriplegia results from UMN damage; paraplegia results from LMN or spinal cord damage

Q239. most likely organisms Otitis --&gt; pneumnia in CF

A239. staph or pseudomonas

Q241. pathophys of meconium ileus

A241. obstruction begins in utero --&gt; underdeveloped distal lumen

Q242. tx of meconium ileus

A242. surgical emergency; gastrograffin enema

Q243. vitamin a deficiency --&gt;

A243. pseudotumor cerebri (among other things); will see bulging fontanelles, headache, n/v; may be seen as first presenting sign in a patient with CF

Q249. pathophys of volvulus

A249. cecum fails to move to right lower quadrant and never adheres to abdominal wall; mesentary and SMA are tethered to narrow stalk and twist about itself; band of adhesive tissue can extend from cecum to ruq -&gt; duodenal obstruction

Q250. presentation of children with secondary HTN

A250. ha; epistaxis; visual sx; easy fatiguability

Q251. describe how biliary atresia can occur post-natally

A251. from scarrin gan dinflammation of intrahepatic or extrahepatic biliary ducts; etiology is unclear

Q252. describe findings of prenatal biliary atresia

A252. Gallbladder is absent

Q253. sx of congenital toxoplasmosis

A253. chorioretinitis; hydrocephalus; intracranial calcifications

Q254. complement levels in post-strep GN

A254. decreased c3

"Q255. what are ""currant jelly stools"""

A255. bloody stools; indicative of intussusception

Q256. germinal matrix

A256. embryonic tissue present near caudate nucleus, often gets damaged by hypoxia/ischemia; #1 place for intraparenchymal bleed

Q257. progression of intraparenchymal bleed in a newborn

A257. blood can flow into ventricles w/i 3 days of life

Q258. what blood vessels are damaged in shaken baby syndrome

A258. bridging veins

Q259. what blood vessels are involved in SAH

A259. circle of willis

Q260. signs of SVT

A260. Heart rate 220-270; no P waves

Q261. pathophys of strawberry hemangioma

A261. vascular tissue fails to communicate with adjoining tissue; enlarges --&gt; raised tumor

Q262. most common causes for pneumonia in a child &gt; 6 yo

A262. mycoplasm; strep pneumo

Q263. cardiac complications in Marfan syndrome

A263. aortic root dilatation,; aortic dissection; MVP

Q264. most common orgs in peritonsillar abscess

A264. anaerobes; GAS

Q265. physical findings of Patau syndrome

A265. forebrain fails to develop (holoprosenceph); midface developmental abnormalities; abnormal genitalia; severe mental retardation

Q266. which chromosomes are associated with; 1. Edwards; 2. Patau

A266. Edwards = Election age (18); Patau = Puberty age (13)

Q267. porencephaly

A267. cyst/cavity in brain that communicates with ventricles

Q268. causes of euvolemic hyponatremia

A268. SIADH; glucocorticoid deficiency; hypothyroid; water intoxication

Q269. features of neonatal listerosis

A269. respiratory distress at 5 days; meningitis

Q270. clinical presentation of congenital hypothyroid

A270. constipation; jaundice; FAILURE TO THRIVE; enlarged fontanelle; umbilical hernia

Q271. presentation of neuroblastoma

A271. asymptomatic abdominal mass; Horner's; dancing eyes; dancing feet; blueberry muffin lesions; HTN

Q272. Patau's or Edwards syndrome:; microcephaly

A272. Patau

Q273. Patau's or Edwards syndrome; prominent occiput

A273. Edwards

Q277. Patau's or Edwards syndrome; micrognathia

A277. Edwards

Q278. Patau's or Edwards syndrome; low-set malformed ears

A278. Edwards

Q279. Patau's or Edwards syndrome; cleft lip

A279. Patau

Q280. Patau's or Edwards syndrome; congenital heart disease

A280. both

Q281. Patau's or Edwards syndrome; omphalocele

A281. Patau

Q282. Patau's or Edwards syndrome; clenched hands with overlapping fingers

A282. both

Q283. Patau's or Edwards syndrome; rocker bottom feet

A283. Edwards

Q284. Patau's or Edwards syndrome; polydactyly

A284. Patau

Q285. Patau's or Edwards syndrome; polycystic kidney disease

A285. Patau

Q286. Patau's or Edwards syndrome; horseshoe kidney

A286. Edwards

Q287. Patau's or Edwards syndrome; crytorchidism

A287. Patau

Q288. Patau's or Edwards syndrome; agenesis of corpus callosum

A288. Patau

Q289. genetic changes in Prader Willi

A289. paternal deletion, 2 defective maternal chromosomes

Q290. physical appearance of pts with Prader Willi

A290. obese; almond shaped eyes; downturned mouth; small hands and feet

Q291. genetic changes in Angelman's

A291. maternal deletion, 2 defective paternal chromosomes

Q292. physical appearance of Angelman's

A292. large mouth; short stature; tiptoe walk; seizures

Q293. galactosemia

A293. galactose 1-p builds up and accumulates in liver and brain

Q294. complications of galactosemia

A294. increased risk of e coli sepsis; LD; premature ovarian failure

Q295. tx of galactosemia

A295. eliminate galactose containing foods from diet

Q296. complications of PKU during pregnancy

A296. if diet not followed, baby can develop microcephaly, mental retardation, and congenital heart disease

Q297. physical features of homocystinemia

A297. Marfan like appearance; dislocated lens

Q300. protocol for neonates whose mom developed varicella infection just after delivery

A300. if mom develops sx within 2 days of delivery or during end of pregnancy, treat baby with iv Ig and acyclovir. if greater amt of time is lapsed, no tx needed

Q301. consequences of neonatal asphyxia

A301. cerebral edema; irritability; seizure; cardiomegaly; renal and heart failure; DIC; RDS

Q302. what is the apt test

A302. used to differentiae fetal from maternal blood

Q303. which drugs are contraindicated for breastfeeding

A303. lithium; cyclosporin; antineoplastic drugs; ergots; bromocriptine; tetracyclines

Q304. classic finding on XR for necrotizing enterocolitis

A304. pneumanitis intestinalis

Q305. consequences of cold temperature in a premie

A305. increased metabolic rate in order to raise body temp, but their ventilation rate increased as well; but, because of respiratory problems in premies they can't oxygenate enough so lactic acid accumulates --&gt; metabolic acidosis

Q306. which is worse:; ABO incompatibility or Rh incompat

A306. Rh incompatibility

Q307. lab findings in ABO incompat

A307. increased reticulocyte count; weakly + coomb's

Q308. lab findings in Rh incompat

A308. strongly + coombs

Q309. pathophys of transient apnea of the newborn

A309. immature respiratory centers, especially in premies

Q310. how to id the underlying cause of congenital hypothyroid

A310. iodine uptake scan

Q311. late development of clavicle fracture during delivery... when does this occur?

A311. callous formation in anterior shoulder; 1 week

Q312. physical findings of subgaleal hemorrhage

A312. feels like cephalohematoma that crosses midline; can lose 1/3 of

Q313. onset of gonococcal conjunctivitis in a newborn

A313. DOL 2-5

Q315. what effects does surfactant deficiency have on lung compliance and lung volume; cardiac effect

A315. decreases both; R--&gt;L shunt

Q316. what should be given to newborns whose moms are infected with Hep B

A316. Hep B Ig and HBV

Q319. when does transient tachypnea of newborn resolve

A319. DOL #3

Q320. significance of 5th finger polydactyly

A320. in black infants, no consequence; in white infants, can be associated with cardiac abnormalities, must do an echo

Q321. twin twin transfusion

A321. donor twin --&gt; oligohydramnios, anemia, hypovolemia; recipient twin --&gt; polyhydramnios, larger size (20% difference in body weight), hyperviscocity, respiratory distress, hyperbili, hypocalcemia, renal vein thrombosis

Q322. when does serum bilirubin peak

A322. DOL 3-5

Q323. Top Childhood Cancers - What are they

A323. 1. Leukemia (ALL); 2. CNS tumors; 3. Lymphoma; 4. Neuroblastoma

Q324. Neuroblastoma - What is it; Associated with?

A324. Embryonal tumor of; neural crest cell origin; MC cancer in infants; more than 1/2 kids &lt; 2 y/o; associations - neurofibromatosis; Hirchsprung's; n-myc oncogene

Q326. Neuroblastoma - Dx

A326. Abdominal CT; 24-hr urinary catecholamines; assess extent of disease - CXR; bone scan; CBC; LFTs; BUN/Cr; coag panel

Q327. Neuroblastoma - Tx

A327. Excision - localized tumors; chemo includes - cyclophosphamide, doxorubicin; adjunctive radiation - if tumor spread beyond origin; prognosis improved for kids with low-risk dis., &lt; 1 y/o, no N- myc amplification, localized

Q328. Wilms' Tumor - What is it; Associated with?

A328. Embryonal cancer of kidney; MC renal tumor in kids; 2-4 y/o; associated with - family History, Beckwith-Wiedemann syndrome, WAGR, neurofibromatosis

Q329. Wilms' Tumor - History/PE

A329. Painless, palpable abdominal mass; does not cross midline; n/v; fever; weight loss; hematuria; HTN

Q330. Wilms' Tumor - Dx

A330. Abdominal CT or US - intrarenal mass; check for metas - CXR, chest CT; CBC; LFTs; BUN/Cr

Q331. Wilms' Tumor - Tx

A331. Transabdominal nephrectomy; postsurgical chemo - vincristine, dactinomycin; flank irradiation (for some); prognosis good - depends on staging &amp; histo

Q332. RDS - What is it

A332. MCC of respiratory failure in premies; surfactant deficiency =&gt; increased surface tension (poor lung compliance); and alveolar collapse (atelectasis); surfactant made by T2 pneumocytes, mainly ~35th week; dipalmitoyl phosphatidylcholine; risk factors - maternal diabetes, males, 2nd born of twins

Q333. RDS - History/PE

A333. Presents in 48-72 hrs of life; RR &gt; 60/min; intercostal retractions; expiratory grunting; nasal flaring; cyanosis; progressive hypoxemia

Q334. RDS - Dx

A334. ABGs; CBC (to rule out infection); BC (to rule out infection); CXR - bilateral diffuse atelectasis, causing ground-glass appearance with visible air bronchograms; lecithin:sphingomyelin ratio &lt; 2

Q335. RDS - Tx

A335. CPAP or intubation and mechanical ventilation; artificial surfactant to prevent - mom gets corticosteroids; monitor fetal lung maturity (L:S ratio)

Q336. RDS - Complications

A336. Persistent PDA; bronchopulmonary dysplasia; retinopathy of prematurity; intraventricular hemorrhage; NEC

Q337. Intussusception - What is it

A337. MCC of bowel obstruction in 1st 2 yrs. of life; males &gt; females; usually proximal to ileocecal valve; cause - idiopathic, in older kids - mass or intestinal abnormality triggers the telescoping:; adenovirus or rotavirus; parasites; CF; celiac disease; polyps; intestinal lymphoma; Meckel's diverticulum; Henoch-Schonlein purpura

Q345. Child Abuse - What is it

A345. Neglect; physical abuse; sexual abuse; emotional abuse; suspect - if History doesn't match physical findings, if there was a delay in getting medical care

Q346. Child Abuse - History/PE

A346. Infants may have apnea, seizures, FAILURE TO THRIVE Exam findings include; cutaneous - ecchymoses of varying ages, patterned injuries; skeletal - spiral fractures of femur and humerus in kids &lt; 3 = abuse unless prove else; epiphyseal/metaphyseal injuries - can happen in infants from pulling/twisting limbs; rib injuries &lt; 2 y/o; sexual abuse - STDs or genital trauma

Q347. Child Abuse - Dx

A347. Rule out conditions that mimic skeletal survey &amp; bone scan; can show fractures in various stages of healing; if sexual abuse suspected - test for gonorrhea, chlamydia and HIV; to rule out shaken baby syndrome - check for retinal hemorrhages; CT for subdural hemorrhages; MRI for white matter changes

Q349. Epiglottitis - What is it

A349. Serious, rapidly progressive infection of supraglottic; before immunization - from H influ type B; now - Streptococcus, nontypable H flu, viral agents

Q351. Epiglottitis - Dx

A351. Clinical; DON'T EXAMINE THROAT unless anesthesiologist present; definitive Dx - direct fiberoptic visual of cherry-red, swollen epiglottis &amp; arytenoids; lateral XR - thumbprint sign

Q352. Epiglottitis - Tx

A352. Emergency; call anesthesiologist; transfer patient to OR; endotrach intubation or trach; IV Antibiotics - ceftriaxone or cefuroxime

Q353. Croup (Laryngotracheobronchitis) - What is it

A353. Inflammation of larynx and upper airway, mainly subglottic space =&gt; narrowing of airway; kids 3 mos. - 3 yrs. MCC - parainfluenza virus 1, also - PIV-2, PIV-3, RSV, rubeola, influenza, adenovirus, Mycoplasma pneumonia

Q354. Croup (Laryngotracheobronchitis) - History/PE

A354. Prodrome - URI Sxs 1-7 days; stridor - worse by agitation; fever - low grade; hoarseness; barking cough

Q356. Croup (Laryngotracheobronchitis) - Tx

A356. Mild - cool mist; moderate - oral corticosteroids; severe - (respiratory distress at rest), admit, nebulized racemic epinephrine

Q357. Bronchiolitis - What is it

A357. Acute inflammation of smallest airways; acute viral bronchiolitis; MCC - RSV; infants &amp; kids &lt; 2 y/o; can progress to respiratory failure; risk for severe RSV - &lt; 6 mos. old, premies, heart or lung dis., immunodeficiency

Q358. Bronchiolitis - History/PE

A358. History - low-grade fever, rhinorrhea, cough, apnea - young infants; PE - tachypnea, wheezing, hyperresonance to percussion

Q359. Bronchiolitis - Dx

A359. CXR - hyperinflation of lungs; interstitial infiltrates; atelectasis; ELISA of nasal washings, for RSV - high sensitivity &amp; specificity

Q360. Bronchiolitis - Tx

A360. Mild - outpatient, fluids, nebulizers, O2 if needed; admit if - marked respiratory distress, O2 saturation &lt; 95%, toxic appearance, dehydration/poor oral feeding, premie (&lt; 34 wks), &lt; 3 mos. old, underlying cardiopulmonary dis., unreliable parents; inpatients - contact isolation, hydration, O2, ribavirin; RSV prophylaxis - RespiGam or Synagis, high-risk patients in winter

Q361. Otitis Media - What is it; Risk Factors

A361. Middle ear infection; MCC - #1 - S. pneumoniae, #2 - H. Flu, #3 - Moraxella catarrhalis; kids predisposed - eusta tube; risk factors - viral URIs, trisomy 21, CF, immunodeficiency, smoke exposure, day-care attendance, bottle feeding, cleft palate, prior otitis media

Q362. Otitis Media - History/PE

A362. Fever; ear tugging; hearing loss; irritability; Erythema; bulging; decreased mobility of tympanic membrane; loss of light reflex and bony landmarks; tympanic membrane may be perforated

Q363. Otitis Media - Dx

A363. Clinical

Q364. Otitis Media - Tx

A364. Amoxicillin - 10 days; Tx failure after 3 days - switch to amoxicillin-clavulanic acid, ceftriaxone or cefuroxime

Q365. Otitis Media - Complications

A365. Mastoiditis; meningitis; hearing loss; cholesteatoma; tympanosclerosis; chronic suppurative OM

Q366. Kawasaki Disease; (Mucocutaneous Lymph Node Syndrome) - What is it

A366. Multisystem acute vasculitis; usually kids &lt; 5 y/o, especially Asian; at risk for coronary artery; aneurysms =&gt; MI

Q368. Kawasaki Disease; (Mucocutaneous Lymph Node Syndrome) - Dx

A368. Clinical; thrombocytosis - wk 2 or 3; increased ESR

Q369. Kawasaki Disease; (Mucocutaneous Lymph Node Syndrome) - Tx

A369. High-dose aspirin; IVIG - to prevent aneurysms; corticosteroids are contraindicated (they may increased aneurysms)

Q370. FAILURE TO THRIVE - What is it

A370. Persistent weight; below 3rd to 5th percentile or falling off growth curve; organic - medical condition; nonorganic - psychosocial; nonorganic is MC; risk factors - chronic illness, poverty, low maternal age, chaotic environment, genetic dis. (CF), inborn errors of metabolism, HIV

Q371. FAILURE TO THRIVE - History/PE

A371. Low weight for age and height; minimal weight gain or weight loss; plot on growth chart; check for signs of systemic dis. diet History; observe caregiver-child interaction

Q372. FAILURE TO THRIVE - Dx

A372. Calorie count; CBC; electrolytes; Cr; albumin; total protein; sweat chloride test; UA/UC; stool culture; O&amp;P; assess bone age

Q373. FAILURE TO THRIVE - Tx

A373. Tx depends on cause; supplement nutrition if breastfeeding inadequate; admit if - neglect, severe malnourishment

Q374. Atrial Septal Defect (ASD) - What is it

A374. Opening in atrial septum lets blood flow bet. atria; L to R shunting due to lower R pressure; blood flow to lungs increased

Q375. Atrial Septal Defect (ASD) - History/PE

A375. History - usually presents in late childhood or early adult onset &amp; severity depends on size; large defect - tire easy (DOE); freq. respiratory infections; FAILURE TO THRIVE =&gt; CHF =&gt; cyanosis; PE - RV heave, wide, fixed split S2, systolic ejection murmur; upper left sternal border

Q377. Atrial Septal Defect (ASD) - Tx

A377. Small defects may close spontaneously (no Tx needed); Antibiotics prophylaxis before dental procedures; surgical closure - infants with CHF, patients &gt; 2:1 pulmonary to systemic blood flow; correct early to prevent - arrhythmia, RV dysfunction, Eisenmenger's syndrome

Q378. Ventric Septal Defect (VSD) - What is it

A378. MC congenital heart defect; More common in patients with - Apert's syndrome; Cri-du-chat; Trisomies 13 &amp; 18

Q379. Ventric Septal Defect (VSD) - History/PE

A379. History - Sxs depends on degree of shunting; small defect usually asymptomatic at birth; large defect - CHF, freq respiratory infections, FAILURE TO THRIVE; PE - pansystolic murmur - lower left sternal border, loud pulmonic S2, in severe defects - systolic thrill, cardiomegaly, crackles

Q381. Ventricular Septal Defect (VSD) - Tx

A381. Most small defects close spontaneously (no Tx needed); Antibiotics prophylaxis before dental or pulmonary procedures; surgical closure - correct early to prevent - Eisenmenger's syndrome, et al; Tx CHF &amp; respiratory infections

Q382. Patent Ductus Arteriosus (PDA); What is it

A382. Failure of DA to close =&gt; L-to-R shunt (aorta to pulmonary artery); risk factors - high altitude (low O2), 1st trimester rubella in mom, premies, females (more common)

Q383. Patent Ductus Arteriosus (PDA); History/PE

A383. History - typically asymptomatic (small PDA); slowed growth; recurrent lower respiratory infections; lower extremity clubbing; CHF Sxs; PE - wide pulse pressure, continuous machine murmur, loud S2, bounding peripheral pulses

Q384. Patent Ductus Arteriosus (PDA); Dx

A384. Small - often no signs of cardiomegaly; Large - echocardiogram - LA &amp; LV enlarged; EKG - LVH; CXR - cardiomegaly; color-flow Doppler- diagnostic

Q385. Patent Ductus Arteriosus (PDA); Tx

A385. Indomethacin; (unless need PDA for survival); if indomethacin fails or; child &gt; 6-8 mos. old - surgical closure preferred

Q386. Coarctation of the Aorta - What is it

A386. Narrowing proximal or distal to DA =&gt; increased flow above, decreased flow below coarctation; more common in males; Turner's, 25% have bicuspid aortic valve

Q387. Coarctation of the Aorta - History/PE

A387. History - often presents in childhood with asymptomatic HTN, headache, syncope, epistaxis, DOE, claudication; PE - systolic BP higher in upper extremities, may be greater in right arm; femoral pulses weak or delayed; late systolic murmur in left axilla; apical impulse forceful; advanced cases - well-developed upper body, lower extremities wasting

Q390. Transposition of the; Great Arteries - What is it

A390. Pulmonary &amp; systemic circulation in parallel; aorta connected to RV; pulmonary artery connected to LV; incompatible with life unless septal defect or PDA; risk factors - babies of DM moms, Apert's syn, Down's, cri- du-chat, Trisomies 13 &amp; 18

Q391. Transposition of the; Great Arteries - History/PE

A391. Critically ill; cyanosis immediately after birth; tachypnea; progressive respiratory failure; CHF (some patients)

Q393. Transposition of the; Great Arteries - Tx

A393. Prostaglandin E1 (PGE1) - to keep PDA open; balloon atrial septostomy; arterial or atrial switch op

Q394. Tetralogy of Fallot - What is it

A394. VSD; pulmonary stenosis; RVH; overriding aorta; right-to-left shunting =&gt; early cyanosis; risk factors - Down's, cri-du-chat, Trisomies 13 &amp; 18

Q395. Tetralogy of Fallot - History/PE

A395. History - cyanosis, dyspnea, fatigability; profound cyanosis = tet spell; squatting for relief; hypoxemia =&gt; FAILURE TO THRIVE, mental status changes; PE - SEM at left sternal border, RV lift, single S2, CHF signs possible

Q396. Tetralogy of Fallot - Dx

A396. Echocardiography; catheterization; CXR - boot-shaped heart, decreased pulmonary vascular markings; EKG - right-axis deviation; RVH

Q397. Tetralogy of Fallot - Tx

A397. PGE1 - keep reopen PDA; for cyanotic spells - O2, propranolol, knee-chest position, fluids, morphine; balloon atrial septostomy before surgical correction

Q398. Cerebral Palsy (CP) - What is it

A398. Group of nonprogressive, nonhereditary neurological disorder; disorder in movement and posture; MC movement disorder in kids; MCC unknown - prenatal, perinatal and post insults; risk factors - prematurity, perinatal asphyxia, intrauterine growth retardation, early infection or trauma, brain malformation, neonatal cerebral hemorrhage

Q399. Cerebral Palsy (CP) - What are the categories

A399. Spastic (pyramidal) - spastic paresis of any limb, 75% of cases, MENTAL RETARDATION up to 90%,; athetoid - extrapyramidal, b. ganglia, uncontrollable jerking, writhing, worse with stress, disappears during sleep; ataxic - cerebellum, hard to coordinate movement, wide- based gait; mixed

Q400. Cerebral Palsy (CP) - History/PE

A400. May be associated with - seizure disorder, behavioral disorder; hearing or vision impaired; learning disabilities; speech deficits; hyperreflexia; Babinski; increased tone/contractures; weakness; underdevelopment; toe walking; scissor gait; hip dislocations; scoliosis

Q401. Cerebral Palsy (CP) - Dx

A401. Clinical; rule out metabolic disorder, cerebellar dysgenesis, spinocerebellar degeneration; EKG (if seizures)

Q402. Cerebral Palsy (CP) - Tx

A402. Special education; physical therapy; braces; surgical relief of contractures; for spasticity - diazepam, dantrolene, baclofen; for severe contractures - baclofen pumps, posterior rhizotomy

Q403. Febrile Seizures - What is it

A403. In kids 6 mos. - 6 y/o; no evidence of intracranial infection or other cause; risk factors - rapid rise in temp, History in close relative

Q434. Rubella - Cause; Characteristics

A434. Rubella virus; prodrome - asymptomatic or; tender, generalized; lymphadenopathy; erythematous, tender,; maculopapular rash; slight fever; polyarthritis in adolescents

Q435. Rubella - Complications

A435. Encephalitis; thrombocytopenia; congenital infections associated; with congenital anomalies

Q408. Neonatal Jaundice - What is it; What are the types; What is kernicterus

A408. Increased serum bilirubin from increased production or decreased excretion; conjugated - always pathologic; unconjugated - patholog or physiolog; physiologic jaundice - not present until 72 hrs after birth, bilirubin peaks &lt; 15 mg/dL, resolves by 1 week in term, resolves by 2 weeks in premies; pathologic jaundice - present in 1st 24 hrs of life, bilirubin rises to &gt; 15 mg/dL, persists past 1 week in term, persists past 2 weeks in premies; kernicterus - unconjug hyperbilirubinemia, bilirubin deposits in pons, basal ganglia, cerebellum, irreversible, can be fatal; risk factors - premies, asphyxia, sepsis

Q409. Neonatal Jaundice - History/PE

A409. History - child breastfed or formula? intrauterine drugs; family History of - hemoglobinopathies, enzyme deficiency, RBC defects; Sxs - abdominal distention, delayed passage of meconium, light-colored stools, dark urine, low Apgar scores, weight loss, vomiting; kernicterus - lethargy, poor feeding, high-pitched cry, hypertonicity, seizures; jaundice may be cephalopedal; check for signs of - infection, congenital malformations, cephalohematomas, bruising, pallor, petechiae, hepatomegaly

Q410. Neonatal Jaundice - Dx

A410. CBC; periph blood smear; blood type mom and baby; Coombs' test; bilirubin levels; direct hyperbilirubinemia - LFTs; bile acids; Blood culture; sweat test; tests for aminoacidopathies &amp; a1-antitrypsin deficiency; Sepsis w/u and ICU - jaundice, febrile, hypotensive and/or tachypneic

Q411. Neonatal Jaundice - Tx

A411. Tx underlying cause; unconjugated - severe - exchange transfusion; if mild - phototherapy, start earlier for premies (start at 10- 15 mg/dL)

Q412. Down Syndrome - What is it

A412. Trisomy 21, MC chromosome disorder; #2 cause of congenital MENTAL RETARDATION; risk increased with mom's age, but 80% of kids are born to women &lt; 35 y/o; flat facial profile; prominent epicanthal folds; simian crease; decreased levels of AFP; brushfield spots; duodenal atresia - double bubble on US/XR; congenital heart disease - septum primum-type ASD due to endocardial cushion defect; Alzheimer's &gt; 35 y/o; increased risk of ALL; meiotic nondisjunction

Q413. Edwards' Syndrome - What is it

A413. Trisomy 18 (election age=18); severe MENTAL RETARDATION; rocker bottom feet; low-set ears; micrognathia; congenital heart disease; clenched hands; prominent occiput; death usually &lt; 1 y/o

Q414. Patau's Syndrome - What is it

A414. Trisomy 13 (puberty=13); severe MENTAL RETARDATION; microphthalmia; microcephaly; cleft lip/palate; abnormal forebrain; polydactyly; congenital heart disease; death usually &lt; 1 y/o

Q415. Klinefelter's Syndrome - What is it

A415. XXY (male); inactivated X (Barr body); 1 of MCC of male hypogonadism testicular atrophy; eunuchoid body shape, long extremities; gynecomastia; female hair distribution

Q416. Turner' Syndrome - What is it

A416. XO (No Barr body); short stature; ovarian dysgenesis; webbing of neck; cystic hygroma; coarctaton of aorta; MCC of primary amenorrhea

Q418. Phenylketonuria (PKU) - What is it

A418. Phenylalanine =&gt; tyrosine; In PKU, decreased phenylalanine hydroxylase or tetrahydrobiopterin cofactor; tyrosine becomes essential; phenylalanine builds up =&gt; excess phenylketones, phenylketones - phenylacetate, phenyllactate, phenylpyruvate; MENTAL RETARDATION; fair skin; eczema; musty body odor; screened for at birth; Tx - decreased phenylalanine (in Nutrasweet) and increased tyrosine in diet

Q419. Fabry's Disease - (lysosomal storage disease); What is it

A419. X-linked recessive; defect of a-galactosidase A; ceramide trihexoside accumulates; renal failure

Q420. Krabbe's Disease - (lysosomal storage disease); What is it

A420. AR (autosomal recessive); deficiency of B-galactosidase; galactocerebroside accums; in the brain; optic atrophy; spasticity; early death

Q422. Niemann-Pick Disease - (lysosomal storage disease); What is it

A422. AR; deficiency of sphingomyelinase; sphingomyelin and cholesterol; build up in reticuloendo-; thelial &amp; parenchymal cells; cherry-red spot on macula; death by age 3

Q423. Tay-Sachs Disease - (lysosomal storage disease); What is it

A423. AR; absence of hexosaminidase A; GM2 ganglioside accums; cherry-red spot on macula; death by age 3; MC lysosomal storage disease; that causes MENTAL RETARDATION

Q424. Metachromatic leukodystrophy - (lysosomal storage disease); What is it

A424. AR; deficiency of arylsulfatase A; sulfatide accums in brain,; kidney, liver, periph n.

Q425. Hurler's Syndrome - (lysosomal storage disease); What is it

A425. AR; deficiency of a-L-iduronidase; corneal clouding; MENTAL RETARDATION

Q426. Hunter's Syndrome - (lysosomal storage disease); What is it

A426. X-linked recessive; deficiency of iduronate sulfatase; mild form of Hurler's; no corneal clouding; mild MENTAL RETARDATION

Q427. Fragile X Syndrome - What is it

A427. X-linked; 3rd MCC of MENTAL RETARDATION; FMR1 gene affected; anticipation; triple repeat of CGG; autism; large testes, jaw, ears; floppy/prolapsed mitral valve

Q428. APGAR Score - Chart

A428. Appearance - skin color 0 = blue all over, 1 = blue at extremities, 2 = normal; Pulse - 0 = none, 1 = &lt; 100, 2 = &gt; 100; Grimace - reflex irritability, 0 = none, 1 = grimace, feeble cry, 2 = sneeze, cough, pull away; Activity - muscle tone, 0 = none, 1 = some flexion, 2 = active movement; Respiration - 0 = none, 1 = weak or irreg, 2 = strong

Q429. APGAR Score - What do total scores mean

A429. Score at 1 min. after birth; then at 5 min. score 8-10 - good cardiopulm adaptation; score 4-7 - possible need for resus; observe; stimulate; possible need for vent support; score 0-3 - resus immed

Q431. Erythema Infectiosum - (Fifth Disease); Complications

A431. Arthritis; hemolytic anemia - aplastic crisis in sickle cell; encephalopathy; associated c hydrops fetalis

Q432. Measles - Cause; Characteristics

A432. Paramyxovirus; prodrome - low-grade fever; conjunctivitis; coryza; cough; Koplik's spots - buccal mucosa after 1-2 days; maculopap rash from ears down

Q433. Measles - Complications

A433. Giant cell pneumonia; otitis media; laryngotracheitis; rare - subac scleros. panencephalitis

Q436. Roseola Infantum - Cause; Characteristics

A436. HHV-6; prodrome - acute onset of high fever; no other Sxs for 3-4 days; maculopap rash as fever breaks; starts on trunk; =&gt; face and extremities; often lasts &lt; 24 hrs

Q437. Roseola Infantum - Complications

A437. Rapid fever onset =&gt;; febrile seizures

Q438. Rotavirus - Characteristics

A438. Primary cause of diarrhea; in kids &lt; 2; fever and vomiting; then diarrhea; upper respiratory Sxs; lasts &lt; 1 wk; infection confirmed by Elisa; oral rehydration sufficient

Q440. Varicella - Complications

A440. In immunocompromised kids - progressive varicella with; meningoencephalitis; and hepatitis; congenital infections =&gt;; congenital anomalies

Q442. Varicella Zoster - Complications

A442. Encephalopathy; aseptic meningitis; pneumonitis; TTP; Guillain-Barre; cellulitis; arthritis

Q443. Hand-Foot-and-Mouth Disease - Cause; Characteristics

A443. Coxsackie A; prodrome - fever; anorexia; oral pain; rash - oral ulcers; maculopap vesicular rash on; hands, feet, buttocks

Q444. Hand-Foot-and-Mouth Disease - Complications

A444. None; (self-limited)

Q445. Tracheoesophageal Fistula - What is it

A445. Tract between trachea &amp; esoph; associated with esoph atresia; &amp; VACTERL anomalies - vertebral; anal; cardiac; tracheal; esophagus; renal; limb

Q446. Tracheoesophageal Fistula - Caused by; Presentation

A446. Polyhydramnios in utero; increased oral secretion; inability to feed; gagging; respiratory distress

Q447. Tracheoesophageal Fistula - Dx

A447. CXR after NGT; air in GI tract; bronchoscopy - to confirm

Q448. Tracheoesophageal Fistula - Tx

A448. Surgical repair

Q449. Congenital Diaph Hernia - What is it

A449. GI tract segment protrudes; thru diaph into thorax; 90% are post. lt. Bochdalek

Q450. Congenital Diaph Hernia - Presentation

A450. Respiratory distress from - pulmonary hypoplasia; pulmonary HTN; sunken abdomen; bowel sounds over; lt. hemithorax

Q451. Congenital Diaph Hernia - Dx

A451. US in utero; postnatal CXR - to confirm

Q452. Congenital Diaph Hernia - Tx

A452. Hi-freq ventilation or; extracorporeal membrane; oxygenation (ECMO); (to manage pulmonary HTN); surgical repair

Q453. Gastroschisis - What is it

A453. Herniation of intestine; thru abdominal wall; next to umbilicus; (usually on right); with no sac

Q454. Gastroschisis - Caused by; Associated with

A454. Polyhydramnios in utero; often premie; associated with - GI stenoses; GI atresia

Q455. Gastroschisis - Tx

A455. Surgical emergency; single-stage closure possible; in only 10%

Q456. Omphalocele - What is it

A456. Herniation of abdominal viscera; thru abdominal wall; at umbilicus; into sac covered by; peritoneum and amniotic memb

Q457. Omphalocele - Caused by; Associated with

A457. Polyhydramnios in utero; often premie; associated with; other GI &amp; cardiac defects

Q458. Omphalocele - Tx

A458. C-section - to prevent sac rupture; if sac intact - postpone surg correction; until patient fully resuscitated; keep sac covered/stable; with petroleum &amp; gauze; intermittent NG suction - to prevent abdominal distention

Q459. Duodenal Atresia - What is it

A459. Complete or partial failure; of duodenal lumen; to recanalize; during gestational wks 8-10

Q460. Duodenal Atresia - Presentation; Caused by; Associated with

A460. Bilious emesis within hrs; after 1st feeding; polyhydramnios in utero; associated with - Down's; other cardiac/GI anomalies:; annular pancreas; malrotation; imperforate anus

Q461. Duodenal Atresia - Dx

A461. Double-bubble sign on XR; (prox to site of atresia)

Q462. Duodenal Atresia - Tx

A462. Surgical repair

Q463. Meckel's Diverticulum - What is it

A463. MC congenital GI tract anomaly; vestigial remnant; of omphalomesenteric duct; rule of 2's - 2x's as many males; 2 ft. from ileocecal valve; 2% of people affected; 2 types of mucosa - gastric; pancreatic

Q464. Meckel's Diverticulum - Presentation

A464. MC presentation - painless rectal bleeding; painful diverticulitis; intest. obstruction from - intussusception or volvulus

Q465. Meckel's Diverticulum - Dx

A465. Meckel's scan - for ectopic gastric mucosa; uses IV technetium; pertechnetate

Q467. Hirschsprung's Disease - (Congenital Aganglionic Megacolon); What is it

A467. Absence of autonomic; innervation of bowel wall; inadeq relaxation; and peristalsis; =&gt; intest. obstruction

Q468. Hirschsprung's Disease - (Congenital Aganglionic Megacolon); Presentation

A468. Abdominal distention; bilious vomiting; fail to pass meconium in; 1st 24 hrs of life

Q469. Hirschsprung's Disease - (Congenital Aganglionic Megacolon); Dx

A469. Barium enema - dilated prox segment; narrowed distal segment; rectal Bx - to confirm

Q470. Hirschsprung's Disease - (Congenital Aganglionic Megacolon); Tx

A470. Colostomy prior to; corrective surgery

Q471. Hypospadias - What is it

A471. Abnormal urethral opening; on ventral surface of penis; due to incomplete dev of; distal urethra

Q472. Hypospadias - Presentation; Associated with

A472. Chordee; associated with - hernias; cryptorchidism

Q473. Hypospadias - Tx

A473. Circumcision contraindicated; surgical repair uses; preputial tissue

Q629. how long can undescended testis remain undescended before you need to bring them down?

A629. one year

Q481. DiGeorge Syndrome; (Thymic aplasia) - What is it

A481. CATCH-22; Cardiac defects; Abnormal facies; Thymic hypoplasia; Cleft palate; Hypocalcemia; 22 - microdeletions in chrom22; tetany in first days of life

Q482. DiGeorge Syndrome; (Thymic aplasia) - Dx

A482. Absolute lymphocyte count,; mitogen stimulation response; and delayed hypersensitivity; skin testing

Q483. DiGeorge Syndrome; (Thymic aplasia) - Tx

A483. BMT - if severe; IVIG; thymus transplant

Q484. Ataxia-Telangiectasia - What is it

A484. DNA repair defect; oculocutaneous telangiectasias; progressive cerebellar ataxia

Q485. Ataxia-Telangiectasia; Tx

A485. No effective Tx for CNS abnorm; neuro deterioration progresses; death by 30 y/o

Q486. Severe Combined; Immunodeficiency (SCID) - What is it

A486. Severe lack of B &amp; T cells; frequent severe bacterial infections; chronic Candidiasis; opportunistic organisms

Q487. Severe Combined; Immunodeficiency (SCID) - Tx

A487. BMT or stem cell transplant; IVIG; PCP prophylaxis until BMT; gene therapy may be; future option

Q488. Wiskott-Aldrich Syndrome - What is it

A488. X-linked recessive; T &amp; B cell dysfunction; thrombocytopenia; small-sized platelets; eczema; high IgE; high IgA; low IgM; bloody diarrhea; bleeding gums; prolonged nosebleeds

Q489. Wiskott-Aldrich Syndrome - Tx

A489. Protective helmet; IVIG; aggressive Antibiotics for infections; HLA-identical BMT; if no BMT - rarely survive to adulthood

Q490. Chronic Granulomatous Disease-; (CGD); What is it

A490. X-linked or AR; deficient superoxide production by PMNs &amp; M0s; usual sites of infection - skin, lungs (pneumonia), lymph nodes, liver (abscesses, hepatitis), bones (osteomyelitis); swollen collections of infected tissue obstruct intestines (IBD) and urinary tract (UTIs)

Q491. Chronic Granulomatous Disease-; (CGD); Dx

A491. Absolute neutrophil count and adhesion assays - chemotaxic, phagocytic, bactericidal; diagnostic - negative nitroblue tetrazolium dye reduction test (NBT)

Q492. Chronic Granulomatous Disease-; (CGD); Tx

A492. Daily TMP-SMX; judicious Antibiotics use during infections; IFN-g - can decreased incidence of serious infection

Q493. Chediak-Higashi - What is it

A493. AR; giant lysosomal granules dev. in neutrophils lysosomes can't fuse with phagosomes =&gt; ingested bact. can't be lysed; oculocutaneous albinism; neuropathy; neutropenia

Q494. C1 Esterase (Inhib) Deficiency - (Hereditary Angioedema); What is it

A494. C1 inhibitor - acute phase protein inhibits proteinases of: complement pathway, clotting pathway, kinin generator pathway, fibrinolytic pathway; deficiency =&gt; hereditary angioedema; AD; can affect - hands &amp; feet - local edema, bowel - extreme abdominal pain, mouth/airway - life-threatening edema; usually lasts 3 days; can be precipitated by trauma, virus; aggravated by stress

Q495. C1 Esterase (Inhib) Deficiency - (Hereditary Angioedema); Dx

A495. Total hemolytic C' assay- CH50; if defect in one component - no CH50 reduction, then detect which component

Q496. C1 Esterase (Inhib) Deficiency - (Hereditary Angioedema); Tx

A496. Daily prophylactic danazol; purified C1 esterase and FFP - prior to surgery

Q497. Terminal Complement Deficiency; (C5-C9) - What is it

A497. AR; recurrent N. meningitides &amp; disseminated gonorrhea infections; rarely - systemic lupus

Q499. Hyper IgM Syndrome - What is it

A499. MC - deficiency in CD40 ligand in T Helper =&gt; can't class switch; normal or high IgM; low IgG, IgA, IgE; XL recessive (most common), AR (others); severe upper &amp; lower respiratory; diarrhea - Cryptosporidium

Q500. Hyper IgM Syndrome - Tx

A500. IVIG; PCP prophylaxis - TMP-SMX

Q501. What is the most common cause of bowel obstruction in the first two years of life?

A501. Intussusception

Q502. What is the classic triad for intussusception

A502. Abdominal pain, vomiting, blood per rectum

Q503. Red currant-jelly stools are indicative of what?

A503. Intussusception

Q504. What is diagnostic for intussusception

A504. Air-contrast barium enema

Q505. The string sign on a barium study indicates what?

A505. Pyloric stenosis

Q506. What type of acid/base abnormality can occur with persistent emesis, such as that seen with pyloric stenosis?

A506. Hypochloremic, hypokalemic metabolic alkalosis

Q507. What type of immunodeficiency presents after six months of age with recurrent sinopulmonary, GI, and urinary tract infections with encapsulated organisms (H. influenzae, Streptococcus pneumoniae, Neisseria meningitidis)?

A507. B-cell deficiencies

Q508. What immunodeficiency is seen in boys only and presents with frequent pseudomonas infections?

A508. X-linked agammaglobulinemia (Bruton's)

Q509. With what immunodeficiency will there be a drop in immunoglobulin levels to the 20s and 30s, increased pyogenic upper and lower respiratory diseases, and increased risk of lymphoma and autoimmune disease?

A509. Common variable immunodeficiency

Q510. What is the most common immunodeficiency

A510. IgA deficiency

Q511. What does CATCH-22 stand for in DiGeorge syndrome?

A511. Congenital heart disease,; Abnormal facies,; Thymic aplasia,; Cleft palate,; Hypocalcemia,; 22q deletion

Q512. With what immunodeficiency syndrome will patients have increased infections with fungi and PCP?

A512. Thymic aplasia; (DiGeorge syndrome)

Q513. With what immunodeficiency will patients present with cerebellar ataxia and increased incidence of non-Hodgkin's lymphoma and gastric carcinoma?

A513. Ataxia-telangiectasia

Q530. What are the acute-phase manifestations of Kawasaki disease? Hint: Crash and Burn

A530. Conjunctivitis,; Rash,; Adenopathy,; Strawberry tongue,; Hands and feet (red, swollen, flacky skin),; AND Burn=fever &gt;40 for 5 days

Q531. What is the treatment for Kawasaki disease?

A531. High-dose aspirin and IVIG

Q532. What treatment is contraindicated with Kawasaki disease?

A532. Corticosteroids; (may increase aneurysm formation)

Q533. Untreated patients with Kawasaki disease are at risk for what complications?

A533. Coronary artery aneurysms and MI

Q534. What disease is defined as an acute inflammatory illness of the small airways that primarily affects infants and children under 2?

A534. Bronchiolitis; (RSV most common)

Q535. What disease is characterized by the classic barking cough (usually at night)?

A535. Croup; (Laryngotracheobronchitis)

Q536. What is the steeple sign?

A536. Subglottic narrowing of the airway,; commonly seen with Croup

Q537. With what disease will patients age 3-7 present with muffled voice and drooling?

A537. Epiglottitis

Q538. The thumbprint sign on lateral film is characteristic of what?

A538. Epiglottitis

Q539. What three organisms commonly cause otitis media?

A539. S.pneumoniae,; H.influenzae,; Moraxella catarrhalis

Q540. What disease is characterized by a slapped-cheek rash that is worse with fever and sun exposure?

A540. Erythema infectiosum,; caused by Parvovirus B19

Q541. What disease has an acute onset of high fever followed by a maculopapular rash that appears as the fever breaks?

A541. Roseola infantum (HSV-6)

"Q542. An abdominal radiograph showing the ""double-bubble"" sign indicates what?"

A542. Duodenal atresia

Q543. What is the most common cause of respiratory distress in infants?

A543. Respiratory distress syndrome

Q544. Ground-glass appearance and air bronchograms on CXR in an infant indicate what disease?

A544. Respiratory distress syndrome

Q545. What are the five Ts and 1P of cyanotic heart disease?

A545. 1. Truncus arteriosus; 2. Transposition of the great arteries; 3. Tricuspid atresia; 4. Tetralogy of Fallot; 5. Total anomalous pulmonary venous return 6. Pulmonary atresia

Q546. What congenital heart defect presents with a harsh holosystolic murmur heard best at the lower left sternal border?

A546. VSD

Q547. What congenital heart defect presents with a wide and fixed split S2 and a systolic ejection murmur heard best at the upper left sternal border?

A547. ASD

Q548. What drug is given to close a patent ductus arteriosus?

A548. Indomethacin

Q551. household and sex contacts of kids/teens with hepatitis A should receive what?

A551. immunoglobulin to prevent hep A

Q552. intracranial trauma without obvious external findings in baby; think?

A552. shaken baby syndrome

Q553. person who gets hives after bee sting. what NSiM?

A553. give Epi-pen at all times (bc had systemic reaction) as well as give prescription for diphenydramine

Q555. urinalysis of &gt; 1.015 makes what highly unlikely?

A555. diabetes insipidus

Q556. primary vs secondary enuresis

A556. secondary = bedwetting after 3 or more months of dry nights;; is usually due to psych factors or changes

Q557. therapy for child over 6 with enuresis?

A557. consider conditioning therapy with bedwetting alarm (works in 30-60%)

Q558. any UTI in baby with vomiting and dehydration, think? caused by? tx with ? what follow up tests?

A558. pyelonephritis;; e coli; ceftriaxone; need to follow up ANY UTI in girl under 5 and any boy, with renal US and VCUG to rule out v-u reflux

Q559. rash post GAS pharyngitis, think?

A559. Scarlet fever!

Q560. boggy mass on scalp? tx?

A560. most likely kerion = inflammatory form of tinea capitis (Trich. tonsurans or Microsporum canis). tx = 8-12 weeks oral griseofulvin and shampooing with selenium sulfide

Q561. oral, feet, and mouth ulcers, think?

A561. hand foot and mouth disease!; coxsackie A16

Q562. ehrlichosis contracted how?

A562. by tick bite

Q563. adults with e. infectiousum may also have what sx?

A563. ARTHROPATHY

Q564. parvovirus B19 ( slapped cheek/ E. infectiousum )in hemolytic anemias may cause?

A564. aplastic anemia

Q565. high and persistent fever that is not very responsive to normal doses of paracetamol; with red cracked lips and red mucous membranes?

A565. kawasaki's disease

Q566. swelling of one eyelid + proptosis, limited ocular movements?

A566. orbital cellulitis;; if just swelling of eyelid think periorbital cellulitis

Q567. 1 y/o black baby solely breastfed may develop?

A567. ricketts due to VitD def in breast milk and poor skin conversion of vit D

Q568. torsion of testicular appendix vs torsion of testicle?

A568. Torsion of testicle: in kids over 12; testicular appendix: in kids 2-11

Q569. acute pain in scrotum of sexually active teen, think?

A569. epididymitis

Q570. m/c pathogens of otitis media?

A570. S pneum,; H influ,; M catarrhalis

Q630. MC type of Ambiguous genitalia

A630. Congenital Adrenal Hyperplasia; (21-hydroxylase deficiency is the MC of the CAH)

Q571. what constitutes increased risk for post-traumatic seizure? what % get? tx if these?

A571. depressed cranial fracture; intracranial hemorrhage; cerebral contusion; or unconscious &gt; 24 hrs. 75% of these will seizure; so must tx with anticonvulsants prophylactically

Q572. main pathogen causing croup? epiglotitis?

A572. parainfluenza;; s pneum,; aureus or GBS; H influenzae in non vaccinated

Q573. cow milk feeding causes?

A573. iron def anemia!

Q574. hirsutism, deepened voice, acne, clitoromegaly, think?

A574. CAH

Q575. can you have allergic rhinitis that presents with rhinitis with NO seasonal variation?

A575. YES = perenial allergic rhinitis ie exposure to house dust, pet danders that are there year round

Q576. bilateral PURULENT discharge with fever, cough, sinus tenderness?

A576. sinusitis

Q577. 3 common pathogens in human bites?

A577. eikenella corrodens; peptostreptococcus; alpha streptococcus

Q578. during Antibiotics tx , diaper rash due to?

A578. candida dermatitis!

Q579. m/c cause of round pneumonia?

A579. strep pneumonia

Q580. HUS is most common in what age?

A580. under 2 y/o's!

Q581. live vaccines?

A581. MMR; varicella

Q582. Guillan Barre has what CSF finding?

A582. increased protein;; mild mononuclear pleocytosis (&lt;10 cells/ml)

Q583. when do you treat lead poisoning with chelation?

A583. only if levels over 45 ug/mL !; otherwise just treat by removing source, ie investigating house for lead content, etc

Q584. name 4 advantages of OPV? 1 of IPV?

A584. OPV:; costs less; increased mucuosal immunity; fewer shots; herd immunity through secondary transmission; still recommended for global polio eradication; IPV: less paralytic polio

Q585. m/c cause of HTN in kids?

A585. secondary (not essential) Renal causes!! ie infection, PCKD, glomerulonephritis, vascular anomalies, tumors

Q586. 2 tx's for Increased ICP

A586. intubation with hyperventilation; Mannitol to induce osmotic shift

Q587. after what type of wounds/pts do you give Td, vs Td + Tet Immune Globulin, vs DTaP?

A587. DTaP: give only in kids under age 7; Td only: if patient has had &lt; 3 immunizations or status unknown AND: minor wound w/o contamination; Td + TIG: if patient has had &lt; 3 immunizations or status unknown AND contaminated wounds

Q588. only 2 exceptions to IC kids NOT receiving varicella vacc (b/c it is live)

A588. ALL kid in remission with lymphocyte count over 700;; HIV infected kid in CDC class I =CD4 lymphocytes&gt;25%. All other immunodeficiencies, don't give!!

Q589. live viruses not to be given to ?

A589. IC pts (see exceptions above) or pregnant women

Q590. m/c organism causing ANY otitis externa?

A590. P. aeruginosa

Q591. causes of intoeing?

A591. adducted great toe,; metatarsus adductus,; medial tibila torson,; femoral anteversion

Q592. m/c back deformity in kids?

A592. idiopathic scoliosis

Q593. tx of b pertussis?

A593. e-mycin

Q594. hacking cough + absolute lymphocytosis, + thick, clear n/p mucus?

A594. think b pertussis

Q595. TCA poisoning?

A595. anticholinergic sx,; mental status changes,; arrhythmias

Q596. otherwise normal 7 y/o with hypospadias. What next step in tx?

A596. renal US to check for enlarged prostatic utricle which predisposes to UTI's

Q597. snuffles =?; acral rash?

A597. mucocutaneous lesions producing persistent, purulent, often bloody nasal discharge!!; acral = on extremities ie hands and feet = syphilis

Q598. febrile seizures do/don't increase risk for epilepsy?

A598. DO! but by only 7%

Q599. painful swollen fingers and toes for 2 days?

A599. think sickle cell hand-foot syndrome

Q600. most common manifestation of sickle cell disease?

A600. acute PAINFUL episodes ie in legs, arms, in younger kids;; head, chest, back, abdomen in older kids

Q601. shortness from GH deficiency usually shows by how many yrs?

A601. 3 yrs old!

Q602. chest pain in young 12 y/o girl athlete during exercise?

A602. asthma!; give inhaled albuterol

Q603. 5 days of high fever; bilateral bulbar non-exudative conjunctivitis, rash, hand and foot edema--&gt;desquamation, strawberry tongue and mouth erythema, cervical lad?

A603. kawasaki's!

Q604. m/c cyanotic heart disease in newborn?

A604. Transposition!

Q622. Newborn eye:; lens opacity

A622. Cataracts; (possible Galactosemia)

Q623. Newborn eye:; Aniridia and hemihypertrophy (one side of body larger then other)

A623. Wilms Tumor

Q624. Dx:; blue baby pinks up on crying; confirmatory test?

A624. Choanal Atresia; test: Catheter doesn't pass thru nose

Q625. MC abdominal mass in newborn

A625. Polycyctic Kidney disease

Q626. when should an umbilical hernia close?

A626. by 5 months

Q627. which has a sac--Omphalocele or Gastroschisis?

A627. Omphalocele; ("O" is like a closed sac)

Q628. what should be avoided in a patient with Epispadias or Hypospadias?

A628. do NOT circumsize

"Q691. Dx:; High fever, tonsil bulges, uvula deviates to non-involved side, ""hot potato voice""; Bug?; Tx? (2 together)"

A691. Peritonsillar Abscess; bug: Group A Strep; Tx:; 1. Drain abscess; 2. Penicillin (Erythromycin if allergy)

Q693. Dx:; High fever, sore throat and ulcers scattered on the soft palate, tonsils and pharynx, plus similar lesions on palms and soles

A693. Hand-Foot-and-Mouth disease; (Coxsackievirus)

Q694. Dx:; child with sudden onset respiratory distress, wheezing; hyperinflated lung on one side and tracheal deviation to the opposite side

A694. Foreign body aspiration

Q695. Dx:; 4 year-old that presents with a Barking cough with Inspiratory stridor; Dx Test?; Tx? (2 together); Tx if advanced or an emergency?

A695. Croup (Parainfluenza virus); Test: P/A neck film: STEEPLE SIGN; Tx:; (1) Humidified air; (2) Steroids beneficial; Emergency: Racemic Epinepherine

Q696. Dx:; Dysphagia, Drooling, muffled voice, leaning forward to maximize air entry; Bug?; Dx test?; Tx? (2)

A696. Epiglottitis (EMERGENCY); Cause: H. flu type b (HIB); Dx Test: Lateral Neck: THUMBPRINT SIGN; Tx:; (1) Secure Airway: CHERRY RED EPIGLOTTIS (seen w/ intubation); (2) antibiotics

Q697. Dx:; patient less then 2yo had a cold, then Wheezing, rales, tachypnea, Accessory muscles are used during respiration; MCC?; Dx test? (2); Prevention (for those predisposed)?

A697. Bronchiolitis; MCC: RSV; Dx test:; (1) Rapid assay from Nasal secretions; (2) CXR (Hyperinflation); Prevention: Palivizumab - Monoclonal antibodies

Q698. MCC of death in 1 - 12 month-old children

A698. SIDS

Q699. What are the (3) main Right-to-Left shunt developmental Heart disorders?

A699. 1. Tetralogy of Fallot;; 2. Transposition of the great vessels;; 3. Tricuspid Atresia

Q700. What are the (3) main Left-to-Right shunt developmental Heart disorders?

A700. 1. VSD; 2. ASD; 3. PDA

"Q701. What are the (3) main ""Mixed"" developmental Heart disorders?"

A701. 1. Truncus Arteriosus; 2. TAPVR; 3. Hypoplastic LH

Q702. Dx:; Continuous murmur w/ wide pulse pressure heard in the upper left sternal border; Tx?

A702. PDA; Tx: Indomethicin

Q703. Dx:; Apical Click followed by a late-systolic murmur

A703. Mitral Prolapse

Q704. Dx:; Holosystolic murmur best heard at the Apex

A704. Mitral Regurgitation

Q705. Dx:; Mid-diastolic murmur followed by opening Snap

A705. Mitral Stenosis

Q706. Dx:; Systolic ejection murmur best heard at the left upper sternal border and a mid-diastolic murmur at the lower left sternal border; Fixed S2

A706. ASD

Q707. Dx:; Loud harsh pansystolic murmur heard in the Lower left sternal border; What can it lead to?

A707. VSD (MC heart defect); Leads to: Eisenmenger's syndrome; (VSD shunt reverses right to left when the PVR exceeds the SVR)

Q708. Dx:; VSD shunt reverses right to left when the PVR exceeds the SVR

A708. Eisenmenger's syndrome

Q720. Dx:; watery to bloody Diarrhea, N/V, fever, possible neurologic Sx, caused by ingesting poultry or raw eggs; Tx?

A720. Salmonella; Tx: Hydration only (no Antibiotics - they prolong Disease)

Q721. Tx for diarrhea from Shigella

A721. TMP-SMX

Q722. Tx of diarrhea from Campylobacter

A722. Erythromycin

Q723. Tx for diarrhea from C. Difficile; (2 together)

A723. 1. Metronidazole; 2. Vancomycin

Q724. First test if suspecting Hirshprung's Disease; Diagnostic test?

A724. First: Barium Enema; Dx test: Colon Biposy

Q725. Dx:; chronic problem in infant consisting of cough, vomiting and apnea; Dx test?

A725. GERD; Dx test in infant: pH probe

Q726. Dx:; Bilious vomiting and Double-bubble sign on abdominal X- ray; What is it assoc with?

A726. Duodenal Atresia; assoc with: Down's Syndrome

Q727. Dx:; Nonbilious, Projectile vomiting; RUQ pain in infant; First Dx test?

A727. Pyloric Stenosis; first test: Ultrasound

Q728. what acid/base problem is seen with pyloric stenosis?

A728. Hypochloremic, Hypokalemic Metabolic Alkalosis

Q737. What is the MCC of Nephrotic syndrome in Pediatrics?; Tx?; Main possible Complication?

A737. Minimal Change Disease (MCD); Tx: Steroids; Complication: Spontaneous Peritonitis

Q739. MCC of Limping and hip problem in age 0 - 3 yo; Dx test? (2 together); Tx?

A739. Congenital Hip Dysplasia; Dx test:; 1. Ordalani or Barlows procedure (Hear a click on PE); 2. Ultrasound; Tx: Harness or Casting

Q740. MCC of limping in Age: 4 - 8 years (Begins as Painless limping leading to limping and pain); Dx test?

A740. Legg-Perthes (Avascular necrosis of femoral head); Dx test: Hip x-ray

Q741. MCC of hip problems in a &gt; 11 year-old obese adolescent, that may complain of Knee pain; Dx Test? (2 together)

A741. Slipped Capital Femoral Epiphysis (Will externally rotate leg in an antalgic position); Dx Test:; (1) Hip x-ray; (2) Thyroid, LH and FSH (for obesity; think deficient gonads)

Q743. MC adolescent bone tumor. It occurs in a Metaphyseal area on the Distal Femur, Proximal Tibia or Proximal Humerus; what is seen on x-ray?; Who has an increased risk?

A743. Osteosarcoma; x-ray: Codman triangle (bony sclerosis); Inc risk: patients with Bilateral Retinoblastoma

"Q745. Dx:; Bone tumor causing ""onion-skinning"" of mid-shaft of femur; what chromosomal problem?"

A745. Ewing's Sarcoma; chromosome: t(11;22)

Q819. Dx:; A child presents with a rash that began on his neck and then spread to his trunk. On exam, you notice both new and old vesicles in the same area

A819. Varicella (Chicken pox)

Q746. Dx:; A child presents with recurrent episodes of non-pitting edema on the skin (including swelling of eyelids and lips), GI tract and respiratory system and a family history of the same problem; C4 complement is low; What is the deficiency?; Genetics?; Tx?

A746. Hereditary Angioedema; Deficiency: C1 Esterase Inhibitor; Genetics: Autosomal Dominant; Tx: Epinepherine and airway (like for anaphylaxis)

Q747. Dx:; a male infant between the ages of 6 - 12 months presents with recurrent lung and sinus infections, especially pneumococcus and H.influenzae; All Immunoglobulins are Low and B-cells are low or absent; Tx?

A747. Bruton's Agammaglobinemia (X-linked recessive); Tx: replace IgG

Q748. Dx:; A newborn child presents with hypocalcemia, truncus arteriosus, Fishmouth, tetany and Micrognathia; What is deficient?

A748. DiGeorge syndrome; Deficiency: T-cells (from hypoplasia of 3rd and 4th pharyngeal pouches; thymus)

Q749. Dx:; Chorioretinitis, gingivitis, Granulomas in GI tract, Lupus syndromes; Recurrent Infections of Pneumonia, Abscesses, Lymphadenitis, Osteomyelitis, Bacterimia / Fungemia, and superficial skin infections (cellulitis and impetigo); Dx Test?; Tx?; Cure?

A749. Chronic Granulomatous Disease (Deficiency in a step of NADPH oxidase w/i cells; X-linked recessive); Dx Test: Nitroblue Tetrazolium dye reduction (NBT); Tx: Long-term TMP-SMX (or Dicloxacillin); Cure: Bone Marrow Transplant

Q750. What are the symptoms of Wiskott-Aldrich syndrome?; (5)*

A750. MENTAL RETARDATION. TExT:; Ig(M) deficient;; Recurrent respiratory infections;; Thrombocytopenia;; Eczema;; (x-linked recessive);; T and B-cell deficiency

Q757. What does a baby have a risk of contracting when born to a mother with SLE?

A757. Congenital Heart block

Q758. Immunizations:; 1. less then 1 month; 2. at 2 months; 3. at 4 months; 4. at 6 months

A758. 1. less then 1 month: (B) Hep-B; 2. at 2 months: (DHIP) DTP, HiB, IPV, Pneumo; 3. at 4 months: (BDHIP) Hep-B, DTP, HiB, IPV, Pneumo; 4. at 6 months: (DHP) DTP, HiB, Pneumo

Q759. Immunizations:; 1. at 15 months; 2. at 18 months; 3. at 4 - 6 years

A759. 1. at 15 months: (HeMP) HiB, MMR, Pneumo; 2. at 18 months: (BDIAV) Hep-B, DTP, IPV, Hep-A, Varicella; 3. at 4 - 6 years: (DIM) DTP, IPV, MMR

Q761. Dx:; Child presents with Fever (for at least 5 days) + 4 of 5 of following: Bilateral, non-purulent Conjunctivitis, Mucous membrane changes, hot red Tongue or red, cracked lips, Peripheral extremity changes, Edema or peeling of the skin on fingers, Rash (leather-like) and cervical lymph nodes; Dx test?; Tx? (2 together)

A761. Kawasaki Disease (Mucocutaneous LN syndrome); Dx test: Echocardiogram (rule-out possible Coronary Vasculitis or Anneurysm); Tx:; 1. Aspirin (not high dose); 2. IV-IgG therapy

Q762. Dx:; Child had a viral or Group A Strep infection, then later gets a rash from the Waist downward, Elbows downward and Face, in addition to Sx of Vomiting, Abdominal pain, Ileus, and possible Upper and Lower GI bleeding; Immuno problem?; Tx? (2 together)

A762. Henoch-Schoenlein purpura; Immuno problem: IgA-mediated Vasculitis; Tx:; (1) Supportive (recovery in 4 - 6 weeks); (2) Corticosteroids for GI Sx

Q763. MCC of iron deficiency in newborn?; in child 9 - 24 months?

A763. newborn: Low Birth Weight; 9 - 24 months: Diet

Q764. What can drinking cows milk before 1 year-old cause in an baby?

A764. Iron Deficiency Anemia from gastric bleeding

Q765. Dx;; Young child with pallor, scleral icterus and mild splenomegaly. CBC shows reticulocytosis, anemia with an Increased Mean Corpuscular Hemoglobin (MCH) concentration and indirect hyperbilirubinemia; Genetics?; What can Dx lead to?; Next Dx Test?; Tx? (2)

A765. Hereditary Spherocytosis; genetics: Autosomal Dominant; leads to: Aplastic Anemia (especially after parvovirus B-19); next Dx Test: OSMOTIC FRAGILITY TEST; Tx:; 1. Folic acid supplementation; 2. Splenectomy (in patient &gt; 6yo)

Q766. What is the first test if you suspect you have a Sickle cell patient?; How is Sickle cell Diagnosis confirmed?; Ongoing Tx? (3)

A766. First test: CBC w/ peripheral smear; Dx test: Hemoglobin Electrophoresis; Tx:; 1. Prophylactic Penicillin; 2. Folate supplementation; 3. Hydration

Q767. Dx:; a patient w/ sickle cell begins to have pain in various areas of the body. Tx? (3 together)

A767. Sickle cell crisis; Tx:; 1. Oxygen; 2. IV fluids; 3. Analgesics (Narcotics)

Q768. Dx:; A 2 month-old develops Dactylitis (hand and foot swelling), vaso-occlusion causing Splenic and bone infarcts, multiple Infections

A768. Sickle cell anemia

Q769. Dx:; A child has Sx of Petechiae (that do not blanch), but patient appears well and possible bleeding of mucous membranes; very low platelet count; Bone marrow is normal (or an increase in Megakaryocytes); Tx? (3 together)

A769. Idiopathic Thrombocytopenia (ITP); Tx:; (1) IV-IG; (2) Steroids (must do bone marrow first); (3) Anti-D globulin

Q770. What (2) coagulopathies present with:; Normal PT,; Normal PTT,; High Bleeding time; What else in blood work differentiates the two?

A770. ITP and TTP; Both have low platelets, but TTP also has LOW RBC

Q809. At what age are solids introduced in an infant?; what is the average weight gain for a term infant per day?; when should they be 2x the original weight?; 3x?

A809. Solid foods: 4 - 6 months; Ave weight gain: 20-30g/day; 2x: 4 - 5 months; 3x: 12 months

Q810. What is the only vaccination that can cause a fever 7 days after it is given?

A810. MMR

Q811. when should a child's car seat start facing front?

A811. &gt; 1 yo + &gt; 10 kg

Q815. A child with sickle cell presents to your office with a fever. Next step?

A815. Immediately admit to hospital

Q816. A child with sickle cell presents with a stroke. Tx?

A816. Exchange Transfusion

Q817. What is the first sign of Nephrotic syndrome in a child?

A817. Pitting Edema of the extremities

Q818. Dx:; A child is involved in a MVA and presents with a HR that goes from 110 to 56 and BP that is 156/96, with irregular respirations.

A818. Increased ICP

Q826. in the steps of fluid management in children, when is the only time normal (0.9%) saline is given?; why is it given?; what is given the other times?

A826. 0.9% = Initial 20cc/kg Bolus (continue boluses until patient urinates, except for DKA...for them monitor vitals and neuro status); Given to Restore Intravascular Volume; 0.45% (1/2 normal) saline is used to correct dehydration and for maintenance fluids in a child

Q827. When is the earliest time to initiate testing for Type 2 DM in children?; How frequent do you test?; What is the best test?

A827. Age of Initiation: 10-yo or at onset of Puberty (whichever is first); Frequency: every 2 years; Test: Fasting Plasma Glucose

Q833. On physical examination, the infant is afebrile with stable vital signs. She can lift her head to 90 degrees, her eyes follow past the midline, she laughs, regards her own hand and has slight awareness of her mother. How old is she?

A833. 4 months

Q834. Dx:; A 5-year-old girl presents with a 3-day history of fever, dyspnea, and intermittent joint pain. She has a history of sore throat about 1 month ago. On physical examination, her temperature is 39.6 C (103.2 F), blood pressure is 94/60 mm Hg, pulse is 114/min, and respirations are 22/min. Her knees and elbow joints are swollen and tender to palpation. There is a grade III/VI diastolic murmur best heard at the apex. Multiple fine, pink macules are noted on her trunk. These macules are blanching in the middle. Tx?

A834. Rheumatic Fever; Tx: Penicillin

Q835. A 4-week-old infant presents with tachycardia, tachypnea, and poor weight gain. His arterial blood gas shows a pH of 7.34, a PaCO2 of 41 mm Hg, and a PaO2 of 74 mm Hg. A chest radiograph shows cardiomegaly. Echocardiography reveals a structurally normal heart, left ventricular dilatation, a left ventricular ejection fraction of 20%, and mild mitral and tricuspid regurgitation. What IV med is the best first step in management?

A835. Furosemide; (patient has CHF)

Q836. Dx:; A term male infant is found to be cyanotic shortly after birth and requires endotracheal intubation. On physical examination, his blood pressure is 68/34 mm Hg (equal in all four extremities), pulse is 180/min, and respirations are 32/min. His precordium is dynamic, has a grade III systolic murmur, and a single S2. Chest radiography shows a normal heart size and increased pulmonary vascular markings. An arterial blood gas on an FiO2 of 100% shows pH 7.34; PaCO2, 47 mm Hg; PaO2, 46 mm Hg

A836. Total Anomalous Pulmonary Venous Return; (characterized by the pulmonary veins forming a confluence behind the left atrium, and draining into the right atrium. Complete mixing takes place in the right atrium, with a right- to-left shunt through the foramen ovale to the left side of the heart)

Q837. Dx:; A 1-month-old baby boy has bloody diarrhea. No infectious agent is identified, but the baby is found to be profoundly thrombocytopenic. The baby is also noted to have a skin rash, and a dermatologist diagnoses eczema. By three months of age, the baby begins to develop recurrent respiratory infections. If this child survives until adolescence, he is at particularly high risk of developing what?

A837. Wiskott-Aldrich syndrome; can lead to: Lymphoma

Q847. A patient presents with painful ulcers on his penis. You think it is HSV. What is the Dx test?

A847. Tzanck preparation

Q848. what is the Dx test for RSV?

A848. Nasopharyngeal aspirate

Q849. Dx:; a 9-yo boy presents with increasing clumsiness, change in speech, wide-based unsteady gait, nystagmus and no DTRs. what heart problem can it lead to in the future?

A849. Friedrich Ataxia; leads to:; Hypertrophic Cardiomyopathy

Q850. Dx:; a 20-mo girl presents with loss of developmental milestones, repetitive movements and acquired microcephalopathy

A850. Rett syndrome

Q851. What is the confirmatory test for DMD?

A851. Muscle biopsy

"Q852. What type of exercise induces ""exercise-induced"" asthma?"

A852. High intensity, continuous, prolonged exercise; (ex: a 5-K run)

Q853. what deformities of the head are associated with congenital toxoplasmosis?; (2)

A853. 1. Microcephaly; 2. Hydrocephalus

Q854. What vitamin is most likely to cause a pseudotumor cerebri?

A854. Vitamin A

Q864. A 2-mo is admitted with constipation and an abdominal mass, fever, feeding poorly and vomiting for the past 2 days. A barium enema is ordered and shows Hirschprung's Disease. what is the most important next step?

A864. IV fluid hydration and IV antibiotics; (reestablish IV volume and prevent sepsis)

Q865. What (2) electrolyte disorders accompany Rhabdomyolysis?

A865. 1. Hyperphosphatemia (leading to...); 2. Hypocalcemia

Q866. What are the minor criteria of Rheumatic fever?; (5)

A866. FAILS:; 1. Fever; 2. Arthralgia; 3. Increased CRP or ESR; 4. Long PR interval; 5. Strep culture + or ASO+

Q867. what type of acid/base disorder do you see with a 2 day history of diarrhea?

A867. Hyperchloremic Normal Anion gap Acidosis; (loss of bicarb in the stool stimulates renal tubular reabsorption of chloride ions)

Q868. What (3) malignancies are associated with EBV?

A868. 1. Burkitt Lymphoma; 2. Hodgkin Disease; 3. Nasopharyngeal CA

Q869. What is the difference in AST/ALT initially between Hepatitis B and Hepatitis C?

A869. Hep B: AST/ALT in Thousands; Hep C: minimal or no elevation of AST/ALT

Q870. What electrolyte abnormality is commonly seen in a newborn macrosomic infant from a mother with diabetes?

A870. Hypocalcemia; (and hypomagnesemia)

Q905. what one sign is most suggestive of Primary TB?

A905. Hilar lymphadenopathy

"Q906. Dx:; a 15-yo boy presents with pain in the right knee, History of easy bruising and chronic fatigue. PE shows mild HSM. X-ray of femur shows ""Erlenmeyer Flask deformity."" Labs show anemia and thrombocytopenia. Bone marrow shows ""wrinkled paper"" appearance. Deficient enzyme?"

A906. Gaucher's Disease Type 1 (seen in adolescent ashkenazi jewish population); enzyme: Acid Beta-Glucosidase

Q996. Dx:; infant with Meningitis from Gonorrhea (meningococcemia) begins to get large purpuric lesions on flank

A996. Waterhouse-Friderichsen syndrome; (acute adrenal failure/hemorrhage from meningococcemia)

Q997. COD:; Meningococcemia

A997. Adrenal Failure/Hemorrhage; (from advancing to Waterhouse-Friderichsen syndrome)

Q998. Tx:; Homocystinuria

A998. Vitamin B6

Q999. Dx:; Beta-Galactosidase deficiency

A999. KraBBe's Disease

Hole in the wall b/w both atria

ASD

What are the muscles a child uses to assist in breathing?

Abdominal

Which muscle group protecting major organs is weaker in a pediatric patient than in an adult patient?

Abdominal

Best initial test for pyloric stenosis

Abdominal ultrasound

Why is a systolic ejection murmur heard w/ Truncus arteriosus?

Abnormal valve leaflets

Other than pneumonia, what can aspiration in the setting of esophageal atresia + TEF lead to?

Abscess formation from anaerobic proliferation

A component of an effective EMSC system is

Access to care

Associate the *Bird's beak sign* w/ its associated condition

Achalasia

102 degrees

Active cooling should be discontinued at...

Condition characterized by the acute loss of fluids and electrolytes in the stool due to underlying pathologic process

Acute diarrhea

Second most common cause of infant death worldwide

Acute diarrhea

Treatment of Herpes

Acyclovir + supportive care

An infant who "wasn't acting right" has a heart rate of 230 beats/min. He is awake but lethargic, and his skin is pale with delayed capillary refill. He has SVT, and vagal maneuvers do not convert the rhythm. Your partner has established vascular access. What is the initial treatment of choice for this child

Adenosine

0.1-0.2mg/kg

Adenosine dose

Endemic, year-round cause of viral infectious diarrhea

Adenovirus

With which age group is the paramedic most likely to encounter behavioral emergencies associated with alcohol or other drug use, eating disorders, depression, suicide and suicide gestures, sexually transmitted diseases, pregnancy, and sexual assault

Adolescent

When is surgical intervention indicated for umbilical hernia?

After age 4

When do umbilical hernias spontaneously close by?

Age 3 (90%)

Age group affected by Legg-Calvé-Perthes disease

Ages 2-8

Reasoning for psychiatric counseling in CAH patients

Aid w/ gender identity issues

X-ray findings associated w/ diaphragmatic hernia

Air fluid levels

The most common causes of cardiac arrest in infants and young children.

Airway and respiratory problems

Etiology of bronchitis

Airway inflammation of bacterial/viral origin

What is ALWAYS the wrong answer in the management of diarrhea and gastroenteritis?

Antidiarrheal compounds (loperamide)

Treatment of Diphtheria

Antitoxin

Which component of the pediatric assessment triangle assesses mental status, muscle tone, and body position?

Appearance

Yes

Are peds more at risk for a pneumothorax?

Yes

Are peds more prone to gastric distension?

Fetal circulation includes two umbilical ____ in the cord carrying ____ blood flowing from the fetus to the placenta

Arteries, Deoxygenated

A child with a gastric feeding tube develops respiratory distress. For what complication should you assess?

Aspiration

What is the purpose of a "doorway assessment?"

Assess the patient's urgency

1 year old

Asthma does not effect kids under...

Presentation of ASD

Asymptomatic + *fixed wide splitting of S2*

2/3 the width of the arm

BP cuff size

Larger

BSA to body weight ratio is...

Reflex characterized by toe extension

Babinski reflex

When does shoulder dystocia occur?

Baby's anterior shoulder gets stuck behind mother's pubic bone after delivery of the fetal head

Where do triscuspid and pulmonary lesions radiate to?

Back

Epiglottis

Bacterial infection of the upper airway that effects children 3-7 years old and has no seasonal preference.

Infection of the subglottic region. presents with copious thick secretions.

Bacterial tracheitis. What is it and how does it present?

Unable to be woken up. Does not recognize parents. Unable to console.

Bad signs of septic shock (3)

Management of recurrent stenosis in surgically treated coarctation of the aorta

Balloon dilation

Most accurate test that is both diagnostic and therapeutic in a patient w/ intussussception

Barium enema

Method of diagnosing small left colon syndrome

Barium study

How is congenital hip dysplasia diagnosed?

Barlow and Ortolani maneuvers

Most fatal type of skull fracture in the newborn

Basilar

Linear fracture to the floor of the skull

Basilar skull fracture is a

Most appropriate next step in management of: a 10.5 lb infant born to a mother with Type 1 DM, presenting w/ shaking, holosystolic murmur over precordium, right arm adducted/internally rotated, & ↑ bilirubin?

Blood sugar level

What is the most common type of diaphragmatic hernia?

Bochdalek diaphragmatic hernia

Effect of hypophosphatemia on bone

Bones weaken because adequate bone mineralization cannot take place

How does diaphragmatic hernia impair ventilation?

Bowel contents move into the chest

PE findings associated w/ diaphragmatic hernia

Bowel sounds in the chest

Injuries secondary to births with traction in the event of shoulder dystocia

Brachial palsy

In a distressed patient this may mean hypoxia and is an ominous sing of impending cardiac arrest.

Bradycardia

Mechanism of action of phototherapy in treatment of hyperbilirubinemia

Breaks down bilirubin to excretable components

Respiration in APGAR measures

Breathing

A 6 month old child has respiratory distress. What is the most likely cause of this complaint in this age group

Bronchiolitis

CXR findings w/ Whooping cough

Butterfly pattern

Definitive diagnostic test for VSD

Cardiac catheterization

Swelling of the soft tissues of the scalp that *does* cross suture lines

Caput succedaneum

Blanch the nail bed, base of the thumb, or sole of the foot. 2 seconds or less

Capillary refill

under 6 years

Capillary refill is considered a reliable sign of perfusion for ages...

Spinal cord roots affected by Duchenne Erb paralysis

C5-C6

Any of the several autosomal recessive diseases resulting in errors in steroidogenesis

CAH

Commonly tested syndrome associated w/ choanal atresia

CHARGE syndrome

Most diagnostic test to confirm choanal atresia

CT scan

What diagnostic test is rarely done in the setting of Croup and is always the wrong answer to the most appropriate next step?

CXR

What diagnostic test is rarely done in the setting of Epiglottitis?

CXR

Condition in which infant is born with a buccopharyngeal membrane between the nostrils and pharyngeal space that prevents breathing during feeding causing respiratory distress

Choanal atresia

Tetralogy of Fallot is associated w/ what?

Chromosome 22 deletions

Which component of the Pediatric Assessment Triangle is most affected by hypovolemia?

Circulation

W/ hypospadias, what is contraindicated?

Circumcision

What is done after suctioning the mouth and nose of a newborn?

Clamp and cut umbilical cord

A frantic mother calls you to check her son's central venous catheter because it is leaking. On arrival, you note that the catheter is cracked and leaking. The child's condition is stable. What action should you take?

Clamp the line

What is the steeple sign?

Classic CXR finding w/ RSV representing narrowing air column in trachea

Most common newborn fracture as a result of shoulder dystocia

Clavicular fracture

Best initial diagnostic test for Fifth disease/Erythema infectiosum

Clinical diagnosis

Best initial diagnostic test for Mumps

Clinical diagnosis

Best initial diagnostic test for Roseola

Clinical diagnosis

Best initial diagnostic test for Rubeola/Measles

Clinical diagnosis

How is bronchitis diagnosed?

Clinical diagnosis

How is diagnosis of Caput succedaneum & Cephalohematoma made?

Clinically

How is the diagnosis of Duchenne Erb paralysis made?

Clinically

How is the diagnosis of Scarlet fever made?

Clinically

How is the diagnosis of facial nerve palsy made?

Clinically

How is diagnosis of Epiglottitis made?

Clinically + radiology

How is the diagnosis of Croup made?

Clinically + radiology (if mild symptoms)

Most accurate diagnostic test for Wilms tumor

Contrast-enhanced CT

Classic presentation of Meckel's diverticulum?

Copious, painless, bright red rectal bleeding

Associate the *steeple sign* w/ its associated condition

Croup

Infectious upper airway condition characterized by severe inflammation

Croup

How is Diphtheria diagnosed?

Culture of superficial membrane

Treatment of galactosemia

Cut out all lactose-containing products

A fairly late sign of respiratory failure and is most frequently seen in the mucous membranes of the mouth and nail beds. Peripheral _____ alone is more likely due to shock than respiratory failure

Cyanosis

Autosomal disorder causing abnormally thick mucus

Cystic fibrosis

A result of tiring and is not necessarily a sign of improvement. IT is an ominous sign and usually means tired

Decreasing respiratory rate

Cause of WAGR syndrome

Deletion on chromosome 11

Type of skull fracture that can cause further cortical damage without surgical intervention

Depressed

90+ 2(age) =systolic Highest 70+ 2(age) = Systolic Lowest 2/3 = diastolic

Determining BP

What do infants and children use to breath more than adults?

Diaphragm

Although pedi's have a tidal volume proportionally similar to adolescents and adults. They require ___ ____ _____? Due to this and less oxygen store they are more susceptible to hypoxia.

Double the metabolic oxygen

Genetic condition associated w/ Hirschsprung disease

Down syndrome

Genetic condition associated w/ imperforate anus

Down syndrome

What is done after clamping and cutting the umbilical cord?

Dry, wrap, and place newborn under warmer

Rationale for antibiotic coverage against anaerobes in the setting of esophageal atresia + TEF

Due to high risk of lung abscess formation secondary to aspiration

Most common complaint w/ varicocele

Dull ache + heaviness in the scrotum

Condition in which a lack/absence of apoptosis leads to improper canalization of the lumen of the duodenum

Duodenal atresia

What is the most likely diagnosis in: a 1-day-old child w/ dark green vomiting after 1st feeding, oblique eye fissues w/ epicanthic skin folds, single palmar crease, holosystolic murmur, double bubble sign on CXR?

Duodenal atresia

What is missing in sufficient quantities to make vitamin K in a neonate?

E.coli in neonate's colonic flora

______ has broadly advanced the state of pediatric emergency care nationwide. It has improved the availability of child-size equipment in ambulance and ED's; initiated hundreds of programs to prevent injuries; and provided thousands of hours oftraining to EMT's, paramedics, and other emergency meical care providors.

EMSC (Emergency Medical Services for Children)

Why is it important to use a "treat as you go" approach when treating children?

Each body system may adversely affect another body system causing an inaccurate assessment

Signs and symptoms of TAPVR w/ obstruction

Early in life w/, respiratory distress & severe cyanosis

Treatment of Truncus arteriosus

Early surgery

Best initial diagnostic test for VSD

Echocardiogram

Most accurate diagnostic test for Hypoplastic left heart syndrome

Echocardiogram

Best initial diagnostic test for PDA

Echocardiography

Most accurate diagnostic test for TAPVR without obstruction

Echocardiography

Omphalocele is associated w/ what?

Edwards syndrome (trisomy 18)

Process in which a left-to-right shunt caused by a VSD reverses into a right-to-left shunt due to hypertrophy of the right ventricle

Eisenmenger syndrome

What could a right ventricular hypertrophy from left-to-right shunting due to VSD eventually lead to?

Eisenmenger syndrome

Best initial therapy for volvulus

Emergent endoscopic decompression

Most effective therapy for volvulus (typically done when endoscopic decompression fails)

Emergent surgical decompression

Most common congenital heart defect in Down syndrome

Endocardial cushion defect of AV canal

You arrive on the scene of a four-year-old child who has fallen on the playground. The child is sitting on the ground with a teacher. The child is very upset; it appears he has a broken arm. What is the best position for you to be in when you begin interacting with the patient?

Ensure that you get to the child's level

IV: 0.01mg/kg 1:10,000 ET: 0.1mg/kg 1:1000

Epi dose: IV ET tube

There is stridor arrest, reduction in airflow, retractions.

Epi should be given for airway emergencies when....

Children's tracheas are shorter and narrower than adults'. What are some other parts of a child's airway which are proportionally different from an adult's?

Epiglottis and larynx

Higher than croup

Epiglottis will have a fever

Severe, life-threatening swelling of the epiglottis and arytenoids due to H. influenza type B

Epiglottitis

Truest medical emergency in pediatrics

Epiglottitis

_____ usually begins suddenly. Typically, the child goes to bed without any symptoms and wakes up complaining of a sore throat and pain on swallowing. The child may have a fever, muffled voice, and drooling from the potential saliva that occurs because difficult and painful swallowing

Epiglottitis Signs are: Drooling, Stridor, & Sudden onset of high fever & dysphagia

which drugs are appropriate for an unresponsive child who became bradycardic after endotracheal suctioning after cardiopulmonary resuscitation has been initiated

Epinephrine and Atropine

Opening of the urethra is found on the dorsal surface

Epispadias

Type of esophageal fistula present in 80-90% of cases

Esophageal atresia+ tracheoesophageal fistula (TEF)

Who receives the Hepatitis B vaccination?

Every child

Reasoning for sigmoidoscopy in the setting of gastroenteritis

Examine for pseudomembranes in setting of C. difficile

Rationale for hearing test

Exclude congenital sensory-neural hearing loss

What is the most likely diagnosis in: an unresponsive 6-month-old presenting w/ seizures, fever of 103℉ for the last 3 days, markedly delayed capillary refill, and blood pressure of 80/20?

Febrile seizure secondary to sepsis

The most common cause of new onset of seizure in children is _____

Fever

Electrolyte abnormalities associated w/ 11β-hydroxylase deficiency

Few

ABD thrusts

FBAO for a child older than 1 year...

5 back blows than 5 chest thrusts. check mouth after every round

FBAO for an infant....

Most accurate diagnostic test for syphilis

FTA-ABS or dark field microscopy

Paralysis of structures innervated by the facial nerve caused by trauma secondary to forcep use in delivery

Facial nerve palsy

When trying to help the newborn maintain body temperature, the majority of heat loss can be prevented by covering the newborn's

Head

You are responding to child who has fallen from a tree house. Which part of the child's body is most likely to hit the ground first due to anatomical differences?

Head

In a normal delivery, the paramedic should suction the infants mouth and nose with a bulb syringe after the

Head appears, between contractions

What is a sign of respiratory distress in a child

Head bobbing

What are some of the physiological changes that occur as a child develops?

Heart and respiratory rates; blood pressure increases

Which vital sign is the most reliable indicator of shock in a pediatric patient?

Heart rate

Condition in which the opening to the anus is missing and the rectum ends in a blind pouch w/ conservation of the sphincter

Imperforate anus

Cause of vitamin D-*dependent* rickets

Inability to convert 25-OH to 1,25(OH)2

What is the difference between the skeletal structure of a pediatric patient and an adult?

Incomplete calcification of the skeleton and weaker cartilaginous growth plates

Most common cause for elevated AFP

Incorrect dating

`The fontanelle may be tight and bulging, thus causing the pulsation to diminish or disappear. Common in head trauma or meningitis

Increased ICP fontanelle

Treatment of PDA

Indomethacin

Ingestions usually occur in

Infant, & Toddler

Febrile seizures usually occurs in

Infant, Toddler, & Preschoolers

Falls usually occur in

Infant, Toddler, & School-Age children

Age group affected by congenital hip dysplasia

Infants

Nose breathers

Infants are obligated....

Which age group is considered "obligate nose-breathers?"

Infants under 6 months old

Etiology of pharyngitis

Inflammation of pharynx + adjacent structures caused by *Group A β-hemolytic Streptococci*

Where is the absent testicle w/ cryptorchidism typically found?

Inguinal canal (90% of cases)

Cause of X-linked hypophosphatemic rickets

Innate kidney defect results in the inability to retain phosphate

Most common wrong answer in management of an IDM

Insulin therapy

Opsomyoclonus

Intermittent jerky eye movements + myoclonic jerks and cerebellar ataxia

Treatment of slipped capital femoral epiphysis

Internal fixation w/ pinning

Effect of 21-hydroxylase deficiency on sex development of girls

Virilized

Pathognomonic AXR findings for necrotizing enterocolitis

Pneumatosis intestinalis (air within bowel wall)

These are more common in children following barotrauma

Pneumothorax

Too much fluid secondary to fetus not swallowing

Polyhydramnios

W/ a Bochdalek diaphragmatic hernia, where is the defect?

Posterolateral

What is renal agenesis associated w/?

Potter syndrome

What is a risk factor associated with a higher incidence if SIDS

Premature birth and low birth weight

Drowning or near drowning usually occurs in

Preschooler & School-Age children

Which age group fears bodily injury & mutilation & interprets words literally

Preschoolers

Which type of ASD presents w/ concomitant mitral valve abnormalities?

Primum defect

Presenting symptom of pyloric stenosis

Projectile vomitus

What condition is not likely to cause seizures

Prolonged dehydration

Without an ASD, where would a LE clot embolize?

Pulmonary circulation

What is the most appropriate next step in management of: a 2-year-old child brought in for severe cough (seallike barking), fever, runny nose, respiratory distress, refusal to lie flat, positive steeple sign on CXR?

Racemic epinephrine

Appearance of rickets on CXR

Rachitic rosary-like appearance of costochondral joints w/ cupping & fraying of the epiphyses

Sign observed on upper GI series in pyloric stenosis characterized by *excess mucosa in the pyloric lumen resulting in 2 columns of barium*

Railroad track sign

How is pharyngitis diagnosed?

Rapid DNAse antigen detection test

Cardiac output is dependent on what?

Rate

Which exam strategy can help reduce anxiety in school-age children

Reassure them that they are not being punished

Grimace in APGAR measures

Reflex irritability

What can happen to a child's developmental age during a stressful situation?

Regression

Most important next step in diarrhea and gastroenteritis

Rehydration

Compression of the rib cage by passing through the mother's vaginal canal helps accomplish what?

Removal of fluid from the lungs

Prognosis of larger VSDs

Require surgical intervention

After IV administration of diazepam, you should monitor closely for

Respiratory Depression

A child's body will often mask injuries or illness due to compensatory rates. What are some of the vital signs affected by compensatory rates?

Respiratory and heart rates

What the the 5 minute APGAR score evaluate?

Response to resuscitative efforts

Goal of surgical intervention in TAPVR without obstruction

Restore proper blood flow

A 4 year old girl took 10 tricyclic antidepressant tablets. Her BP is 60 mm Hg, by palpation, P is 130/min, and she is drowsy. What intervention is indicated to improve her cardiac output

Sodium bicarbonate (1 mEq/kg)

Which anatomical difference causes children to be more susceptible to blunt trauma?

Soft chest wall

What is the difference between an adult chest wall and a pediatric chest wall?

Softer

What is a succussion splash?

Sound of stomach contents slapping into the pylorus

In the middle of the night with no prior infection

Spasmodic croup occurs....

Which of the following is an important quality about your voice that you should keep in mind when speaking to a pediatric patient?

Speaking calmly and quietly

Normal PE finding in transient polycythemia of the newborn

Splenomegaly

Treat these as fractures and immobilize them accordingly. Until children reach adolescence, they have softer more porous bones than adults.

Sprains and strains

AAP recommendations for vitamin D supplementation

Start at 2 months of age (in infants exclusively breastfeeding)

What is the most likely diagnosis in: a 5-year-old boy presenting w/ SOB on exertion, bluish hue to his lips after exertion, and squatting while playing outside during recess?

Tetralogy of Fallot

Why is surgical intervention indicated for umbilical hernia past age 4?

To prevent bowel strangulation and necrosis

Rationale for fluid resuscitation prior to surgery in the setting of esophageal atresia + TEF

To prevent dehydration

Why is early surgery critical in the treatment of Truncus arteriosus?

To prevent pulmonary hypertension

A 1-3 year old is most correctly referred to as a(n)

Toddler

8 years old

Uncuffed ET tubes should be used in children under...

What should an unresponsive 6-month-old presenting w/ seizures, fever of 103℉ for the last 3 days, markedly delayed capillary refill, and blood pressure of 80/20 be evaluated for?

Underlying cause of sepsis

Proceed with the rapid secondary assessment, if trauma is suspected then transport is immediately with further assessment and treatment performed en route

Urgent Transport

Epispadias is associated w/?

Urinary incontinence

Most accurate diagnostic test for CMV

Urine/saliva PCR

Best initial diagnostic test for CMV

Urine/saliva viral titers

Which method of palpation is recommended when assessing a pediatric abdomen?

Use the side of your hand and rotate accross the abdomen

Congenital heart defects common in Down (trisomy 21), Edwards (trisomy 18), & Patau (trisomy 13) syndromes?

VSD

Most common congenital heart lesion

VSD

Etiology of Varicella

Varicella zoster virus

Presentation of Herpes in the 2nd week

Vesicular skin lesions

You are on the scene of a two-car, head-on MVC. You are assigned a six-year-old female patient. She was restrained and her airbag deployed. Which injury pattern should you expect?

a. Lower extremities and facial injuries b. Upper and lower extremity fractures c. Head, cervical, eye, and chest injuries

You are on the scene of a child struck by a vehicle traveling at a high rate of speed. Which injuries are associated with this type of mechanism?

a. Lower extremities, head, neck, abdomen b. Lower extremities, head, facial, abdomen c. Upper and lower extremities, head and facial

How do the unique anatomical features of a pediatric patient affect your assessment?

a. Smaller airways require you always assess breathing before airway b. Larger heads, smaller airways and parental separation and anxiety c. Consider predictable injury patterns based on mechanism of injury

1:1,000

concentration for sub-Q or IM Epinephrine in an allergic reaction

1:10,000

concentration for sub-Q or IM Epinephrine in a severe allergic reaction

Mild contraction and mild anxiety

moderate croup presents with....


Conjuntos de estudio relacionados

History of Photography Final Images!

View Set

Chapter 34 - Male Reproductive System

View Set

PSYCH107: Chapter 4 Quiz, pages 121-132

View Set

Marketing Quiz 1 (Chapter 16: Sustainability)

View Set

"Pedagogy of the Oppressed" by Freire

View Set